You are on page 1of 154

THE MANILA WINE MERCHANTS, INC.

, Petitioner,
v. THE COMMISSIONER OF INTERNAL REVENUE, Respondent.
SYLLABUS
1. TAXATION; NATIONAL INTERNAL REVENUE CODE; CORPORATE INCOME TAX;
ADDITIONAL TAX ON ACCUMULATED EARNINGS; EXEMPTION THEREFROM. A
prerequisite to the imposition of the tax has been that the corporation be formed or availed of for
the purpose of avoiding the income tax (or surtax) on its shareholders, or on the shareholders of
any other corporation by permitting the earnings and profits of the corporation to accumulate
instead of dividing them among or distributing them to the shareholders. If the earnings and
profits were distributed, the shareholders would be required to pay an income tax thereon
whereas, if the distribution were not made to them, they would incur no tax in respect to the
undistributed earnings and profits of the corporation (Mertens, Law on Federal Income Taxation,
Vol. 7, Chapter 39, p. 44). The touchstone of liability is the purpose behind the accumulation of
the income and not the consequences of the accumulation (Ibid., p. 47). Thus, if the failure to
pay dividends is due to some other cause, such as the use of undistributed earnings and profits
for the reasonable needs of the business, such purpose does not fall within the interdiction of
the statute (Ibid., p. 45).
2. ID.; ID.; ID.; ID.; ID.; WHEN ACCUMULATION CONSIDERED UNREASONABLE. An
accumulation of earnings or profits (including undistributed earnings or profits of prior years) is
unreasonable if it is not required for the purpose of the business, considering all the
circumstances of the case (Sec. 21, Revenue Regulations No. 2).
3. ID.; ID.; ID.; ID.; ID.; "REASONABLE NEEDS OF THE BUSINESS," CONSTRUED. To
determine the "reasonable needs" of the business in order to justify an accumulation of
earnings, the Courts of the United States have invented the so-called "Immediacy Test" which
construed the words "reasonable needs of the business" to mean the immediate needs of the
business, and it was generally held that if the corporation did not prove an immediate need for
the accumulation of the earnings and profits, the accumulation was not for the reasonable
needs of the business, and the penalty tax would apply. American cases likewise hold that
investment of the earnings and profits of the corporation in stock or securities of an unrelated
business usually indicates an accumulation beyond the reasonable needs of the business.
(Helvering v. Chicago Stockyards Co., 318 US 693; Helvering v. National Grocery Co., 304 US
282).
4. REMEDIAL LAW; APPEALS; FACTUAL FINDINGS OF THE COURT OF TAX APPEALS,
BINDING. The finding of the Court of Tax Appeals that the purchase of the U.S.A. Treasury
bonds were in no way related to petitioners business of importing and selling wines whisky,
liquors and distilled spirits, and thus construed as an investment beyond the reasonable needs
of the business is binding on Us, the same being factual (Renato Raymundo v. Hon. De Jova,
101 SCRA 495). Furthermore, the wisdom behind thus finding cannot be doubted, The case of
J.M. Perry & Co. v. Commissioner of Internal Revenue supports the same.
5. TAXATION; NATIONAL INTERNAL REVENUE CODE; INCOME TAX OF CORPORATIONS;
ADDITIONAL TAX ON ACCUMULATED EARNINGS; EXCEPTION THEREFROM;
ACCUMULATION OF EARNINGS, MUST BE USED FOR REASONABLE NEEDS OF
BUSINESS WITHIN A REASONABLE TIME. The records further reveal that from May 1951
when petitioner purchased the U.S.A. Treasury shares, until 1962 when it finally liquidated the
same, it (petitioner) never had the occasion to use the said shares in aiding or financing its
importation. This militates against the purpose enunciated earlier by petitioner that the shares
were purchased to finance its importation business. To justify an accumulation of earnings and
1

profits for the reasonably anticipated future needs, such accumulation must be used within a
reasonable time after the close of the taxable year (Mertens, Ibid., p. 104).
6. ID.; ID.; ID.; ID.; ID.; ID.; INTENTION AT THE TIME OF ACCUMULATION, BASIS OF THE
TAX; ACCUMULATION OF PROFITS IN CASE AT BAR, UNREASONABLE. In order to
determine whether profits are accumulated for the reasonable needs of the business as to avoid
the surtax upon shareholders, the controlling intention of the taxpayer is that which is
manifested at the time of accumulation not subsequently declared intentions which are merely
the product of afterthought (Basilan Estates, Inc. v. Comm. of Internal Revenue, 21 SCRA 17
citing Jacob Mertens, Jr., The law of Federal Income Taxation, Vol. 7, Cumulative Supplement,
p. 213; Smoot and San & Gravel Corp. v. Comm., 241 F 2d 197). A speculative and indefinite
purpose will not suffice. The mere recognition of a future problem and the discussion of possible
and alternative solutions is not sufficient. Definiteness of plan coupled with action taken towards
its consummation are essential (Fuel Carriers, Inc. v. US 202 F supp. 497; Smoot Sand &
Gravel Corp. v. Comm., supra). Viewed on the foregoing analysis and tested under the
"immediacy doctrine," We are convinced that the Court of Tax Appeals is correct in finding that
the investment made by petitioner in the U.S.A. Treasury shares in 1951 was an accumulation
of profits in excess of the reasonable needs of petitioners business.chanroblesvirtuallawlibrary
7. ID.; ID.; ID.; ID.; ACCUMULATIONS OF PRIOR YEARS TAKEN INTO ACCOUNT IN
DETERMINATION OF LIABILITY THEREFOR. The rule is now settled in Our jurisprudence
that undistributed earnings or profits of prior years are taken into consideration in determining
unreasonable accumulation for purposes of the 25% surtax. The case of Basilan Estates, Inc. v.
Commissioner of Internal Revenue further strengthen this rule in determining unreasonable
accumulation for the year concerned. In determining whether accumulations of earnings or
profits in a particular year are within the reasonable needs of a corporation, it is necessary to
take into account prior accumulations, since accumulations prior to the year involved may have
been sufficient to cover the business needs and additional accumulations during the year
involved would not reasonably be necessary.
DECISION
GUERRERO, J.:
In this Petition for Review on Certiorari, Petitioner, the Manila Wine Merchants, Inc., disputes
the decision of the Court of Tax Appeals ordering it (petitioner) to pay respondent, the
Commissioner of Internal Revenue, the amount of P86,804.38 as 25% surtax plus interest
which represents the additional tax due petitioner for improperly accumulating profits or surplus
in the taxable year 1957 under Sec. 25 of the National Internal Revenue Code.
The Court of Tax Appeals made the following finding of facts, to wit:
"Petitioner, a domestic corporation organized in 1937, is principally engaged in the importation
and sale of whisky, wines, liquors and distilled spirits. Its original subscribed and paid capital
was P500,000.00. Its capital of P500,000.00 was reduced to P250,000.00 in 1950 with the
approval of the Securities and Exchange Commission but the reduction of the capital was never
implemented. On June 21, 1958, petitioners capital was increased to P1,000,000.00 with the
approval of the said Commission.
On December 31, 1957, herein respondent caused the examination of herein petitioners book
of account and found the latter of having unreasonably accumulated surplus of P428,934.32 for
the calendar year 1947 to 1957, in excess of the reasonable needs of the business subject to
the 25% surtax imposed by Section 25 of the Tax Code.

On February 26, 1963, the Commissioner of Internal Revenue demanded upon the Manila Wine
Merchants, Inc. payment of P126,536.12 as 25% surtax and interest on the latters
unreasonable accumulation of profits and surplus for the year 1957, computed as follows:
Unreasonable accumulation of surtax P428,934.42

25% surtax due thereon P107,234.00


Add: 1/2% monthly interest from June 20,
1959 to June 20, 1962 19,302.12

TOTAL AMOUNT DUE AND COLLECTIBLE P126,536.12


=========
Respondent contends that petitioner has accumulated earnings beyond the reasonable needs
of its business because the average ratio of the cash dividends declared and paid by petitioner
from 1947 to 1957 was 40.33% of the total surplus available for distribution at the end of each
calendar year. On the other hand, petitioner contends that in 1957, it distributed 100% of its net
earnings after income tax and part of the surplus for prior years. Respondent further submits
that the accumulated earnings tax should be based on 25% of the total surplus available at the
end of each calendar year while petitioner maintains that the 25% surtax is imposed on the total
surplus or net income for the year after deducting therefrom the income tax due.
The records show the following analysis of petitioners net income, cash dividends and earned
surplus for the years 1946 to 1957:

1Percentage of Dividends to Net Income Total Cash Net Income Balance After Income
Dividends After of Earned
Year Tax Paid Income Tax Surplus
1946 P 613,790.00 P 200,000. 32.58% P 234,104.81
1947 425,719.87 360,000. 84.56% 195,167.10
1948 415,591.83 375,000. 90.23% 272,991.38
1949 335,058.06 200,000. 59.69% 893,113.42
1950 399,698.09 600,000. 150.11% 234,987.07
1951 346,257.26 300,000. 86.64% 281,244.33
1952 196,161.97 200,000. 101.96% 277,406.30
1953 169,714.04 200,000. 117.85% 301,138.84
1954 238,124.85 250,000. 104.99% 289,262.69
3

1955 312,284.74 200,000. 64.04% 401,548.43


1956 374,240.28 300,000. 80.16% 475,788.71
1957 353,145.71 400,000. 113.27% 428,934.42

P4,179,787.36 P3,585.000. 85.77% P3,785.688.50
========== ========= ======= ==========

Another basis of respondent in assessing petitioner for accumulated earnings tax is its
substantial investment of surplus or profits in unrelated business. These investments are
itemized as follows:
1. Acme Commercial Co., Inc. P 27,501.00
2. Union Insurance Society of Canton 1,145.76
3. U.S.A. Treasury Bond 347,217.50
4. Wack Wack Golf & Country Club 1.00

375,865.26
=========
As to the investment of P27,501.00 made by petitioner in the Acme Commercial Co., Inc., Mr.
N.R.E. Hawkins, president of the petitioner corporation 2 explained as follows:
The first item consists of shares of Acme Commercial Co., Inc. which the Company acquired in
1947 and 1949. In the said years, we thought it prudent to invest in a business which patronizes
us. As a supermarket, Acme Commercial Co., Inc. is one of our best customers. The investment
has proven to be beneficial to the stockholders of this Company. As an example, the Company
received cash dividends in 1961 totalling P16,875.00 which was included in its income tax
return for the said year.
As to the investments of petitioner in Union Insurance Society of Canton and Wack Wack Golf
Club in the sums of P1,145.76 and P1.00, respectively, the same official of the petitionercorporation stated that: 3
The second and fourth items are small amounts which we believe would not affect this case
substantially. As regards the Union Insurance Society of Canton shares, this was a pre-war
investment, when Wise & Co., Inc., Manila Wine Merchants and the said insurance firm were
common stockholders of the Wise Bldg. Co.,, Inc. and the three companies were all housed in
the same building. Union Insurance invested in Wise Bldg. Co., Inc. but invited Manila Wine
Merchants, Inc. to buy a few of its shares.
4

As to the U.S.A. Treasury Bonds amounting to P347,217.50, Mr. Hawkins explained as follows:
4
With regards to the U.S.A. Treasury Bills in the amount of P347,217.50, in 1950, our balance
sheet for the said year shows the Company had deposited in current account in various banks
P629,403.64 which was not earning any interest. We decided to utilize part of this money as
reserve to finance our importations and to take care of future expansion including acquisition of
a lot and the construction of our own office building and bottling plant.
At that time, we believed that a dollar reserve abroad would be useful to the Company in
meeting immediate urgent orders of its local customers. In order that the money may earn
interest, the Company, on May 31, 1951 purchased US Treasury bills with 90-day maturity and
earning approximately 1% interest with the face value of US$175,000.00. US Treasury Bills are
easily convertible into cash and for the said reason they may be better classified as cash rather
than investments.
The Treasury Bills in question were held as such for many years in view of our expectation that
the Central Bank inspite of the controls would allow no-dollar licenses importations. However,
since the Central Bank did not relax its policy with respect thereto, we decided sometime in
1957 to hold the bills for a few more years in view of our plan to buy a lot and construct a
building of our own. According to the lease agreement over the building formerly occupied by us
in Dasmarias St., the lease was to expire sometime in 1957. At that time, the Company was
not yet qualified to own real property in the Philippines. We therefore waited until 60% of the
stocks of the Company would be owned by Filipino citizens before making definite plans. Then
in 1959 when the Company was already more than 60% Filipino owned, we commenced looking
for a suitable location and then finally in 1961, we bought the man lot with an old building on
Otis St., Paco, our present site, for P665,000.00. Adjoining smaller lots were bought later. After
the purchase of the main property, we proceeded with the remodelling of the old building and
the construction of additions, which were completed at a cost of P143,896.00 in April, 1962.
In view of the needs of the business of this Company and the purchase of the Otis lots and the
construction of the improvements thereon, most of its available funds including the Treasury
Bills had been utilized, but inspite of the said expenses the Company consistently declared
dividends to its stockholders. The Treasury Bills were liquidated on February 15, 1962.
Respondent found that the accumulated surplus in question were invested to unrelated
business which were not considered in the immediate needs of the Company such that the
25% surtax be imposed therefrom."
Petitioner appealed to the Court of Tax Appeals.
On the basis of the tabulated figures, supra, the Court of Tax Appeals found that the average
percentage of cash dividends distributed was 85.77% for a period of 11 years from 1946 to 1957
and not only 40.33% of the total surplus available for distribution at the end of each calendar
year actually distributed by the petitioner to its stockholders, which is indicative of the view that
the Manila Wine Merchants, Inc. was not formed for the purpose of preventing the imposition of
income tax upon its shareholders. 5

With regards to the alleged substantial investment of surplus or profits in unrelated business,
the Court of Tax Appeals held that the investment of petitioner with Acme Commercial Co., Inc.,
Union Insurance Society of Canton and with the Wack Wack Golf and Country Club are
5

harmless accumulation of surplus and, therefore, not subject to the 25% surtax provided in
Section 25 of the Tax Code. 6
As to the U.S.A. Treasury Bonds amounting to P347,217.50, the Court of Tax Appeals ruled that
its purchase was in no way related to petitioners business of importing and selling wines,
whisky, liquors and distilled spirits. Respondent Court was convinced that the surplus of
P347,217.50 which was invested in the U.S.A. Treasury Bonds was availed of by petitioner for
the purpose of preventing the imposition of the surtax upon petitioners shareholders by
permitting its earnings and profits to accumulate beyond the reasonable needs of business.
Hence, the Court of Tax Appeals modified respondents decision by imposing upon petitioner the
25% surtax for 1957 only in the amount of P86,804.38 computed as follows:
Unreasonable accumulation
of surplus P347,217.50

25% surtax due thereon P 86,804.38 7


On May 30, 1966, the Court of Tax Appeals denied the motion for reconsideration filed by
petitioner on March 30, 1966. Hence, this petition.
Petition assigns the following errors:
I
The Court of Tax Appeals erred in holding that petitioner was availed of for the purpose of
preventing the imposition of a surtax on its shareholders.
II
The Court of Tax Appeals erred in holding that petitioners purchase of U.S.A. Treasury Bills in
1951 was an investment in unrelated business subject to the 25% surtax in 1957 as surplus
profits improperly accumulated in the latter years.
III
The Court of Tax Appeals erred in not finding that petitioner did not accumulate its surplus
profits improperly in 1957, and in not holding that such surplus profits, including the so-called
unrelated investments, were necessary for its reasonable business needs.
IV
The Court of Tax Appeals erred in not holding that petitioner had overcome the prima facie
presumption provided for in Section 25(c) of the Revenue Code.
V
The Court of Tax Appeals erred in finding petition liable for the payment of the surtax of
P86,804.38 and in denying petitioners Motion for Reconsideration and/or New Trial.
The issues in this case can be summarized as follows: (1) whether the purchase of the U.S.A.
Treasury bonds by petitioner in 1951 can be construed as an investment to an unrelated
business and hence, such was availed of by petitioner for the purpose of preventing the
imposition of the surtax upon petitioners shareholders by permitting its earnings and profits to
6

accumulate beyond the reasonable needs of the business, and if so, (2) whether the penalty tax
of twenty-five percent (25%) can be imposed on such improper accumulation in 1957 despite
the fact that the accumulation occurred in 1951.
The pertinent provision of the National Internal Revenue Code reads as follows:
"Sec. 25. Additional tax on corporations improperly accumulating profits or surplus. (a)
Imposition of Tax. If any corporation, except banks, insurance companies, or personal
holding companies whether domestic or foreign, is formed or availed of for the purpose of
preventing the imposition of the tax upon its shareholders or members or the shareholders or
members of another corporation, through the medium of permitting its gains and profits to
accumulate instead of being divided or distributed, there is levied and assessed against such
corporation, for each taxable year, a tax equal to twenty-five per centum of the undistributed
portion of its accumulated profits or surplus which shall be in addition to the tax imposed by
section twenty-four and shall be computed, collected and paid in the same manner and subject
to the same provisions of law, including penalties, as that tax: Provided, that no such tax shall
be levied upon any accumulated profits or surplus, if they are invested in any dollar-producing or
dollar-saving industry or in the purchase of bonds issued by the Central Bank of the Philippines.
x

(c) Evidence determinative of purpose. The fact that the earnings of profits of a corporation
are permitted to accumulate beyond the reasonable needs of the business shall be
determinative of the purpose to avoid the tax upon its shareholders or members unless the
corporation, by clear preponderance of evidence, shall prove the contrary." (As amended by
Republic Act No. 1823).
As correctly pointed out by the Court of Tax Appeals, inasmuch as the provisions of Section 25
of the National Internal Revenue Code were bodily lifted from Section 102 of the U.S. Internal
Revenue Code of 1939, including the regulations issued in connection therewith, it would be
proper to resort to applicable cases decided by the American Federal Courts for guidance and
enlightenment.
A prerequisite to the imposition of the tax has been that the corporation be formed or availed of
for the purpose of avoiding the income tax (or surtax) on its shareholders, or on the
shareholders of any other corporation by permitting the earnings and profits of the corporation to
accumulate instead of dividing them among or distributing them to the shareholders. If the
earnings and profits were distributed, the shareholders would be required to pay an income tax
thereon whereas, if the distribution were not made to them, they would incur no tax in respect to
the undistributed earnings and profits of the corporation. 8 The touchstone of liability is the
purpose behind the accumulation of the income and not the consequences of the accumulation.
9 Thus, if the failure to pay dividends is due to some other cause, such as the use of
undistributed earnings and profits for the reasonable needs of the business, such purpose does
not fall within the interdiction of the statute. 10
An accumulation of earnings or profits (including undistributed earnings or profits of prior years)
is unreasonable if it is not required for the purpose of the business, considering all the
circumstances of the case. 11
In purchasing the U.S.A. Treasury Bonds, in 1951, petitioner argues that these bonds were so
purchased (1) in order to finance their importation; and that a dollar reserve abroad would be
useful to the Company in meeting urgent orders of its local customers and (2) to take care of
future expansion including the acquisition of a lot and the construction of their office building and
bottling plant.
7

We find no merit in the petition.


To avoid the twenty-five percent (25%) surtax, petitioner has to prove that the purchase of the
U.S.A. Treasury Bonds in 1951 with a face value of $175,000.00 was an investment within the
reasonable needs of the Corporation.
To determine the "reasonable needs" of the business in order to justify an accumulation of
earnings, the Courts of the United States have invented the so-called "Immediacy Test" which
construed the words "reasonable needs of the business" to mean the immediate needs of the
business, and it was generally held that if the corporation did not prove an immediate need for
the accumulation of the earnings and profits, the accumulation was not for the reasonable
needs of the business, and the penalty tax would apply. 12 American cases likewise hold that
investment of the earnings and profits of the corporation in stock or securities of an unrelated
business usually indicates an accumulation beyond the reasonable needs of the business. 13
The finding of the Court of Tax Appeals that the purchase of the U.S.A. Treasury bonds were in
no way related to petitioners business of importing and selling wines whisky, liquors and
distilled spirits, and thus construed as an investment beyond the reasonable needs of the
business 14 is binding on Us, the same being factual. 15 Furthermore, the wisdom behind thus
finding cannot be doubted, The case of J.M. Perry & Co. v. Commissioner of Internal Revenue
16 supports the same. In that case, the U.S. Court said the following:
"It appears that the taxpayer corporation was engaged in the business of cold storage and
warehousing in Yahima, Washington. It maintained a cold storage plant, divided into four units,
having a total capacity of 490,000 boxes of fruits. It presented evidence to the effect that various
alterations and repairs to its plant were contemplated in the tax years, . . .
It also appeared that in spite of the fact that the taxpayer contended that it needed to maintain
this large cash reserve on hand, it proceeded to make various investments which had no
relation to its storage business. In 1934, it purchased mining stock which it sold in 1935 at a
profit of US $47,995.29. . . .
All these things may reasonably have appealed to the Board as incompatible with a purpose to
strengthen the financial position of the taxpayer and to provide for needed alteration."
The records further reveal that from May 1951 when petitioner purchased the U.S.A. Treasury
shares, until 1962 when it finally liquidated the same, it (petitioner) never had the occasion to
use the said shares in aiding or financing its importation. This militates against the purpose
enunciated earlier by petitioner that the shares were purchased to finance its importation
business. To justify an accumulation of earnings and profits for the reasonably anticipated future
needs, such accumulation must be used within a reasonable time after the close of the taxable
year. 17
Petitioner advanced the argument that the U.S.A. Treasury shares were held for a few more
years from 1957, in view of a plan to buy a lot and construct a building of their own; that at that
time (1957), the Company was not yet qualified to own real property in the Philippines, hence it
(petitioner) had to wait until sixty percent (60%) of the stocks of the Company would be owned
by Filipino citizens before making definite plans. 18
These arguments of petitioner indicate that it considers the U.S.A. Treasury shares not only for
the purpose of aiding or financing its importation but likewise for the purpose of buying a lot and
constructing a building thereon in the near future, but conditioned upon the completion of the
60% citizenship requirement of stock ownership of the Company in order to qualify it to
purchase and own a lot. The time when the company would be able to establish itself to meet
8

the said requirement and the decision to pursue the same are dependent upon various future
contingencies. Whether these contingencies would unfold favorably to the Company and if so,
whether the Company would decide later to utilize the U.S.A. Treasury shares according to its
plan, remains to be seen. From these assertions of petitioner, We cannot gather anything
definite or certain. This, We cannot approve.
In order to determine whether profits are accumulated for the reasonable needs of the business
as to avoid the surtax upon shareholders, the controlling intention of the taxpayer is that which
is manifested at the time of accumulation not subsequently declared intentions which are merely
the product of afterthought. 19 A speculative and indefinite purpose will not suffice. The mere
recognition of a future problem and the discussion of possible and alternative solutions is not
sufficient. Definiteness of plan coupled with action taken towards its consummation are
essential. 20 The Court of Tax Appeals correctly made the following ruling:
"As to the statement of Mr. Hawkins in Exh. "B" regarding the expansion program of the
petitioner by purchasing a lot and building of its own, we find no justifiable reason for the
retention in 1957 or thereafter of the US Treasury Bonds which were purchased in 1951.
x

"Moreover, if there was any thought for the purchase of a lot and building for the needs of
petitioners business, the corporation may not with impunity permit its earnings to pile up merely
because at some future time certain outlays would have to be made. Profits may only be
accumulated for the reasonable needs of the business, and implicit in this is further requirement
of a reasonable time."
Viewed on the foregoing analysis and tested under the "immediacy doctrine," We are convinced
that the Court of Tax Appeals is correct in finding that the investment made by petitioner in the
U.S.A. Treasury shares in 1951 was an accumulation of profits in excess of the reasonable
needs of petitioners business.
Finally, petitioner asserts that the surplus profits allegedly accumulated in the form of U.S.A.
Treasury shares in 1951 by it (petitioner) should not be subject to the surtax in 1957. In other
words, petitioner claims that the surtax of 25% should be based on the surplus accumulated in
1951 and not in 1957.
This is devoid of merit.
The rule is now settled in Our jurisprudence that undistributed earnings or profits of prior years
are taken into consideration in determining unreasonable accumulation for purposes of the 25%
surtax. 22 The case of Basilan Estates, Inc. v. Commissioner of Internal Revenue 23 further
strengthen this rule, and We quote:
"Petitioner questions why the examiner covered the period from 1948-1953 when the taxable
year on review was 1953. The surplus of P347,507.01 was taken by the examiner from the
balance sheet of the petitioner for 1953. To check the figure arrived at, the examiner traced the
accumulation process from 1947 until 1953, and petitioners figure stood out to be correct.
There was no error in the process applied, for previous accumulations should be considered in
determining unreasonable accumulation for the year concerned. In determining whether
accumulations of earnings or profits in a particular year are within the reasonable needs of a
corporation, it is necessary to take into account prior accumulations, since accumulations prior
to the year involved may have been sufficient to cover the business needs and additional
accumulations during the year involved would not reasonably be necessary."
9

WHEREFORE, IN VIEW OF THE FOREGOING, the decision of the Court of Tax Appeals is
AFFIRMED in toto, with costs against petitioner.
SO ORDERED.

10

Republic of the Philippines


SUPREME COURT
Manila
FIRST DIVISION
G.R. No. 85749 May 15, 1989
COMMISSIONER OF INTERNAL REVENUE, petitioner,
vs.
ANTONIO TUASON, INC. and THE COURT OF TAX APPEALS, respondents.
GRIO-AQUINO, J.:
Elevated to this Court for review is the decision dated October 14, 1988 of the Court of Tax
Appeals in CTA Case No. 3865, entitled "Antonio Tuason, Inc. vs. Commissioner of Internal
Revenue," which set aside the petitioner Revenue Commissioner's assessment of
P1,151,146.98 as the 25% surtax on the private respondent's unreasonable accumulation of
surplus for the years 1975-1978.
Under date of February 27, 1981, the petitioner, Commissioner of Internal Revenue, assessed
Antonio Tuason, Inc.
a. Deficiency income tax for the years 1975,1976 and 1978 . . . . . . .
.. P37,491.83.
(b) Deficiency corporate quarterly income tax for the first quarter of 1975 . . . . .
. . . . . . . . . . . . . . . . 161.49.
(c) 25% surtax on unreasonable accumulation of surplus for the years 1975-1978
. . . . . . . . . . . . 1,151,146.98.
The private respondent did not object to the first and second items and, therefore, paid the
amounts demanded. However, it protested the assessment on a 25% surtax on the third item on
the ground that the accumulation of surplus profits during the years in question was solely for
the purpose of expanding its business operations as real estate broker. The request for
reinvestigation was granted on condition that a waiver of the statute of limitations should be filed
by the private respondent. The latter replied that there was no need of a waiver of the statute of
limitations because the right of the Government to assess said tax does not prescribe.
No investigation was conducted nor a decision rendered on Antonio Tuazon Inc.'s protest.
meantime, the Revenue Commissioner issued warrants of distraint and levy to enforce
collection of the total amount originally assessed including the amounts already paid.
The private respondent filed a petition for review in the Court of Tax Appeals with a request that
pending determination of the case on the merits, an order be issued restraining the
Commissioner and/or his representatives from enforcing the warrants of distraint and levy. Since
the right asserted by the Commissioner to collect the taxes involved herein by the summary
methods of distraint and levy was not clear, and it was shown that portions of the tax liabilities
involved in the assessment had already been paid, a writ of injunction was issued by the Tax
Court on November 26, 1984, ordering the Commissioner to refrain fron enforcing said warrants
of distraint and levy. It did not require the petitioner to file a bond (Annex A, pp. 28-30, Rollo).
11

In view of the reversal of the Commissioner's decision by the Court of Tax Appeals, the
petitioner appealed to this Court, raising the following issues:
1. Whether or not private respondent Antonio Tuason, Inc. is a holding company
and/or investment company;
2. Whether or not privaaate respondent Antonio Tuason, Inc. accumulated
surplus for the years 1975 to 1978; and
3. Whether or not Antonio Tuason, Inc. is liable for the 25% surtax on undue
accumulation of surplus for the years 1975 to 1978.
Section 25 of the Tax Code at the time the surtax was assessed, provided:
Sec. 25. Additional tax on corporation improperly accumulating profits or surplus.

(a) Imposition of tax. If any corporation, except banks, insurance companies,


or personal holding companies, whether domestic or foreign, is formed or availed
of for the purpose of preventing the imposition of the tax upon its shareholders or
members or the shareholders or members of another corporation, through the
medium of permitting its gains and profits to accumulate instead of being divided
or distributed, there is levied and assessed against such corporation, for each
taxable year, a tax equal to twenty-five per centum of the undistributed portion of
its accumulated profits or surplus which shall be in addition to the tax imposed by
section twenty-four, and shall be computed, collected and paid in the same
manner and subject to the same provisions of law, including penalties, as that
tax.
(b) Prima facie evidence. The fact that any corporation is a mere holding
company shall be prima facie evidence of a purpose to avoid the tax upon its
shareholders or members. Similar presumption will lie in the case of an
investment company where at any time during the taxable year more than fifty
per centum in value of its outstanding stock is owned, directly or indirectly, by
one person.
(c) Evidence determinative of purpose. The fact that the earnings or profits of
a corporation are permitted to accumulate beyond the reasonable needs of the
business shall be determinative of the purpose to avoid the tax upon its
shareholders or members unless the corporation, by clear preponderance of
evidence, shall prove the contrary.
The petition for review is meritorious.
The Court of Tax Appeals conceded that the Revenue Commissioner's determination that
Antonio Tuason, Inc. was a mere holding or investment company, was "presumptively correct"
(p. 7, Annex A), for the corporation did not involve itself in the development of subdivisions but
merely subdivided its own lots and sold them for bigger profits. It derived its income mostly from
interest, dividends and rental realized from the sale of realty.
Another circumstance supporting that presumption is that 99.99% in value of the outstanding
stock of Antonio Tuason, Inc., is owned by Antonio Tuason himself. The Commissioner
12

"conclusively presumed" that when the corporation accumulated (instead of distributing to the
shareholders) a surplus of over P3 million fron its earnings in 1975 up to 1978, the purpose was
to avoid the imposition of the progressive income tax on its shareholders.
That Antonio Tuason, Inc. accumulated surplus profits amounting to P3,263,305.88 for 1975 up
to 1978 is not disputed. However, the private respondent vehemently denies that its purpose
was to evade payment of the progressive income tax on such dividends by its stockholders.
According to the private respondent, surplus profits were set aside by the company to build up
sufficient capital for its expansion program which included the construction in 1979-1981 of an
apartment building, and the purchase in 1980 of a condominium unit which was intended for
resale or lease.
However, while these investments were actually made, the Commissioner points out that the
corporation did not use up its surplus profits. It allegation that P1,525,672.74 was spent for the
construction of an apartment building in 1979 and P1,752,332.87 for the purchase of a
condominium unit in Urdaneta Village in 1980 was refuted by the Declaration of Real Property
on the apartment building (Exh. C) which shows that its market value is only P429,890.00, and
the Tax Declaration on the condominium unit which reflects a market value of P293,830.00 only
(Exh. D-1). The enormous discrepancy between the alleged investment cost and the declared
market value of these pieces of real estate was not denied nor explained by the private
respondent.
Since the company as of the time of the assessment in 1981, had invested in its business
operations only P 773,720 out of its accumulated surplus profits of P3,263,305.88 for 19751978, its remaining accumulated surplus profits of P2,489,858.88 are subject to the 25% surtax.
All presumptions are in favor of the correctness of petitioner's assessment against the private
respondent. It is incumbent upon the taxpayer to prove the contrary (Mindanao Bus Company
vs. Commissioner of Internal Revenue, 1 SCRA 538). Unfortunately, the private respondent
failed to overcome the presumption of correctness of the Commissioner's assessment.
The touchstone of liability is the purpose behind the accumulation of the income and not the
consequences of the accumulation. Thus, if the failure to pay dividends were for the purpose of
using the undistributed earnings and profits for the reasonable needs of the business, that
purpose would not fall within the interdiction of the statute" (Mertens Law of Federal Income
Taxation, Vol. 7, Chapter 39, p. 45 cited in Manila Wine Merchants, Inc. vs. Commissioner of
Internal Revenue, 127 SCRA 483, 493).
It is plain to see that the company's failure to distribute dividends to its stockholders in 19751978 was for reasons other than the reasonable needs of the business, thereby falling within the
interdiction of Section 25 of the Tax Code of 1977.
WHEREFORE, the appealed decision of the Court of Tax Appeals is hereby set aside. The
petitioner's assessment of a 25% surtax against the Antonio Tuason, Inc. is reinstated but only
on the latter's unspent accumulated surplus profits of P2,489,585.88. No costs.
SO ORDERED.

Republic of the Philippines


SUPREME COURT
Manila
13

SECOND DIVISION
G.R. No. 108067

January 20, 2000

CYANAMID PHILIPPINES, INC., petitioner,


vs.
THE COURT OF APPEALS, THE COURT OF TAX APPEALS and COMMISSIONER OF
INTERNAL REVENUE,respondent.
QUISUMBING, J.:
Petitioner disputes the decision1 of the Court of Appeals which affirmed the decision2 of the
Court of Tax Appeals, ordering petitioner to pay respondent Commissioner of Internal Revenue
the amount of three million, seven hundred seventy-four thousand, eight hundred sixty seven
pesos and fifty centavos (P3,774,867.50) as 25% surtax on improper accumulation of profits for
1981, plus 10% surcharge and 20% annual interest from January 30, 1985 to January 30, 1987,
under Sec. 25 of the National Internal Revenue Code.
The Court of Tax Appeals made the following factual findings:
Petitioner, Cyanamid Philippines, Inc., a corporation organized under Philippine laws, is a wholly
owned subsidiary of American Cyanamid Co. based in Maine, USA. It is engaged in the
manufacture of pharmaceutical products and chemicals, a wholesaler of imported finished
goods, and an importer/indentor.
On February 7, 1985, the CIR sent an assessment letter to petitioner and demanded the
payment of deficiency income tax of one hundred nineteen thousand eight hundred seventeen
(P119,817.00) pesos for taxable year 1981, as follows:
Net income disclosed by the return as audited

14,575,210.00

Add: Discrepancies:
Professional fees/yr.

17018

per investigation

261,877.00
110,399.37

Total Adjustment

152,477.00

Net income per Investigation

14,727,687.00

Less: Personal and additional exemptions


Amount subject to tax
Income tax due thereon . . . 25% Surtax

14,727,687.00
2,385,231.50

Less: Amount already assessed

5,161,788.00

BALANCE
monthly interest from

3,237,495.00

75,709.00
1,389,639.00

44,108.00

Compromise penalties

14

TOTAL AMOUNT DUE

3,774,867.50

119,817.003

On March 4, 1985, petitioner protested the assessments particularly, (1) the 25% Surtax
Assessment of P3,774,867.50; (2) 1981 Deficiency Income Assessment of P119,817.00; and
1981 Deficiency Percentage Assessment of P8,846.72.4 Petitioner, through its external
accountant, Sycip, Gorres, Velayo & Co., claimed, among others, that the surtax for the undue
accumulation of earnings was not proper because the said profits were retained to increase
petitioner's working capital and it would be used for reasonable business needs of the company.
Petitioner contended that it availed of the tax amnesty under Executive Order No. 41, hence
enjoyed amnesty from civil and criminal prosecution granted by the law.
On October 20, 1987, the CIR in a letter addressed to SGV & Co., refused to allow the
cancellation of the assessment notices and rendered its resolution, as follows:
It appears that your client availed of Executive Order No. 41 under File No. 32A-F000455-41B as certified and confirmed by our Tax Amnesty Implementation Office on
October 6, 1987.
In reply thereto, I have the honor to inform you that the availment of the tax amnesty
under Executive Order No. 41, as amended is sufficient basis, in appropriate cases, for
the cancellation of the assessment issued after August 21, 1986. (Revenue
Memorandum Order No. 4-87) Said availment does not, therefore, result in cancellation
of assessments issued before August 21, 1986. as in the instant case. In other words,
the assessments in this case issued on January 30, 1985 despite your client's availment
of the tax amnesty under Executive Order No. 41, as amended still subsist.
Such being the case, you are therefore, requested to urge your client to pay this Office
the aforementioned deficiency income tax and surtax on undue accumulation of surplus
in the respective amounts of P119,817.00 and P3,774,867.50 inclusive of interest
thereon for the year 1981, within thirty (30) days from receipt hereof, otherwise this office
will be constrained to enforce collection thereof thru summary remedies prescribed by
law.
This constitutes the final decision of this Office on this matter.5
Petitioner appealed to the Court of Tax Appeals. During the pendency of the case, however,
both parties agreed to compromise the 1981 deficiency income tax assessment of P119,817.00.
Petitioner paid a reduced amount twenty-six thousand, five hundred seventy-seven pesos
(P26,577.00) as compromise settlement. However, the surtax on improperly accumulated
profits remained unresolved.
Petitioner claimed that CIR's assessment representing the 25% surtax on its accumulated
earnings for the year 1981 had no legal basis for the following reasons: (a) petitioner
accumulated its earnings and profits for reasonable business requirements to meet working
capital needs and retirement of indebtedness; (b) petitioner is a wholly owned subsidiary of
American Cyanamid Company, a corporation organized under the laws of the State of Maine, in
the United States of America, whose shares of stock are listed and traded in New York Stock
Exchange. This being the case, no individual shareholder income taxes by petitioner's
accumulation of earnings and profits, instead of distribution of the same.
In denying the petition, the Court of Tax Appeals made the following pronouncements:
15

Petitioner contends that it did not declare dividends for the year 1981 in order to use the
accumulated earnings as working capital reserve to meet its "reasonable business
needs". The law permits a stock corporation to set aside a portion of its retained
earnings for specified purposes (citing Section 43, paragraph 2 of the Corporation Code
of the Philippines). In the case at bar, however, petitioner's purpose for accumulating its
earnings does not fall within the ambit of any of these specified purposes.
More compelling is the finding that there was no need for petitioner to set aside a portion
of its retained earnings as working capital reserve as it claims since it had considerable
liquid funds. A thorough review of petitioner's financial statement (particularly the
Balance Sheet, p. 127, BIR Records) reveals that the corporation had considerable
liquid funds consisting of cash accounts receivable, inventory and even its sales for the
period is adequate to meet the normal needs of the business. This can be determined by
computing the current asset to liability ratio of the company:
current ratio

= current assets/ current liabilities


= P 47,052,535.00 / P21,275,544.00
= 2.21: 1
========

The significance of this ratio is to serve as a primary test of a company's solvency to


meet current obligations from current assets as a going concern or a measure of
adequacy of working capital.
xxx

xxx

xxx

We further reject petitioner's argument that "the accumulated earnings tax does not
apply to a publicly-held corporation" citing American jurisprudence to support its position.
The reference finds no application in the case at bar because under Section 25 of the
NIRC as amended by Section 5 of P.D. No. 1379 [1739] (dated September 17, 1980),
the exceptions to the accumulated earnings tax are expressly enumerated, to wit: Bank,
non-bank financial intermediaries, corporations organized primarily, and authorized by
the Central Bank of the Philippines to hold shares of stock of banks, insurance
companies, or personal holding companies, whether domestic or foreign. The law on the
matter is clear and specific. Hence, there is no need to resort to applicable cases
decided by the American Federal Courts for guidance and enlightenment as to whether
the provision of Section 25 of the NIRC should apply to petitioner.
Equally clear and specific are the provisions of E.O. 41 particularly with respect to its
effectivity and coverage . . .
. . . Said availment does not result in cancellation of assessments issued before August
21, 1986 as petitioner seeks to do in the case at bar. Therefore, the assessments in this
case, issued on January 30, 1985 despite petitioner's availment of the tax amnesty
under E.O. 41 as amended, still subsist.
xxx

xxx

xxx

WHEREFORE, petitioner Cyanamid Philippines, Inc., is ordered to pay respondent


Commissioner of Internal Revenue the sum of P3,774,867.50 representing 25% surtax
16

on improper accumulation of profits for 1981, plus 10% surcharge and 20% annual
interest from January 30, 1985 to January 30, 1987.6
Petitioner appealed the Court of Tax Appeal's decision to the Court of Appeals. Affirming the
CTA decision, the appellate court said:
In reviewing the instant petition and the arguments raised herein, We find no compelling
reason to reverse the findings of the respondent Court. The respondent Court's decision
is supported by evidence, such as petitioner corporation's financial statement and
balance sheets (p. 127, BIR Records). On the other hand the petitioner corporation
could only come up with an alternative formula lifted from a decision rendered by a
foreign court (Bardahl Mfg. Corp. vs. Commissioner, 24 T.C.M. [CCH] 1030). Applying
said formula to its particular financial position, the petitioner corporation attempts to
justify its accumulated surplus earnings. To Our mind, the petitioner corporation's
alternative formula cannot overturn the persuasive findings and conclusion of the
respondent Court based, as it is, on the applicable laws and jurisprudence, as well as
standards in the computation of taxes and penalties practiced in this jurisdiction.
WHEREFORE, in view of the foregoing, the instant petition is hereby DISMISSED and
the decision of the Court of Tax Appeals dated August 6, 1992 in C.T.A. Case No. 4250
is AFFIRMED in toto.7
Hence, petitioner now comes before us and assigns as sole issue:
WHETHER THE RESPONDENT COURT ERRED IN HOLDING THAT THE
PETITIONER IS LIABLE FOR THE ACCUMULATED EARNINGS TAX FOR THE YEAR
1981.8
Sec. 259 of the old National Internal Revenue Code of 1977 states:
Sec. 25. Additional tax on corporation improperly accumulating profits or surplus
(a) Imposition of tax. If any corporation is formed or availed of for the purpose of
preventing the imposition of the tax upon its shareholders or members or the
shareholders or members of another corporation, through the medium of permitting its
gains and profits to accumulate instead of being divided or distributed, there is levied
and assessed against such corporation, for each taxable year, a tax equal to twentyfive per-centum of the undistributed portion of its accumulated profits or surplus which
shall be in addition to the tax imposed by section twenty-four, and shall be computed,
collected and paid in the same manner and subject to the same provisions of law,
including penalties, as that tax.
(b) Prima facie evidence. The fact that any corporation is mere holding company shall
be prima facie evidence of a purpose to avoid the tax upon its shareholders or members.
Similar presumption will lie in the case of an investment company where at any time
during the taxable year more than fifty per centum in value of its outstanding stock is
owned, directly or indirectly, by one person.
(c) Evidence determinative of purpose. The fact that the earnings or profits of a
corporation are permitted to accumulate beyond the reasonable needs of the business
shall be determinative of the purpose to avoid the tax upon its shareholders or members
unless the corporation, by clear preponderance of evidence, shall prove the contrary.
17

(d) Exception. The provisions of this sections shall not apply to banks, non-bank
financial intermediaries, corporation organized primarily, and authorized by the Central
Bank of the Philippines to hold shares of stock of banks, insurance companies, whether
domestic or foreign.
The provision discouraged tax avoidance through corporate surplus accumulation. When
corporations do not declare dividends, income taxes are not paid on the undeclared dividends
received by the shareholders. The tax on improper accumulation of surplus is essentially a
penalty tax designed to compel corporations to distribute earnings so that the said earnings by
shareholders could, in turn, be taxed.
Relying on decisions of the American Federal Courts, petitioner stresses that the accumulated
earnings tax does not apply to Cyanamid, a wholly owned subsidiary of a publicly owned
company.10 Specifically, petitioner citesGolconda Mining Corp. vs. Commissioner, 507 F.2d 594,
whereby the U.S. Ninth Circuit Court of Appeals had taken the position that the accumulated
earnings tax could only apply to a closely held corporation.
A review of American taxation history on accumulated earnings tax will show that the application
of the accumulated earnings tax to publicly held corporations has been problematic. Initially, the
Tax Court and the Court of Claims held that the accumulated earnings tax applies to publicly
held corporations. Then, the Ninth Circuit Court of Appeals ruled in Golconda that the
accumulated earnings tax could only apply to closely held corporations. Despite Golconda, the
Internal Revenue Service asserted that the tax could be imposed on widely held corporations
including those not controlled by a few shareholders or groups of shareholders. The Service
indicated it would not follow the Ninth Circuit regarding publicly held corporations. 11 In 1984,
American legislation nullified the Ninth Circuit's Golconda ruling and made it clear that the
accumulated earnings tax is not limited to closely held corporations.12 Clearly, Golconda is no
longer a reliable precedent.
The amendatory provision of Section 25 of the 1977 NIRC, which was PD 1739, enumerated
the corporations exempt from the imposition of improperly accumulated tax: (a) banks; (b) nonbank financial intermediaries; (c) insurance companies; and (d) corporations organized primarily
and authorized by the Central Bank of the Philippines to hold shares of stocks of banks.
Petitioner does not fall among those exempt classes. Besides, the rule on enumeration is that
the express mention of one person, thing, act, or consequence is construed to exclude all
others.13 Laws granting exemption from tax are construed strictissimi juris against the taxpayer
and liberally in favor of the taxing power.14 Taxation is the rule and exemption is the
exception.15 The burden of proof rests upon the party claiming exemption to prove that it is, in
fact, covered by the exemption so claimed,16 a burden which petitioner here has failed to
discharge.
Another point raised by the petitioner in objecting to the assessment, is that increase of working
capital by a corporation justifies accumulating income. Petitioner asserts that respondent court
erred in concluding that Cyanamid need not infuse additional working capital reserve because it
had considerable liquid funds based on the 2.21:1 ratio of current assets to current liabilities.
Petitioner relies on the so-called "Bardahl" formula, which allowed retention, as working capital
reserve, sufficient amounts of liquid assets to carry the company through one operating cycle.
The "Bardahl"17 formula was developed to measure corporate liquidity. The formula requires an
examination of whether the taxpayer has sufficient liquid assets to pay all of its current liabilities
and any extraordinary expenses reasonably anticipated, plus enough to operate the business
during one operating cycle. Operating cycle is the period of time it takes to convert cash into raw
materials, raw materials into inventory, and inventory into sales, including the time it takes to
collect payment for the sales.18
18

Using this formula, petitioner contends, Cyanamid needed at least P33,763,624.00 pesos as
working capital. As of 1981, its liquid asset was only P25,776,991.00. Thus, petitioner asserts
that Cyanamid had a working capital deficit of P7,986,633.00. 19 Therefore, the P9,540,926.00
accumulated income as of 1981 may be validly accumulated to increase the petitioner's working
capital for the succeeding year.
We note, however, that the companies where the "Bardahl" formula was applied, had operating
cycles much shorter than that of petitioner. In Atlas Tool Co., Inc, vs. CIR,20 the company's
operating cycle was only 3.33 months or 27.75% of the year. In Cataphote Corp. of Mississippi
vs. United States,21 the corporation's operating cycle was only 56.87 days, or 15.58% of the
year. In the case of Cyanamid, the operating cycle was 288.35 days, or 78.55% of a year,
reflecting that petitioner will need sufficient liquid funds, of at least three quarters of the year, to
cover the operating costs of the business. There are variations in the application of the
"Bardahl" formula, such as average operating cycle or peak operating cycle. In times when
there is no recurrence of a business cycle, the working capital needs cannot be predicted with
accuracy. As stressed by American authorities, although the "Bardahl" formula is wellestablished and routinely applied by the courts, it is not a precise rule. It is used only for
administrative convenience.22 Petitioner's application of the "Bardahl" formula merely creates a
false illusion of exactitude.
Other formulas are also used, e.g. the ratio of current assets to current liabilities and the
adoption of the industry standard.23 The ratio of current assets to current liabilities is used to
determine the sufficiency of working capital. Ideally, the working capital should equal the current
liabilities and there must be 2 units of current assets for every unit of current liability, hence the
so-called "2 to 1" rule.24
As of 1981 the working capital of Cyanamid was P25,776,991.00, or more than twice its current
liabilities. That current ratio of Cyanamid, therefore, projects adequacy in working capital. Said
working capital was expected to increase further when more funds were generated from the
succeeding year's sales. Available income covered expenses or indebtedness for that year, and
there appeared no reason to expect an impending "working capital deficit" which could have
necessitated an increase in working capital, as rationalized by petitioner.
In Basilan Estates, Inc. vs. Commissioner of Internal Revenue,25 we held that:
. . . [T]here is no need to have such a large amount at the beginning of the following year
because during the year, current assets are converted into cash and with the income
realized from the business as the year goes, these expenses may well be taken care of.
[citation omitted]. Thus, it is erroneous to say that the taxpayer is entitled to retain
enough liquid net assets in amounts approximately equal to current operating needs for
the year to cover "cost of goods sold and operating expenses:" for "it excludes proper
consideration of funds generated by the collection of notes receivable as trade accounts
during the course of the year."26
If the CIR determined that the corporation avoided the tax on shareholders by permitting
earnings or profits to accumulate, and the taxpayer contested such a determination, the burden
of proving the determination wrong, together with the corresponding burden of first going
forward with evidence, is on the taxpayer. This applies even if the corporation is not a mere
holding or investment company and does not have an unreasonable accumulation of earnings
or profits.27
In order to determine whether profits are accumulated for the reasonable needs to avoid the
surtax upon shareholders, it must be shown that the controlling intention of the taxpayer is
19

manifest at the time of accumulation, not intentions declared subsequently, which are mere
afterthoughts.28 Furthermore, the accumulated profits must be used within a reasonable time
after the close of the taxable year. In the instant case, petitioner did not establish, by clear and
convincing evidence, that such accumulation of profit was for the immediate needs of the
business.
In Manila Wine Merchants, Inc. vs. Commissioner of Internal Revenue,29 we ruled:
To determine the "reasonable needs" of the business in order to justify an accumulation
of earnings, the Courts of the United States have invented the so-called "Immediacy
Test" which construed the words "reasonable needs of the business" to mean the
immediate needs of the business, and it was generally held that if the corporation did not
prove an immediate need for the accumulation of the earnings and profits, the
accumulation was not for the reasonable needs of the business, and the penalty tax
would apply. (Mertens. Law of Federal Income Taxation, Vol. 7, Chapter 39, p, 103).30
In the present case, the Tax Court opted to determine the working capital sufficiency by using
the ratio between current assets to current liabilities. The working capital needs of a business
depend upon nature of the business, its credit policies, the amount of inventories, the rate of the
turnover, the amount of accounts receivable, the collection rate, the availability of credit to the
business, and similar factors. Petitioner, by adhering to the "Bardahl" formula, failed to impress
the tax court with the required definiteness envisioned by the statute. We agree with the tax
court that the burden of proof to establish that the profits accumulated were not beyond the
reasonable needs of the company, remained on the taxpayer. This Court will not set aside lightly
the conclusion reached by the Court of Tax Appeals which, by the very nature of its function, is
dedicated exclusively to the consideration of tax problems and has necessarily developed an
expertise on the subject, unless there has been an abuse or improvident exercise of
authority.31 Unless rebutted, all presumptions generally are indulged in favor of the correctness
of the CIR's assessment against the taxpayer. With petitioner's failure to prove the CIR
incorrect, clearly and conclusively, this Court is constrained to uphold the correctness of tax
court's ruling as affirmed by the Court of Appeals.
WHEREFORE, the instant petition is DENIED, and the decision of the Court of Appeals,
sustaining that of the Court of Tax Appeals, is hereby AFFIRMED. Costs against petitioner.
SO ORDERED.

Republic of the Philippines


SUPREME COURT
Manila
EN BANC
G.R. No. L-12954

February 28, 1961


20

COLLECTOR OF INTERNAL REVENUE, petitioner,


vs.
ARTHUR HENDERSON, respondent.
x---------------------------------------------------------x
G.R. No. L-13049

February 28, 1961

ARTHUR HENDERSON, petitioner,


vs.
COLLECTOR OF INTERNAL REVENUE, respondent.
Office of the Solicitor General for petitioner.
Formilleza & Latorre for respondent.
PADILLA, J.:
These are petitioner filed by the Collector of Internal Revenue (G.R. No. L-12954) and by Arthur
Henderson (G.R. No. L-13049) under the provisions of section 18, Republic Act No. 1125, for
review of a judgment dated 26 June 1957 and a resolution dated 28 September 1957 rendered
and adopted by the Court of Tax Appeals in Case No. 237.
The spouses Arthur Henderson and Marie B. Henderson (later referred to as the taxpayers) filed
with the Bureau of Internal Revenue returns of annual net income for the years 1948 to 1952,
inclusive, where the following net incomes, personal exemptions and amounts subject to tax
appear:

1948:

Net
P29,573.79
Income ..............................................
.........

Less:Personal
Exemption ..............................

Amount subject to
tax .......................................

2,500.00

P27,073.79

1949:

Net
P31,817.66
Income ..............................................
21

.........

Less:Personal
Exemption ..............................

Amount subject to
tax .......................................

2,500.00

P29,317.66

1950:

Net
P34,815.74
Income ..............................................
.........

Less:Personal
Exemption ..............................

Amount subject to
tax .......................................

3,000.00

P31,815.74

1951:

Net
P32,605.83
Income ..............................................
..........

Less:Personal
Exemption ..............................

Amount subject to
tax .......................................

3,000.00

P29,605.83

1952:

22

Net
P36,780.11
Income ..............................................
.........

Less:Personal
Exemption ..............................

Amount subject to
tax .......................................

3,000.00

P33,780.11

(Exhibits 1, 3, 5, 7, 9, A, F, J, N, R). In due time the taxpayers received from the Bureau of
Internal Revenue assessment notices Nos. 15804-48, 25450-49, 15255-50, 25705-51 and
22527-52 and paid the amounts assessed as follows:

1948:

14 May 1949, O.R. No. 52991, Exhibit B ....


..

P2,068.12

12 September 1950, O.R. No. 160473, Exhibit


B-1 .

2,068.11

Total Paid .........................................................

P4,136.23

13 May 1950, O.R. No. 232366, Exhibit


G ...........

P2,314.95

1949:

15 September 1950, O.R. No. 247918, Exhibit


G-1 .

2,314.94

Total Paid .........................................................

P4,629.89

23

1950:

27 April 1951, O.R. No. 323173, Exhibit K ...


.

P7,273.00

Amount withheld from salary and paid by


employer .

P5,780.40

1951:

15 May 1952, O.R. No. 33250, Exhibit


O .................

360.50

15 August 1952, O.R. No. 383318, Exhibit O1 ....

361.20

Total Paid .........................................................

P6,502.10

Amount withheld from salary and paid by


employer .

P5,660.40

1952:

18 May 1953, O.R. No. 438026, Exhibit T ..

1,160.30

13 August 1953, O.R. No. 443483, Exhibit T1 .....

1,160.30

Total Paid .........................................................

P7,981.00

On 28 November 1953, after investigation and verification, the Bureau of Internal Revenue
reassessed the taxpayers' income for the years 1948 to 1952, inclusive, as follows:
24

1948:

Net income per return ..................................

P29,573.79

Add:

Rent expense .........................................................


..

Additional bonus for 1947 received May 13, 1948 .

7,200.00

6,500.00

Other income:

Manager's residential expense (2/29/48


a/c/#4.51)

1,400.00

Manager's residential expense (refer to 1948 P


& L) ..

1,849.32

Entrance fee Marikina Gun & Country Club ..


..

Net income per investigation .........................................


...

200.00

P46,723.11

Less: Personal exemption ...............................................


.

2,500.00

Net taxable income ..........................................................

P44,223.11

25

Tax due thereon ...............................................................

Less: Amount of tax already paid per OR #52991 &


160473 ..

Deficiency tax still due & assessable ............................

P8,562.47

4,136.23

P4,426.24

1949:

Net income per return ..................................

P31,817.66

Add: disallowances

Capital loss (no capital


gain) ...................

P3,248.84

Undeclared bonus ...................


..

3,857.75

Rental allowance from


A.I.U. ...................

1,800.00

Subsistence allowance from A.I.U. .


..

6,051.30

Net income per investigation .........................................


...

Less: Personal exemption ...............................................


..

14,958.09

P46,775.75

2,500.00

26

Amount of income subject to tax ..................................


.

43,275.75

Tax due thereon ...............................................................


.

P8,292.21

Less: tax already assessed & paid per OR Nos. 232366


& 247918

4,629.89

Deficiency tax due ............................................................


.

P3,662.23

(Should be) ......................................................................

3,662.32

1950:

Net income per return ..................................

P34,815.74

Add:

Rent, electricity, water allowances .......................


..

Net income per investigation .........................................


...

8,373.73

P43,189.47

Less: Personal exemption ...............................................


..

3,000.00

Net taxable income ..........................................................


..

P40,189.47

27

Tax due thereon ...............................................................


.

P10,296.00

Less: tax already paid per OR No. #323173

7,273.00

Deficiency tax due & assessable .................


..

P3,023.00

1951:

Net income per return ..................................

Add: house rental allowance from AIU

Net income per investigation .........................................


...

Less: Personal exemption ...............................................


..

P32,605.83

5,782.91

P83,388.74

3,000.00

Amount of income subject to tax ..................................


..

P35,388.74

Tax due thereon ...............................................................


.

P 8,560.00

Less: tax already assessed and paid per O.R. Nos.


A33250
& 383318 .......................

6,502.00

28

Deficiency tax due .................


.

P2,058.00

1952:

Net income per return ..................................

P36,780.11

Add:

Withholding tax paid by


company .....................................

600.00

Travelling
allowances .......................................................

3,247.40

Allowances for rent, telephone, water, electricity,


etc. .....

7,044.67

Net income per investigation .........................................


...

Less: Personal exemption ...............................................


..

P47,672.18

3,000.00

Net taxable income ..................................

P44,672.18

Tax due thereon ...............................................................


.

P12,089.00

Less: Tax already withheld

P5,660.40

29

Tax already paid per O.R. Nos. #438026,


443484

2,320.60

Deficiency tax still due & collectible ...............................

7,981.00

P4,108.00

(Exhibits 2, 4, 6, 8, 10) and demanded payment of the deficiency taxes on or before 28


February 1954 with respect to those due for the years 1948, 1949, 1950 and 1952and on or
before 15 February 1954 with respect to that due for the year 1951 (Exhibits B-2, H, L, P, S).
In the foregoing assessments, the Bureau of Internal Revenue considered as part of their
taxable income the taxpayer-husband's allowances for rental, residential expenses,
subsistence, water, electricity and telephone; bonus paid to him; withholding tax and entrance
fee to the Marikinagun and Country Bluc paid by his employer for his account; and travelling
allowance of his wife. On 26 and27 January 1954 the taxpayers asked for reconsideration of the
foregoing assessment (pp. 29, 31, BIR rec.) and on 11 February 1954 and 28 February 1955
stated the grounds and reasons in support of their request for reconsideration (pp. 36-38, 62-66,
BIR rec.). The claim that as regards the husband-taxpayer's allowances for rental and utilities
such as water, electricity and telephone, he did not receive the money for said allowances, but
that they lived in the apartment furnished and paid for by his employer for its convenience; that
they had no choice but live in the said apartment furnished by his employer, otherwise they
would have lived in a less expensive one; that as regards his allowances for rental of P7,200
and residential expenses of P1,400 and P1,849.32 in 1948, rental of P1,800 and subsistence of
P6,051.50 (the latter merely consisting of allowances for rent and utilities such as light, water,
telephone, etc.) in 1949 rental, electricity and water of P8,373.73 in 1950, rental of P5,782.91 in
1951 and rental, telephone, water, electricity, etc. of P7,044.67 in 1952, only the amount of
P3,900 for each year, which is the amount they would have spent for rental of an apartment
including utilities, should be taxed; that as regards the amount ofP200 representing entrance fee
to the Marikina Gun and Country Club paid for him by his employer in 1948, the same should
not be considered as part of their income for it was an expense of his employer and his
membership therein was merely incidental to his duties of increasing and sustaining the
business of his employer; and that as regards the wife-taxpayer's travelling allowance of
P3,247.40 in 1952, it should not be considered as part of their income because she merely
accompanied him in his business trip to New York as his secretary and, at the behest of her
husband's employer, to study and look into the details of the plans and decorations of the
building intended to be constructed by his employer in its property at Dewey Boulevard. On 15
and 27 February 1954, the taxpayers paid the deficiency taxes assessed under Official Receipts
Nos. 451841, 451842, 451843, 451748 and 451844 (Exhibits, I, M, Q, and Y). After hearing
conducted by the Conference Staff of the Bureau of Internal Revenue on5 October 1954 (pp.
74-85, BIR rec.), on 27 May 1955the Staff recommended to the Collector of Internal Revenue
that the assessments made on 28 November 1953 (Exhibits2, 4, 6, 8, 10) be sustained except
that the amount of P200 as entrance fee to the Marikina Gun and Country Club paid for the
husband-taxpayer's account by his employer in 1948 should not be considered as part of the
taxpayers' taxable income for that year (pp. 95-107, BIR rec.). On 14 July 1955, in line with the
recommendation of the Conference Staff, the Collector of Internal Revenue denied the
taxpayers' request for reconsideration, except as regards the assessment of their income tax
due for the year 1948, which was modified as follows:

30

Net income per return

Add: Rent expense

P29,573.79

7,200.00

Additional bonus for 1947


received on May 13, 1948

6,500.00

Manager's residential expense


(2/29/48 a/c #4.41)

1,400.00

Manager's residential expense


(1948 profit and loss)

1,849.32

Net income per investigation

Less: Personal exemption

P46,523.11

2,500.00

Net taxable income

P44,023.11

Tax due thereon

P 8,506.47

Less; Amount already paid

Deficiency tax still due

4,136.23

P 4,370.24

and demanded payment of the deficiency taxes of P4,370.24 for 1948, P3,662.23 for 1949,
P3,023 for 1950, P2,058 for1951 and P4,108 for 1952, 5% surcharge and 1% monthly interest
thereon from 1 March 1954 to the date of payment and P80 as administrative penalty for late
payment, to the City Treasurer of Manila not later than 31 July1955 (Exhibit 14). On 30 January
1956 the taxpayers again sought a reconsideration of the denial of their request for
reconsideration and offered to settle the case on a more equitable basis by increasing the
amount of the taxable portion of the husband-taxpayer's allowances for rental, etc. from P3,000
yearly to P4,800 yearly, which "is the value to the employee of the benefits he derived therefrom
measured by what he had saved on account thereof in the ordinary course of his life ... for
which he would have spent in any case'". The taxpayers also reiterated their previous stand
31

regarding the transportation allowance of the wife-taxpayer of P3,247.40 in 1952 and requested
the refund of the amounts of P3,477.18, P569.33,P1,294, P354 and P2,164, or a total of
P7,858.51, (Exhibit Z). On 10 February 1956 the taxpayers again requested the Collector of
Internal Revenue to refund to them the amounts allegedly paid in excess as income taxes for
the years 1948 to 1952, inclusive (Exhibit Z-1). The Collector of Internal Revenue did not take
any action on the taxpayers request for refund.
On 15 February 1956 the taxpayers filed in the Court of Tax Appeals a petition to review the
decision of the Collector of Internal Revenue (C.T.A. Case No. 237). After hearing, on 26 June
1957 the Court rendered judgment holding "that the inherent nature of petitioner's(the husbandtaxpayer) employment as president of the American International Underwriters as president of
the American International Underwriters of the Philippines, Inc. does not require him to occupy
the apartments supplied by his employer-corporation;" that, however, only the amount of P4,800
annually, the ratable value to him of the quarters furnished constitutes a part of taxable income;
that since the taxpayers did not receive any benefit out of the P3,247.40 traveling expense
allowance granted in 1952 to the wife-taxpayer and that she merely undertook the trip abroad at
the behest of her husband's employer, the same could not be considered as income; and that
even if it were considered as such, still it could not be subject to tax because it was deductible
as travel expense; and ordering the Collector of Internal Revenue to refund to the taxpayers the
amount of P5,109.33 with interest from 27 February 1954, without pronouncement as to costs.
The taxpayers filed a motion for reconsideration claiming that the amount of P5,986.61 is the
amount refundable to them because the amounts of P1,400 and P1,849.32 as manager's
residential expenses in 1948 should not be included in their taxable net income for the reason
that they are of the same nature as the rentals for the apartment, they being mainly expenses
for utilities as light, water and telephone in the apartment furnished by the husband-taxpayer's
employer. The Collector of Internal Revenue filed an opposition to their motion for
reconsideration. He also filed a separate motion for reconsideration of the decision claiming that
his assessment under review was correct and should have been affirmed. The taxpayers filed
an opposition to this motion for reconsideration of the Collector of Internal Revenue; the latter, a
reply thereto. On 28 September 1957 the Court denied both motions for reconsideration. On 7
October1957 the Collector of Internal Revenue filed a notice of appeal in the Court of Tax
Appeals and on 21 October1957, within the extension of time previously granted by this Court, a
petition for review (G.R. No. L-12954). On29 October 1957 the taxpayers filed a notice of appeal
in the Court of Tax Appeals and a petition for review in this Court (G.R. No. L-13049).
The Collector of Internal Revenue had assigned the following errors allegedly committed by the
Court of Tax Appeals:
I. The Court of Tax Appeals erred in finding that the herein respondent did not have any
choice in the selection of the living quarters occupied by him.
II. The Court of Tax Appeals erred in not considering the fact that respondent is not a
minor company official but the President of his employer-corporation, in the appreciation
of respondent's alleged lack of choice in the matter of the selection of the quarters
occupied by him.
III. The Court of Tax Appeals erred in giving full weight and credence to respondent's
allegation, a self-serving and unsupported declaration that the ratable value to him of the
living quarters and subsistence allowance was only P400.00 a month.
IV. The Court of Tax Appeals erred in holding that only the ratable value of P4,800.00 per
annum, or P400.00 a month constitutes income to respondent.
32

V. The Court of Tax Appeals erred in arbitrarily fixing the amount of P4,800.00 per
annum, or P400.00 a month as the only amount taxable against respondent during the
five tax years in question.
VI. The Court of Tax Appeals erred in not finding that travelling allowance in the amount
of P3,247.40 constituted income to respondent and, therefore, subject to the income tax.
VII. The Court of Tax Appeals erred in ordering the refund of the sum of P5,109.33 with
interest from February 17, 1954. (G.R. No. L-12954.)
The taxpayers have assigned the following errors allegedly committed by the Court of Tax
Appeals:
I. The Court of Tax Appeals erred in its computation of the 1948 income tax and
consequently in the amount that should be refunded for that year.
II. The Court of Tax Appeals erred in denying our motion for reconsideration as contained
in its resolution dated September 28, 1957. (G.R. No. L-13049.)
The Government's appeal:
The Collector of Internal Revenue raises questions of fact. He claims that the evidence is not
sufficient to support the findings and conclusion of the Court of Tax Appeals that the quarters
occupied by the taxpayers were not of their choice but that of the husband-taxpayer's employer;
that it did not take into consideration the fact that the husband-taxpayer is not a mere minor
company official, but the highest executive of his employer-corporation; and that the wifetaxpayer's trip abroad in 1952 was not, as found by the Court, a business but a vacation trip. In
Collector of Internal Revenue vs. Aznar, 56, Off. Gaz. 2386, this Court held that in petitions for
review under section 18, Republic Act No. 1125, it may review the findings of fact of the Court of
Tax Appeals.
The determination of the main issue in the case requires a review of the evidence. Are the
allowances for rental of the apartment furnished by the husband-taxpayer's employercorporation, including utilities such as light, water, telephone, etc. and the allowance for travel
expenses given by his employer-corporation to his wife in 1952 part of taxable income? Section
29, Commonwealth Act No. 466, National Internal Revenue Code, provides:
"Gross income" includes gains, profits, and income derived from salaries, wages, or
compensation for personal service of whatever kind and in whatever form paid, or from
professions, vocations, trades, businesses, commerce, sales, or dealings in property,
whether real or personal, growing out of the ownership or use of or interest in such
property; also from interest, rents dividend, securities, or the transaction of any business
carried on for gain or profit, or gains, profits, and income derived from any source
whatever. (Emphasis ours.)
The Court of Tax Appeals found that the husband-taxpayer "is the president of the American
International Underwriters for the Philippines, Inc., a domestic corporation engaged in insurance
business;" that the taxpayers "entertained officials, guests and customers of his employercorporation, in apartments furnished by the latter and successively occupied by him as
president thereof; that "In 1952, petitioner's wife, Mrs. Marie Henderson, upon request o Mr. C.
V. Starr, chairman of the parent corporation of the American International Underwriters for the
Philippines, Inc., undertook a trip to New York in connection with the purchase of a lot in Dewey
33

Boulevard by petitioner's employer-corporation, the construction of a building thereon, the


drawing of prospectus and plans for said building, and other related matters."
Arthur H. Henderson testified that he is the President of American International Underwriters for
the Philippines, Inc., which represent a group of American insurance companies engaged in the
business of general insurance except life insurance; that he receives a basic annual salary of
P30,000 and allowance for house rental and utilities like light, water, telephone, etc.; that he and
his wife are childless and are the only two in the family; that during the years 1948 to 1952, they
lived in apartments chosen by his employer; that from 1948 to the early part of 1950, they lived
at the Embassy Apartments on Dakota Street, Manila, where they had a large sala, three
bedrooms, dining room, two bathrooms, kitchen and a large porch, and from the early part of
1950 to 1952, they lived at the Rosaria Apartments on the same street where they had a
kitchen, sala, dining room two bedrooms and bathroom; that despite the fact that they were the
only two in the family, they had to live in apartments of the size beyond their personal needs
because as president of the corporation, he and his wife had to entertain and put up
houseguests; that during all those years of 1948 to 1952, inclusive, they entertained and put up
houseguests of his company's officials, guests and customers such as the president of C, V.
Starr & Company, Inc., who spent four weeks in his apartment, Thomas Cocklin, a lawyer from
Washington, D.C., and Manuel Elizalde, a stockholder of AIUPI; that were he not required by his
employer to live in those apartments furnished to him, he and his wife would have chosen an
apartment only large enough for them and spend from P300 to P400 monthly for rental; that of
the allowances granted to him, only the amount of P4,800 annually, the maximum they would
have spent for rental, should be considered as taxable income and the excess treated as
expense of the company; and that the trip to New York undertaken by his wife in 1952, for which
she was granted by his employer-corporation travelling expense allowance of P3,247.40, was
made at the behest of his employer to assist its architect in the preparation of the plans for a
proposed building in Manila and procurement of supplies and materials for its use, hence the
said amount should not be considered as part of taxable income. In support of his claim, letters
written by his wife while in New York concerning the proposed building, inquiring about the
progress made in the acquisition of the lot, and informing him of the wishes of Mr. C. V. Starr,
chairman of the board of directors of the parent-corporation (Exhibits U-1, U-1-A, V, V-1 and W)
and a letter written by the witness to Mr. C. V. Starr concerning the proposed building (Exhibits
X, X-1) were presented in evidence.
Mrs. Marie Henderson testified that for almost three years, she and her husband gave parties
every Friday night at their apartment for about 18 to 20 people; that their guests were officials of
her husband's employer-corporation and other corporations; that during those parties movies for
the entertainment of the guests were shown after dinner; that they also entertained during
luncheons and breakfasts; that these involved and necessitated the services of additional
servants; and that in 1952 she was asked by Mr. C. V. Starr to come to New York to take up
problems concerning the proposed building and entertainment because her husband could not
make the trip himself, and because "the woman of the family is closer to those problems."
The evidence presented at the hearing of the case substantially supports the findings of the
Court of Tax Appeals. The taxpayers are childless and are the only two in the family. The
quarters, therefore, that they occupied at the Embassy Apartments consisting of a large sala,
three bedrooms, dining room, two bathrooms, kitchen and a large porch, and at the Rosaria
Apartments consisting of a kitchen, sala dining room, two bedrooms and a bathroom, exceeded
their personal needs. But the exigencies of the husband-taxpayer's high executive position, not
to mention social standing, demanded and compelled them to live in amore spacious and
pretentious quarters like the ones they had occupied. Although entertaining and putting up
houseguests and guests of the husband-taxpayer's employer-corporation were not his
predominant occupation as president, yet he and his wife had to entertain and put up
34

houseguests in their apartments. That is why his employer-corporation had to grant him
allowances for rental and utilities in addition to his annual basic salary to take care of those
extra expenses for rental and utilities in excess of their personal needs. Hence, the fact that the
taxpayers had to live or did not have to live in the apartments chosen by the husband-taxpayer's
employer-corporation is of no moment, for no part of the allowances in question redounded to
their personal benefit or was retained by them. Their bills for rental and utilities were paid
directly by the employer-corporation to the creditors (Exhibit AA to DDD, inclusive; pp. 104, 170193, t.s.n.). Nevertheless, as correctly held by the Court of Tax Appeals, the taxpayers are
entitled only to a ratable value of the allowances in question, and only the amount of P4,800
annually, the reasonable amount they would have spent for house rental and utilities such as
light, water, telephone, etc., should be the amount subject to tax, and the excess considered as
expenses of the corporation.
Likewise, the findings of the Court of Tax Appeals that the wife-taxpayer had to make the trip to
New York at the behest of her husband's employer-corporation to help in drawing up the plans
and specifications of a proposed building, is also supported by the evidence. The parts of the
letters written by the wife-taxpayer to her husband while in New York and the letter written by
the husband-taxpayer to Mr. C. V. Starr support the said findings (Exhibits U-2, V-1, W-1, X). No
part of the allowance for travelling expenses redounded to the benefit of the taxpayers. Neither
was a part thereof retained by them. The fact that she had herself operated on for tumors while
in New York was but incidental to her stay there and she must have merely taken advantage of
her presence in that city to undergo the operation.
The taxpayers' appeal:
The taxpayers claim that the Court of Tax Appeals erred in considering the amounts of P1,400
and P1,849.32, or a total of P3,249.32, for "manager's residential expense" in 1948 as taxable
income despite the fact "that they were of the same nature as the rentals for the apartment, they
being expenses for utilities, such as light, water and telephone necessarily incidental to the
apartment furnished to him by his employer."
Mrs. Crescencia Perez Ramos, an examiner of the Bureau of Internal Revenue who examined
the books of account of the American International Underwriters for the Philippines, Inc.,
testified that he total amount of P3,249.32 was reflected in its books as "living expenses of Mr.
and Mrs. Arthur Henderson in the quarters they occupied in 1948;" and that "the amount of
P1,400 was included as manager's residential expense while the amount of P1,849.32 was
entered as profit and loss account."
Buenaventura Loberiza, acting head of the accouting department of the American International
Underwriters for the Philippines, Inc., testified that rentals, utilities, water, telephone and electric
bills of executives of the corporation were entered in the books of account as "subsistence
allowances and expenses;" that there was a separate account for salaries and wages of
employees and officers; and that expenses for rentals and other utilities were not charged to
salary accounts.
The taxpayers' claim is supported by the evidence. The total amount of P3,249.32 "for
manager's residential expense" in 1948 should be treated as rentals for apartments and utilities
and should not form part of the ratable value subject to tax.
The computation made by the taxpayers is correct. Adding to the amount of P29,573.79, their
net income per return, the amount of P6,500, the bonus received in 1948, and P4,800, the
taxable ratable value of the allowances, brings up their gross income to P40,873.79. Deducting
therefrom the amount of P2,500 for personal exemption, the amount of P38,373.79 is the
35

amount subject to income tax. The income tax due on this amount is P6,957.19 only. Deducting
the amount of income tax due, P6,957.19, from the amount already paid, P8,562.47 (Exhibits B,
B-1, C), the amount of P1,605.28 is the amount refundable to the taxpayers. Add this amount to
P563.33, P1,294.00, P354.00 and P2,154.00, refundable to the taxpayers for 1949, 1950, 1951
and 1952 and the total is P5,986.61.
The judgment under review is modified as above indicated. The Collector of Internal Revenue is
ordered to refund to the taxpayers the sum of P5,986.61, without pronouncement as to costs.
Bengzon, Actg. C.J., Bautista Angelo, Labrador, Concepcion, Reyes, J.B.L., Barrera, Paredes
and Dizon, JJ.,concur.

Republic of the Philippines


SUPREME COURT
Manila
SECOND DIVISION
G.R. No. 96016 October 17, 1991
COMMISSIONER OF INTERNAL REVENUE, petitioner,
vs.
THE COURT OF APPEALS and EFREN P. CASTANEDA, respondents.
Leovigildo Monasterial for private respondent.
RESOLUTION
36

PADILLA, J.:p
The issue to be resolved in this petition for review on certiorari is whether or not terminal leave
pay received by a government official or employee on the occasion of his compulsory retirement
from the government service is subject to withholding (income) tax.
We resolve the issue in the negative.
Private respondent Efren P. Castaneda retired from the government service as Revenue
Attache in the Philippine Embassy in London, England, on 10 December 1982 under the
provisions of Section 12 (c) of Commonwealth Act 186, as amended. Upon retirement, he
received, among other benefits, terminal leave pay from which petitioner Commissioner of
Internal Revenue withheld P12,557.13 allegedly representing income tax thereon.
Castaneda filed a formal written claim with petitioner for a refund of the P12,557.13, contending
that the cash equivalent of his terminal leave is exempt from income tax. To comply with the
two-year prescriptive period within which claims for refund may be filed, Castaneda filed on 16
July 1984 with the Court of Tax Appeals a Petition for Review, seeking the refund of income tax
withheld from his terminal leave pay.
The Court of Tax Appeals found for private respondent Castaneda and ordered the
Commissioner of Internal Revenue to refund Castaneda the sum of P12,557.13 withheld as
income tax. (,Annex "C", petition).
Petitioner appealed the above-mentioned Court of Tax Appeals decision to this Court, which
was docketed as G.R. No. 80320. In turn, we referred the case to the Court of Appeals for
resolution. The case was docketed in the Court of Appeals as CA-G.R. SP No. 20482.
On 26 September 1990, the Court of Appeals dismissed the petition for review and affirmed the
decision of the Court of Tax Appeals. Hence, the present recourse by the Commissioner of
Internal Revenue.
The Solicitor General, acting on behalf of the Commissioner of Internal Revenue, contends that
the terminal leave pay is income derived from employer-employee relationship, citing in support
of his stand Section 28 of the National Internal Revenue Code; that as part of the compensation
for services rendered, terminal leave pay is actually part of gross income of the recipient. Thus

. . . It (terminal leave pay) cannot be viewed as salary for purposes which would
reduce it. . . . there can thus be no "commutation of salary" when a government
retiree applies for terminal leave because he is not receiving it as salary. What he
applies for is a "commutation of leave credits." It is an accumulation of credits
intended for old age or separation from service. . . .
The Court has already ruled that the terminal leave pay received by a government official or
employee is not subject to withholding (income) tax. In the recent case of Jesus N. Borromeo
vs. The Hon. Civil Service Commission, et al., G.R. No. 96032, 31 July 1991, the Court
explained the rationale behind the employee's entitlement to an exemption from withholding
(income) tax on his terminal leave pay as follows:
. . . commutation of leave credits, more commonly known as terminal leave, is
applied for by an officer or employee who retires, resigns or is separated from the
37

service through no fault of his own. (Manual on Leave Administration Course for
Effectiveness published by the Civil Service Commission, pages 16-17). In the
exercise of sound personnel policy, the Government encourages unused leaves
to be accumulated. The Government recognizes that for most public servants,
retirement pay is always less than generous if not meager and scrimpy. A modest
nest egg which the senior citizen may look forward to is thus avoided. Terminal
leave payments are given not only at the same time but also for the same policy
considerations governing retirement benefits.
In fine, not being part of the gross salary or income of a government official or employee but a
retirement benefit, terminal leave pay is not subject to income tax.
ACCORDINGLY, the petition for review is hereby DENIED.
SO ORDERED.
Paras and Regalado, JJ., concur.
Melencio-Herrera (Chairman), J., is on leave.

Republic of the Philippines


SUPREME COURT
Manila
FIRST DIVISION
G.R. No. 108576 January 20, 1999
COMMISSIONER OF INTERNAL REVENUE, petitioner,
vs.
THE COURT OF APPEALS, COURT OF TAX APPEALS and A. SORIANO
CORP., respondents.
MARTINEZ, J.:
Petitioner Commissioner of Internal Revenue (CIR) seeks the reversal of the decision of the
Court of Appeals (CA) 1 which affirmed the ruling of the Court of Tax Appeals (CTA) 2 that private
respondent A. Soriano Corporation's (hereinafter ANSCOR) redemption and exchange of the
38

stocks of its foreign stockholders cannot be considered as "essentially equivalent to a


distribution of taxable dividends" under, Section 83(b) of the 1939 Internal Revenue Act. 3
The undisputed facts are as follows:
Sometime in the 1930s, Don Andres Soriano, a citizen and resident of the United States, formed
the corporation "A. Soriano Y Cia", predecessor of ANSCOR, with a P1,000,000.00
capitalization divided into 10,000 common shares at a par value of P100/share. ANSCOR is
wholly owned and controlled by the family of Don Andres, who are all non-resident aliens. 4 In
1937, Don Andres subscribed to 4,963 shares of the 5,000 shares originally issued. 5
On September 12, 1945, ANSCOR's authorized capital stock was increased to P2,500,000.00
divided into 25,000 common shares with the same par value of the additional 15,000 shares,
only 10,000 was issued which were all subscribed by Don Andres, after the other stockholders
waived in favor of the former their pre-emptive rights to subscribe to the new issues. 6 This
increased his subscription to 14,963 common shares. 7 A month later, 8 Don Andres transferred
1,250 shares each to his two sons, Jose and Andres, Jr., as their initial investments in
ANSCOR. 9 Both sons are foreigners. 10
By 1947, ANSCOR declared stock dividends. Other stock dividend declarations were made
between 1949 and December 20, 1963. 11 On December 30, 1964 Don Andres died. As of that
date, the records revealed that he has a total shareholdings of 185,154 shares 12 50,495 of
which are original issues and the balance of 134.659 shares as stock dividend
declarations. 13Correspondingly, one-half of that shareholdings or 92,577 14 shares were
transferred to his wife, Doa Carmen Soriano, as her conjugal share. The other half formed part
of his estate. 15
A day after Don Andres died, ANSCOR increased its capital stock to P20M 16 and in 1966 further
increased it to P30M. 17 In the same year (December 1966), stock dividends worth 46,290 and
46,287 shares were respectively received by the Don Andres estate 18 and Doa Carmen from
ANSCOR. Hence, increasing their accumulated shareholdings to 138,867 and
138,864 19 common shares each. 20
On December 28, 1967, Doa Carmen requested a ruling from the United States Internal
Revenue Service (IRS), inquiring if an exchange of common with preferred shares may be
considered as a tax avoidance scheme 21under Section 367 of the 1954 U.S. Revenue Act. 22 By
January 2, 1968, ANSCOR reclassified its existing 300,000 common shares into 150,000
common and 150,000 preferred shares. 23
In a letter-reply dated February 1968, the IRS opined that the exchange is only a recapitalization
scheme and not tax avoidance. 24 Consequently, 25 on March 31, 1968 Doa Carmen
exchanged her whole 138,864 common shares for 138,860 of the newly reclassified preferred
shares. The estate of Don Andres in turn, exchanged 11,140 of its common shares, for the
remaining 11,140 preferred shares, thus reducing its (the estate) common shares to 127,727. 26
On June 30, 1968, pursuant to a Board Resolution, ANSCOR redeemed 28,000 common
shares from the Don Andres' estate. By November 1968, the Board further increased
ANSCOR's capital stock to P75M divided into 150,000 preferred shares and 600,000 common
shares. 27 About a year later, ANSCOR again redeemed 80,000 common shares from the Don
Andres' estate, 28 further reducing the latter's common shareholdings to 19,727. As stated in the
Board Resolutions, ANSCOR's business purpose for both redemptions of stocks is to partially
retire said stocks as treasury shares in order to reduce the company's foreign exchange
remittances in case cash dividends are declared. 29
39

In 1973, after examining ANSCOR's books of account and records, Revenue examiners issued
a report proposing that ANSCOR be assessed for deficiency withholding tax-at-source, pursuant
to Sections 53 and 54 of the 1939 Revenue Code, 30 for the year 1968 and the second quarter
of 1969 based on the transactions of exchange 31 and redemption of stocks. 31The Bureau of
Internal Revenue (BIR) made the corresponding assessments despite the claim of ANSCOR
that it availed of the tax amnesty under Presidential Decree (P.D.) 23 32 which were amended by
P.D.'s 67 and 157. 33 However, petitioner ruled that the invoked decrees do not cover Sections
53 and 54 in relation to Article 83(b) of the 1939 Revenue Act under which ANSCOR was
assessed. 34ANSCOR's subsequent protest on the assessments was denied in 1983 by
petitioner. 35
Subsequently, ANSCOR filed a petition for review with the CTA assailing the tax assessments
on the redemptions and exchange of stocks. In its decision, the Tax Court reversed petitioner's
ruling, after finding sufficient evidence to overcome the prima facie correctness of the
questioned assessments. 36 In a petition for review the CA as mentioned, affirmed the ruling of
the CTA. 37 Hence, this petition.
The bone of contention is the interpretation and application of Section 83(b) of the 1939
Revenue Act 38 which provides:
Sec. 83. Distribution of dividends or assets by corporations.
(b) Stock dividends A stock dividend representing the transfer of surplus to
capital account shall not be subject to tax. However, if a corporation cancels or
redeems stock issued as a dividend at such time and in such manner as to make
the distribution and cancellation or redemption, in whole or in part, essentially
equivalent to the distribution of a taxable dividend, the amount so distributed in
redemption or cancellation of the stock shall be considered as taxable income to
the extent it represents a distribution of earnings or profits accumulated after
March first, nineteen hundred and thirteen. (Emphasis supplied)
Specifically, the issue is whether ANSCOR's redemption of stocks from its stockholder
as well as the exchange of common with preferred shares can be considered as
"essentially equivalent to the distribution of taxable dividend" making the proceeds
thereof taxable under the provisions of the above-quoted law.
Petitioner contends that the exchange transaction a tantamount to "cancellation" under Section
83(b) making the proceeds thereof taxable. It also argues that the Section applies to stock
dividends which is the bulk of stocks that ANSCOR redeemed. Further, petitioner claims that
under the "net effect test," the estate of Don Andres gained from the redemption. Accordingly, it
was the duty of ANSCOR to withhold the tax-at-source arising from the two transactions,
pursuant to Section 53 and 54 of the 1939 Revenue Act. 39
ANSCOR, however, avers that it has no duty to withhold any tax either from the Don Andres
estate or from Doa Carmen based on the two transactions, because the same were done for
legitimate business purposes which are (a) to reduce its foreign exchange remittances in the
event the company would declare cash dividends, 40 and to (b) subsequently "filipinized"
ownership of ANSCOR, as allegedly, envisioned by Don Andres. 41 It likewise invoked the
amnesty provisions of P.D. 67.
We must emphasize that the application of Sec. 83(b) depends on the special factual
circumstances of each case.42 The findings of facts of a special court (CTA) exercising particular
expertise on the subject of tax, generally binds this Court, 43 considering that it is substantially
40

similar to the findings of the CA which is the final arbiter of questions of facts. 44 The issue in this
case does not only deal with facts but whether the law applies to a particular set of facts.
Moreover, this Court is not necessarily bound by the lower courts' conclusions of law drawn from
such facts. 45
AMNESTY:
We will deal first with the issue of tax amnesty. Section 1 of P.D. 67 46 provides:
1. In all cases of voluntary disclosures of previously untaxed income and/or
wealth such as earnings, receipts, gifts, bequests or any other acquisitions from
any source whatsoever which are taxable under the National Internal Revenue
Code, as amended, realized here or abroad by any taxpayer, natural or judicial;
the collection of all internal revenue taxes including the increments or penalties
or account of non-payment as well as all civil, criminal or administrative liabilities
arising from or incident to such disclosures under the National Internal Revenue
Code, the Revised Penal Code, the Anti-Graft and Corrupt Practices Act, the
Revised Administrative Code, the Civil Service laws and regulations, laws and
regulations on Immigration and Deportation, or any other applicable law or
proclamation, are hereby condoned and, in lieu thereof, a tax of ten (10%) per
centum on such previously untaxed income or wealth, is hereby imposed, subject
to the following conditions: (conditions omitted) [Emphasis supplied].
The decree condones "the collection of all internal revenue taxes including the
increments or penalties or account of non-payment as well as all civil, criminal or
administrative liable arising from or incident to" (voluntary) disclosures under the NIRC of
previously untaxed income and/or wealth "realized here or abroad by any taxpayer,
natural or juridical."
May the withholding agent, in such capacity, be deemed a taxpayer for it to avail of the
amnesty? An income taxpayer covers all persons who derive taxable income. 47 ANSCOR was
assessed by petitioner for deficiency withholding tax under Section 53 and 54 of the 1939 Code.
As such, it is being held liable in its capacity as a withholding agent and not its personality as a
taxpayer.
In the operation of the withholding tax system, the withholding agent is the payor, a separate
entity acting no more than an agent of the government for the collection of the tax 48 in order to
ensure its payments;49 the payer is the taxpayer he is the person subject to tax impose by
law; 50 and the payee is the taxing authority. 51 In other words, the withholding agent is merely a
tax collector, not a taxpayer. Under the withholding system, however, the agent-payor becomes
a payee by fiction of law. His (agent) liability is direct and independent from the
taxpayer, 52 because the income tax is still impose on and due from the latter. The agent is not
liable for the tax as no wealth flowed into him he earned no income. The Tax Code only
makes the agent personally liable for the tax 53arising from the breach of its legal duty to
withhold as distinguish from its duty to pay tax since:
the government's cause of action against the withholding is not for the collection
of income tax, but for the enforcement of the withholding provision of Section 53
of the Tax Code, compliance with which is imposed on the withholding agent and
not upon the taxpayer. 54
Not being a taxpayer, a withholding agent, like ANSCOR in this transaction is not
protected by the amnesty under the decree.
41

Codal provisions on withholding tax are mandatory and must be complied with by the
withholding agent. 55 The taxpayer should not answer for the non-performance by the
withholding agent of its legal duty to withhold unless there is collusion or bad faith. The former
could not be deemed to have evaded the tax had the withholding agent performed its duty. This
could be the situation for which the amnesty decree was intended. Thus, to curtail tax evasion
and give tax evaders a chance to reform, 56 it was deemed administratively feasible to grant tax
amnesty in certain instances. In addition, a "tax amnesty, much like a tax exemption, is never
favored nor presumed in law and if granted by a statute, the term of the amnesty like that of a
tax exemption must be construed strictly against the taxpayer and liberally in favor of the taxing
authority. 57 The rule on strictissimi juris equally applies. 58 So that, any doubt in the application
of an amnesty law/decree should be resolved in favor of the taxing authority.
Furthermore, ANSCOR's claim of amnesty cannot prosper. The implementing
rules of P.D. 370 which expanded amnesty on previously untaxed income under
P.D. 23 is very explicit, to wit:
Sec. 4. Cases not covered by amnesty. The following cases are not covered
by the amnesty subject of these regulations:
xxx xxx xxx
(2) Tax liabilities with or without assessments, on withholding tax at source
provided under Section 53 and 54 of the National Internal Revenue Code, as
amended; 59
ANSCOR was assessed under Sections 53 and 54 of the 1939 Tax Code. Thus, by
specific provision of law, it is not covered by the amnesty.

TAX ON STOCK DIVIDENDS


General Rule
Sec. 83(b) of the 1939 NIRC was taken from the Section 115(g)(1) of the U.S. Revenue Code of
1928. 60 It laid down the general rule known as the proportionate test 61 wherein stock dividends
once issued form part of the capital and, thus, subject to income tax. 62 Specifically, the general
rule states that:
A stock dividend representing the transfer of surplus to capital account shall not
be subject to tax.
Having been derived from a foreign law, resort to the jurisprudence of its origin may shed light.
Under the US Revenue Code, this provision originally referred to "stock dividends" only, without
any exception. Stock dividends, strictly speaking, represent capital and do not constitute income
to its recipient. 63 So that the mere issuance thereof is not yet subject to income tax 64 as they
are
nothing
but
enrichment
through
increase
in
value
of
capital
investment." 65 As capital, the stock dividends postpone the realization of profits because the
"fund represented by the new stock has been transferred from surplus to capital and no longer
available for actual distribution." 66 Income in tax law is "an amount of money coming to a person
within a specified time, whether as payment for services, interest, or profit from investment." 67 It
means cash or its equivalent. 68 It is gain derived and severed from capital, 69 from labor or from
42

both combined 70 so that to tax a stock dividend would be to tax a capital increase rather than
the income. 71 In a loose sense, stock dividends issued by the corporation, are considered
unrealized gain, and cannot be subjected to income tax until that gain has been realized. Before
the realization, stock dividends are nothing but a representation of an interest in the corporate
properties. 72 As capital, it is not yet subject to income tax. It should be noted that capital and
income are different. Capital is wealth or fund; whereas income is profit or gain or the flow of
wealth.73 The determining factor for the imposition of income tax is whether any gain or profit
was derived from a transaction. 74
The Exception
However, if a corporation cancels or redeems stock issued as a dividend at such
time and in such manner as to make the distribution and cancellation or
redemption, in whole or in part, essentially equivalent to the distribution of
a taxable dividend, the amount so distributed in redemption or cancellation of the
stock shall be considered as taxable income to the extent it represents a
distribution of earnings or profits accumulated after March first, nineteen hundred
and thirteen. (Emphasis supplied).
In a response to the ruling of the American Supreme Court in the case of Eisner v.
Macomber 75 (that pro rata stock dividends are not taxable income), the exempting clause
above quoted was added because provision corporation found a loophole in the original
provision. They resorted to devious means to circumvent the law and evade the tax. Corporate
earnings would be distributed under the guise of its initial capitalization by declaring the stock
dividends previously issued and later redeem said dividends by paying cash to the stockholder.
This process of issuance-redemption amounts to a distribution of taxable cash dividends which
was lust delayed so as to escape the tax. It becomes a convenient technical strategy to avoid
the effects of taxation.
Thus, to plug the loophole the exempting clause was added. It provides that the redemption
or cancellation of stock dividends, depending on the "time" and "manner" it was made, is
essentially equivalent to a distribution of taxable dividends," making the proceeds thereof
"taxable income" "to the extent it represents profits". The exception was designed to prevent the
issuance and cancellation or redemption of stock dividends, which is fundamentally not taxable,
from being made use of as a device for the actual distribution of cash dividends, which is
taxable. 76 Thus,
the provision had the obvious purpose of preventing a corporation from avoiding
dividend tax treatment by distributing earnings to its shareholders in two
transactions a pro rata stock dividend followed by a pro rata redemption
that would have the same economic consequences as a simple dividend. 77
Although redemption and cancellation are generally considered capital transactions, as
such. they are not subject to tax. However, it does not necessarily mean that a
shareholder may not realize a taxable gain from such transactions. 78 Simply put,
depending on the circumstances, the proceeds of redemption of stock dividends are
essentially distribution of cash dividends, which when paid becomes the absolute
property of the stockholder. Thereafter, the latter becomes the exclusive owner thereof
and can exercise the freedom of choice. 79Having realized gain from that redemption, the
income earner cannot escape income tax. 80
As qualified by the phrase "such time and in such manner," the exception was not intended to
characterize as taxable dividend every distribution of earnings arising from the redemption of
43

stock dividend. 81 So that, whether the amount distributed in the redemption should be treated
as the equivalent of a "taxable dividend" is a question of fact, 82 which is determinable on "the
basis of the particular facts of the transaction in question. 83 No decisive test can be used to
determine the application of the exemption under Section 83(b). The use of the words "such
manner" and "essentially equivalent" negative any idea that a weighted formula can resolve a
crucial issue Should the distribution be treated as taxable dividend. 84 On this aspect,
American courts developed certain recognized criteria, which includes the following: 85
1) the presence or absence of real business purpose,
2) the amount of earnings and profits available for the declaration
of a regular dividends and the corporation's past record with
respect to the declaration of dividends,
3) the effect of the distribution, as compared with the declaration
of regular dividend,
4) the lapse of time between issuance and redemption, 86
5) the presence of a substantial surplus 87 and a generous supply
of cash which invites suspicion as does a meager policy in relation
both to current earnings and accumulated surplus, 88
REDEMPTION AND CANCELLATION
For the exempting clause of Section, 83(b) to apply, it is indispensable that: (a) there is
redemption or cancellation; (b) the transaction involves stock dividends and (c) the "time
and manner" of the transaction makes it "essentially equivalent to a distribution of
taxable dividends." Of these, the most important is the third.
Redemption is repurchase, a reacquisition of stock by a corporation which issued the stock 89 in
exchange for property, whether or not the acquired stock is cancelled, retired or held in the
treasury.90 Essentially, the corporation gets back some of its stock, distributes cash or property
to the shareholder in payment for the stock, and continues in business as before. The
redemption of stock dividends previously issued is used as a veil for the constructive distribution
of cash dividends. In the instant case, there is no dispute that ANSCOR redeemed shares of
stocks from a stockholder (Don Andres) twice (28,000 and 80,000 common shares). But where
did the shares redeemed come from? If its source is the original capital subscriptions upon
establishment of the corporation or from initial capital investment in an existing enterprise, its
redemption to the concurrent value of acquisition may not invite the application of Sec. 83(b)
under the 1939 Tax Code, as it is not income but a mere return of capital. On the contrary, if the
redeemed shares are from stock dividend declarations other than as initial capital investment,
the proceeds of the redemption is additional wealth, for it is not merely a return of capital but a
gain thereon.
It is not the stock dividends but the proceeds of its redemption that may be deemed as taxable
dividends. Here, it is undisputed that at the time of the last redemption, the original common
shares owned by the estate were only 25,247.5 91 This means that from the total of 108,000
shares redeemed from the estate, the balance of 82,752.5 (108,000 less 25,247.5) must have
come from stock dividends. Besides, in the absence of evidence to the contrary, the Tax Code
presumes that every distribution of corporate property, in whole or in part, is made out of
corporate profits 92such as stock dividends. The capital cannot be distributed in the form of
redemption of stock dividends without violating the trust fund doctrine wherein the capital
44

stock, property and other assets of the corporation are regarded as equity in trust for the
payment of the corporate creditors. 93 Once capital, it is always capital. 94 That doctrine was
intended for the protection of corporate creditors. 95
With respect to the third requisite, ANSCOR redeemed stock dividends issued just 2 to 3 years
earlier. The time alone that lapsed from the issuance to the redemption is not a sufficient
indicator to determine taxability. It is a must to consider the factual circumstances as to the
manner of both the issuance and the redemption. The "time" element is a factor to show a
device to evade tax and the scheme of cancelling or redeeming the same shares is a method
usually adopted to accomplish the end sought. 96 Was this transaction used as a "continuing
plan," "device" or "artifice" to evade payment of tax? It is necessary to determine the "net effect"
of the transaction between the shareholder-income taxpayer and the acquiring (redeeming)
corporation. 97 The "net effect" test is not evidence or testimony to be considered; it is rather an
inference to be drawn or a conclusion to be reached. 98 It is also important to know whether the
issuance of stock dividends was dictated by legitimate business reasons, the presence of which
might negate a tax evasion plan. 99
The issuance of stock dividends and its subsequent redemption must be separate, distinct, and
not related, for the redemption to be considered a legitimate tax scheme. 100 Redemption cannot
be used as a cloak to distribute corporate earnings. 101 Otherwise, the apparent intention to
avoid tax becomes doubtful as the intention to evade becomes manifest. It has been ruled that:
[A]n operation with no business or corporate purpose is a mere devise which
put on the form of a corporate reorganization as a disguise for concealing its real
character, and the sole object and accomplishment of which was the
consummation of a preconceived plan, not to reorganize a business or any part
of a business, but to transfer a parcel of corporate shares to a stockholder. 102
Depending on each case, the exempting provision of Sec. 83(b) of the 1939 Code may not be
applicable if the redeemed shares were issued with bona fide business purpose, 103 which is
judged after each and every step of the transaction have been considered and the whole
transaction does not amount to a tax evasion scheme.
ANSCOR invoked two reasons to justify the redemptions (1) the alleged "filipinization"
program and (2) the reduction of foreign exchange remittances in case cash dividends are
declared. The Court is not concerned with the wisdom of these purposes but on their relevance
to the whole transaction which can be inferred from the outcome thereof. Again, it is the "net
effect rather than the motives and plans of the taxpayer or his corporation" 104 that is the
fundamental guide in administering Sec. 83(b). This tax provision is aimed at the result. 105 It
also applies even if at the time of the issuance of the stock dividend, there was no intention to
redeem it as a means of distributing profit or avoiding tax on dividends. 106 The existence of
legitimate business purposes in support of the redemption of stock dividends is immaterial in
income taxation. It has no relevance in determining "dividend equivalence". 107 Such purposes
may be material only upon the issuance of the stock dividends. The test of taxability under the
exempting clause, when it provides "such time and manner" as would make the redemption
"essentially equivalent to the distribution of a taxable dividend", is whether the redemption
resulted into a flow of wealth. If no wealth is realized from the redemption, there may not be a
dividend equivalence treatment. In the metaphor of Eisner v. Macomber, income is not deemed
"realize" until the fruit has fallen or been plucked from the tree.
The three elements in the imposition of income tax are: (1) there must be gain or and profit, (2)
that the gain or profit is realized or received, actually or constructively, 108 and (3) it is not
exempted by law or treaty from income tax. Any business purpose as to why or how the income
45

was earned by the taxpayer is not a requirement. Income tax is assessed on income received
from any property, activity or service that produces the income because the Tax Code stands as
an indifferent neutral party on the matter of where income comes from. 109
As stated above, the test of taxability under the exempting clause of Section 83(b) is, whether
income was realized through the redemption of stock dividends. The redemption converts into
money the stock dividends which become a realized profit or gain and consequently, the
stockholder's separate property. 110 Profits derived from the capital invested cannot escape
income tax. As realized income, the proceeds of the redeemed stock dividends can be reached
by income taxation regardless of the existence of any business purpose for the redemption.
Otherwise, to rule that the said proceeds are exempt from income tax when the redemption is
supported by legitimate business reasons would defeat the very purpose of imposing tax on
income. Such argument would open the door for income earners not to pay tax so long as the
person from whom the income was derived has legitimate business reasons. In other words, the
payment of tax under the exempting clause of Section 83(b) would be made to depend not on
the income of the taxpayer, but on the business purposes of a third party (the corporation
herein) from whom the income was earned. This is absurd, illogical and impractical considering
that the Bureau of Internal Revenue (BIR) would be pestered with instances in determining the
legitimacy of business reasons that every income earner may interposed. It is not
administratively feasible and cannot therefore be allowed.
The ruling in the American cases cited and relied upon by ANSCOR that "the redeemed shares
are the equivalent of dividend only if the shares were not issued for genuine business
purposes", 111 or the "redeemed shares have been issued by a corporation bona fide" 112 bears
no relevance in determining the non-taxability of the proceeds of redemption ANSCOR, relying
heavily and applying said cases, argued that so long as the redemption is supported by valid
corporate purposes the proceeds are not subject to tax. 113 The adoption by the courts
below 114 of such argument is misleading if not misplaced. A review of the cited American cases
shows that the presence or absence of "genuine business purposes" may be material with
respect to the issuance or declaration of stock dividends but not on its subsequent redemption.
The issuance and the redemption of stocks are two different transactions. Although the
existence of legitimate corporate purposes may justify a corporation's acquisition of its own
shares under Section 41 of the Corporation Code, 115such purposes cannot excuse the
stockholder from the effects of taxation arising from the redemption. If the issuance of stock
dividends is part of a tax evasion plan and thus, without legitimate business reasons, the
redemption becomes suspicious which exempting clause. The substance of the whole
transaction, not its form, usually controls the tax consequences. 116
The two purposes invoked by ANSCOR, under the facts of this case are no excuse for its tax
liability. First, the alleged "filipinization" plan cannot be considered legitimate as it was not
implemented until the BIR started making assessments on the proceeds of the redemption.
Such corporate plan was not stated in nor supported by any Board Resolution but a mere
afterthought interposed by the counsel of ANSCOR. Being a separate entity, the corporation can
act only through its Board of Directors. 117 The Board Resolutions authorizing the redemptions
state only one purpose reduction of foreign exchange remittances in case cash dividends are
declared. Not even this purpose can be given credence. Records show that despite the
existence of enormous corporate profits no cash dividend was ever declared by ANSCOR from
1945 until the BIR started making assessments in the early 1970's. Although a corporation
under certain exceptions, has the prerogative when to issue dividends, yet when no cash
dividends was issued for about three decades, this circumstance negates the legitimacy of
ANSCOR's alleged purposes. Moreover, to issue stock dividends is to increase the
shareholdings of ANSCOR's foreign stockholders contrary to its "filipinization" plan. This would
also increase rather than reduce their need for foreign exchange remittances in case of cash
46

dividend declaration, considering that ANSCOR is a family corporation where the majority
shares at the time of redemptions were held by Don Andres' foreign heirs.
Secondly, assuming arguendo, that those business purposes are legitimate, the same cannot
be a valid excuse for the imposition of tax. Otherwise, the taxpayer's liability to pay income tax
would be made to depend upon a third person who did not earn the income being taxed.
Furthermore, even if the said purposes support the redemption and justify the issuance of stock
dividends, the same has no bearing whatsoever on the imposition of the tax herein assessed
because the proceeds of the redemption are deemed taxable dividends since it was shown that
income was generated therefrom.
Thirdly, ANSCOR argued that to treat as "taxable dividend" the proceeds of the redeemed stock
dividends would be to impose on such stock an undisclosed lien and would be extremely unfair
to intervening purchase, i.e. those who buys the stock dividends after their issuance. 118 Such
argument, however, bears no relevance in this case as no intervening buyer is involved. And
even if there is an intervening buyer, it is necessary to look into the factual milieu of the case if
income was realized from the transaction. Again, we reiterate that the dividend equivalence test
depends on such "time and manner" of the transaction and its net effect. The undisclosed
lien 119 may be unfair to a subsequent stock buyer who has no capital interest in the company.
But the unfairness may not be true to an original subscriber like Don Andres, who holds stock
dividends as gains from his investments. The subsequent buyer who buys stock dividends is
investing capital. It just so happen that what he bought is stock dividends. The effect of its (stock
dividends) redemption from that subsequent buyer is merely to return his capital subscription,
which is income if redeemed from the original subscriber.
After considering the manner and the circumstances by which the issuance and redemption of
stock dividends were made, there is no other conclusion but that the proceeds thereof are
essentially considered equivalent to a distribution of taxable dividends. As "taxable dividend"
under Section 83(b), it is part of the "entire income" subject to tax under Section 22 in relation to
Section 21 120 of the 1939 Code. Moreover, under Section 29(a) of said Code, dividends are
included in "gross income". As income, it is subject to income tax which is required to be
withheld at source. The 1997 Tax Code may have altered the situation but it does not change
this disposition.
EXCHANGE OF COMMON WITH PREFERRED SHARES 121
Exchange is an act of taking or giving one thing for another involving 122 reciprocal
transfer 123 and is generally considered as a taxable transaction. The exchange of common
stocks with preferred stocks, or preferred for common or a combination of either for both, may
not produce a recognized gain or loss, so long as the provisions of Section 83(b) is not
applicable. This is true in a trade between two (2) persons as well as a trade between a
stockholder and a corporation. In general, this trade must be parts of merger, transfer to
controlled corporation, corporate acquisitions or corporate reorganizations. No taxable gain or
loss may be recognized on exchange of property, stock or securities related to
reorganizations. 124
Both the Tax Court and the Court of Appeals found that ANSCOR reclassified its shares into
common and preferred, and that parts of the common shares of the Don Andres estate and all
of Doa Carmen's shares were exchanged for the whole 150.000 preferred shares. Thereafter,
both the Don Andres estate and Doa Carmen remained as corporate subscribers except that
their subscriptions now include preferred shares. There was no change in their proportional
interest after the exchange. There was no cash flow. Both stocks had the same par value.
Under the facts herein, any difference in their market value would be immaterial at the time of
47

exchange because no income is yet realized it was a mere corporate paper transaction. It
would have been different, if the exchange transaction resulted into a flow of wealth, in which
case income tax may be imposed. 125
Reclassification of shares does not always bring any substantial alteration in the subscriber's
proportional interest. But the exchange is different there would be a shifting of the balance of
stock features, like priority in dividend declarations or absence of voting rights. Yet neither the
reclassification nor exchange per se, yields realize income for tax purposes. A common stock
represents the residual ownership interest in the corporation. It is a basic class of stock
ordinarily and usually issued without extraordinary rights or privileges and entitles the
shareholder to a pro rata division of profits. 126 Preferred stocks are those which entitle the
shareholder to some priority on dividends and asset distribution. 127
Both shares are part of the corporation's capital stock. Both stockholders are no different from
ordinary investors who take on the same investment risks. Preferred and common shareholders
participate in the same venture, willing to share in the profits and losses of the
enterprise. 128 Moreover, under the doctrine of equality of shares all stocks issued by the
corporation are presumed equal with the same privileges and liabilities, provided that the
Articles of Incorporation is silent on such differences. 129
In this case, the exchange of shares, without more, produces no realized income to the
subscriber. There is only a modification of the subscriber's rights and privileges which is not a
flow of wealth for tax purposes. The issue of taxable dividend may arise only once a subscriber
disposes of his entire interest and not when there is still maintenance of proprietary interest. 130
WHEREFORE, premises considered, the decision of the Court of Appeals is MODIFIED in that
ANSCOR's redemption of 82,752.5 stock dividends is herein considered as essentially
equivalent to a distribution of taxable dividends for which it is LIABLE for the withholding tax-atsource. The decision is AFFIRMED in all other respects. SO ORDERED.
Republic of the Philippines
SUPREME COURT
Manila
FIRST DIVISION
G.R. Nos. L-28508-9 July 7, 1989
ESSO STANDARD EASTERN, INC., (formerly, Standard-Vacuum Oil Company), petitioner,
vs.
THE COMMISSIONER OF INTERNAL REVENUE, respondent.
Padilla Law Office for petitioner.
CRUZ, J.:
On appeal before us is the decision of the Court of Tax Appeals 1 denying petitioner's claims for
refund of overpaid income taxes of P102,246.00 for 1959 and P434,234.93 for 1960 in CTA
Cases No. 1251 and 1558 respectively.
I
48

In CTA Case No. 1251, petitioner ESSO deducted from its gross income for 1959, as part of its
ordinary and necessary business expenses, the amount it had spent for drilling and exploration
of its petroleum concessions. This claim was disallowed by the respondent Commissioner of
Internal Revenue on the ground that the expenses should be capitalized and might be written off
as a loss only when a "dry hole" should result. ESSO then filed an amended return where it
asked for the refund of P323,279.00 by reason of its abandonment as dry holes of several of its
oil wells. Also claimed as ordinary and necessary expenses in the same return was the amount
of P340,822.04, representing margin fees it had paid to the Central Bank on its profit
remittances to its New York head office.
On August 5, 1964, the CIR granted a tax credit of P221,033.00 only, disallowing the claimed
deduction for the margin fees paid.
In CTA Case No. 1558, the CR assessed ESSO a deficiency income tax for the year 1960, in
the amount of P367,994.00, plus 18% interest thereon of P66,238.92 for the period from April
18,1961 to April 18, 1964, for a total of P434,232.92. The deficiency arose from the
disallowance of the margin fees of Pl,226,647.72 paid by ESSO to the Central Bank on its profit
remittances to its New York head office.
ESSO settled this deficiency assessment on August 10, 1964, by applying the tax credit of
P221,033.00 representing its overpayment on its income tax for 1959 and paying under protest
the additional amount of P213,201.92. On August 13, 1964, it claimed the refund of P39,787.94
as overpayment on the interest on its deficiency income tax. It argued that the 18% interest
should have been imposed not on the total deficiency of P367,944.00 but only on the amount of
P146,961.00, the difference between the total deficiency and its tax credit of P221,033.00.
This claim was denied by the CIR, who insisted on charging the 18% interest on the entire
amount of the deficiency tax. On May 4,1965, the CIR also denied the claims of ESSO for
refund of the overpayment of its 1959 and 1960 income taxes, holding that the margin fees paid
to the Central Bank could not be considered taxes or allowed as deductible business expenses.
ESSO appealed to the CTA and sought the refund of P102,246.00 for 1959, contending that the
margin fees were deductible from gross income either as a tax or as an ordinary and necessary
business expense. It also claimed an overpayment of its tax by P434,232.92 in 1960, for the
same reason. Additionally, ESSO argued that even if the amount paid as margin fees were not
legally deductible, there was still an overpayment by P39,787.94 for 1960, representing excess
interest.
After trial, the CTA denied petitioner's claim for refund of P102,246.00 for 1959 and P434,234.92
for 1960 but sustained its claim for P39,787.94 as excess interest. This portion of the decision
was appealed by the CIR but was affirmed by this Court in Commissioner of Internal Revenue v.
ESSO, G.R. No. L-28502- 03, promulgated on April 18, 1989. ESSO for its part appealed the
CTA decision denying its claims for the refund of the margin fees P102,246.00 for 1959 and
P434,234.92 for 1960. That is the issue now before us.
II
The first question we must settle is whether R.A. 2009, entitled An Act to Authorize the Central
Bank of the Philippines to Establish a Margin Over Banks' Selling Rates of Foreign Exchange, is
a police measure or a revenue measure. If it is a revenue measure, the margin fees paid by the
petitioner to the Central Bank on its profit remittances to its New York head office should be
deductible from ESSO's gross income under Sec. 30(c) of the National Internal Revenue Code.
49

This provides that all taxes paid or accrued during or within the taxable year and which are
related to the taxpayer's trade, business or profession are deductible from gross income.
The petitioner maintains that margin fees are taxes and cites the background and legislative
history of the Margin Fee Law showing that R.A. 2609 was nothing less than a revival of the
17% excise tax on foreign exchange imposed by R.A. 601. This was a revenue measure
formally proposed by President Carlos P. Garcia to Congress as part of, and in order to balance,
the budget for 1959-1960. It was enacted by Congress as such and, significantly, properly
originated in the House of Representatives. During its two and a half years of existence, the
measure was one of the major sources of revenue used to finance the ordinary operating
expenditures of the government. It was, moreover, payable out of the General Fund.
On the claimed legislative intent, the Court of Tax Appeals, quoting established principles,
pointed out that
We are not unmindful of the rule that opinions expressed in debates, actual proceedings of the
legislature, steps taken in the enactment of a law, or the history of the passage of the law
through the legislature, may be resorted to as an aid in the interpretation of a statute which is
ambiguous or of doubtful meaning. The courts may take into consideration the facts leading up
to, coincident with, and in any way connected with, the passage of the act, in order that they
may properly interpret the legislative intent. But it is also well-settled jurisprudence that only in
extremely doubtful matters of interpretation does the legislative history of an act of Congress
become important. As a matter of fact, there may be no resort to the legislative history of the
enactment of a statute, the language of which is plain and unambiguous, since such legislative
history may only be resorted to for the purpose of solving doubt, not for the purpose of creating
it. [50 Am. Jur. 328.]
Apart from the above consideration, there are at least two cases where we have held that a
margin fee is not a tax but an exaction designed to curb the excessive demands upon our
international reserve.
In Caltex (Phil.) Inc. v. Acting Commissioner of Customs, 2 the Court stated through Justice Jose
P. Bengzon:
A margin levy on foreign exchange is a form of exchange control or restriction
designed to discourage imports and encourage exports, and ultimately, 'curtail
any excessive demand upon the international reserve' in order to stabilize the
currency. Originally adopted to cope with balance of payment pressures,
exchange restrictions have come to serve various purposes, such as limiting
non-essential imports, protecting domestic industry and when combined with the
use of multiple currency rates providing a source of revenue to the government,
and are in many developing countries regarded as a more or less inevitable
concomitant of their economic development programs. The different measures of
exchange control or restriction cover different phases of foreign exchange
transactions, i.e., in quantitative restriction, the control is on the amount of foreign
exchange allowable. In the case of the margin levy, the immediate impact is on
the rate of foreign exchange; in fact, its main function is to control the exchange
rate without changing the par value of the peso as fixed in the Bretton Woods
Agreement Act. For a member nation is not supposed to alter its exchange rate
(at par value) to correct a merely temporary disequilibrium in its balance of
payments. By its nature, the margin levy is part of the rate of exchange as fixed
by the government.
50

As to the contention that the margin levy is a tax on the purchase of foreign exchange and
hence should not form part of the exchange rate, suffice it to state that We have already held
the contrary for the reason that a tax is levied to provide revenue for government operations,
while the proceeds of the margin fee are applied to strengthen our country's international
reserves.
Earlier, in Chamber of Agriculture and Natural Resources of the Philippines v. Central
Bank, 3 the same idea was expressed, though in connection with a different levy, through Justice
J.B.L. Reyes:
Neither do we find merit in the argument that the 20% retention of exporter's
foreign exchange constitutes an export tax. A tax is a levy for the purpose of
providing revenue for government operations, while the proceeds of the 20%
retention, as we have seen, are applied to strengthen the Central Bank's
international reserve.
We conclude then that the margin fee was imposed by the State in the exercise of its police
power and not the power of taxation.
Alternatively, ESSO prays that if margin fees are not taxes, they should nevertheless be
considered necessary and ordinary business expenses and therefore still deductible from its
gross income. The fees were paid for the remittance by ESSO as part of the profits to the head
office in the Unites States. Such remittance was an expenditure necessary and proper for the
conduct of its corporate affairs.
The applicable provision is Section 30(a) of the National Internal Revenue Code reading as
follows:
SEC. 30. Deductions from gross income in computing net income there shall be
allowed as deductions
(a) Expenses:
(1) In general. All the ordinary and necessary expenses paid or incurred
during the taxable year in carrying on any trade or business, including a
reasonable allowance for salaries or other compensation for personal services
actually rendered; traveling expenses while away from home in the pursuit of a
trade or business; and rentals or other payments required to be made as a
condition to the continued use or possession, for the purpose of the trade or
business, of property to which the taxpayer has not taken or is not taking title or
in which he has no equity.
(2) Expenses allowable to non-resident alien individuals and foreign corporations.
In the case of a non-resident alien individual or a foreign corporation, the
expenses deductible are the necessary expenses paid or incurred in carrying on
any business or trade conducted within the Philippines exclusively.
In the case of Atlas Consolidated Mining and Development Corporation v. Commissioner of
Internal Revenue, 4the Court laid down the rules on the deductibility of business expenses, thus:
The principle is recognized that when a taxpayer claims a deduction, he must
point to some specific provision of the statute in which that deduction is
51

authorized and must be able to prove that he is entitled to the deduction which
the law allows. As previously adverted to, the law allowing expenses as
deduction from gross income for purposes of the income tax is Section 30(a) (1)
of the National Internal Revenue which allows a deduction of 'all the ordinary and
necessary expenses paid or incurred during the taxable year in carrying on any
trade or business.' An item of expenditure, in order to be deductible under this
section of the statute, must fall squarely within its language.
We come, then, to the statutory test of deductibility where it is axiomatic that to
be deductible as a business expense, three conditions are imposed, namely: (1)
the expense must be ordinary and necessary, (2) it must be paid or incurred
within the taxable year, and (3) it must be paid or incurred in carrying on a trade
or business. In addition, not only must the taxpayer meet the business test, he
must substantially prove by evidence or records the deductions claimed under
the law, otherwise, the same will be disallowed. The mere allegation of the
taxpayer that an item of expense is ordinary and necessary does not justify its
deduction.
While it is true that there is a number of decisions in the United States delving on
the interpretation of the terms 'ordinary and necessary' as used in the federal tax
laws, no adequate or satisfactory definition of those terms is possible. Similarly,
this Court has never attempted to define with precision the terms 'ordinary and
necessary.' There are however, certain guiding principles worthy of serious
consideration in the proper adjudication of conflicting claims. Ordinarily, an
expense will be considered 'necessary' where the expenditure is appropriate and
helpful in the development of the taxpayer's business. It is 'ordinary' when it
connotes a payment which is normal in relation to the business of the taxpayer
and the surrounding circumstances. The term 'ordinary' does not require that the
payments be habitual or normal in the sense that the same taxpayer will have to
make them often; the payment may be unique or non-recurring to the particular
taxpayer affected.
There is thus no hard and fast rule on the matter. The right to a deduction
depends in each case on the particular facts and the relation of the payment to
the type of business in which the taxpayer is engaged. The intention of the
taxpayer often may be the controlling fact in making the determination. Assuming
that the expenditure is ordinary and necessary in the operation of the taxpayer's
business, the answer to the question as to whether the expenditure is an
allowable deduction as a business expense must be determined from the nature
of the expenditure itself, which in turn depends on the extent and permanency of
the work accomplished by the expenditure.
In the light of the above explanation, we hold that the Court of Tax Appeals did not err when it
held on this issue as follows:
Considering the foregoing test of what constitutes an ordinary and necessary
deductible expense, it may be asked: Were the margin fees paid by petitioner on
its profit remittance to its Head Office in New York appropriate and helpful in the
taxpayer's business in the Philippines? Were the margin fees incurred for
purposes proper to the conduct of the affairs of petitioner's branch in the
Philippines? Or were the margin fees incurred for the purpose of realizing a profit
or of minimizing a loss in the Philippines? Obviously not. As stated in the Lopez
case, the margin fees are not expenses in connection with the production or
52

earning of petitioner's incomes in the Philippines. They were expenses incurred


in the disposition of said incomes; expenses for the remittance of funds after they
have already been earned by petitioner's branch in the Philippines for the
disposal of its Head Office in New York which is already another distinct and
separate income taxpayer.
xxx
Since the margin fees in question were incurred for the remittance of funds to
petitioner's Head Office in New York, which is a separate and distinct income
taxpayer from the branch in the Philippines, for its disposal abroad, it can never
be said therefore that the margin fees were appropriate and helpful in the
development of petitioner's business in the Philippines exclusively or were
incurred for purposes proper to the conduct of the affairs of petitioner's branch in
the Philippines exclusively or for the purpose of realizing a profit or of minimizing
a loss in the Philippines exclusively. If at all, the margin fees were incurred for
purposes proper to the conduct of the corporate affairs of Standard Vacuum Oil
Company in New York, but certainly not in the Philippines.
ESSO has not shown that the remittance to the head office of part of its profits was made in
furtherance of its own trade or business. The petitioner merely presumed that all corporate
expenses are necessary and appropriate in the absence of a showing that they are illegal
or ultra vires. This is error. The public respondent is correct when it asserts that "the paramount
rule is that claims for deductions are a matter of legislative grace and do not turn on mere
equitable considerations ... . The taxpayer in every instance has the burden of justifying the
allowance of any deduction claimed." 5
It is clear that ESSO, having assumed an expense properly attributable to its head office, cannot
now claim this as an ordinary and necessary expense paid or incurred in carrying on its own
trade or business.
WHEREFORE, the decision of the Court of Tax Appeals denying the petitioner's claims for
refund of P102,246.00 for 1959 and P434,234.92 for 1960, is AFFIRMED, with costs against the
petitioner.

53

Republic of the Philippines


SUPREME COURT
Manila
EN BANC
G.R. No. L-15290

May 31, 1963

MARIANO ZAMORA, petitioner,


vs.
COLLECTOR OF INTERNAL REVENUE and COURT OF TAX APPEALS, respondents.
----------------------------G.R. No. L-15280

May 31, 1963

COLLECTOR OF INTERNAL REVENUE, petitioner,


vs.
MARIANO ZAMORA, respondent.
----------------------------G.R. No. L-15289

May 31, 1963


54

ESPERANZA A. ZAMORA, as Special Administratrix of Estate of FELICIDAD


ZAMORA, petitioner,
vs.
COLLECTOR OF INTERNAL REVENUE and COURT OF TAX APPEALS, respondents.
----------------------------G.R. No. L-15281

May 31, 1963

COLLECTOR OF INTERNAL REVENUE, petitioner,


vs.
ESPERANZA A. ZAMORA, as Special Administratrix, etc. respondent.
Office of the Solicitor General for petitioner.
Rodegelio M. Jalandoni for respondents.
PAREDES, J.:
In the above-entitled cases, a joint decision was rendered by the lower court because they
involved practically the same issues. We do so, likewise, for the same reason.
Cases Nos. L-15290 and L-15280
Mariano Zamora, owner of the Bay View Hotel and Farmacia Zamora, Manila, filed his income
tax returns the years 1951 and 1952. The Collector of Internal Revenue found that he failed to
file his return of the capital gains derived from the sale of certain real properties and claimed
deductions which were not allowable. The collector required him to pay the sums of P43,758.50
and P7,625.00, as deficiency income tax for the years 1951 and 1952, respectively (C.T.A. Case
No. 234, now L-15290). On appeal by Zamora, the Court of Tax Appeals on December 29,
1958, modified the decision appealed from and ordered him to pay the reduced total sum of
P30,258.00 (P22,980.00 and P7,278.00, as deficiency income tax for the years 1951 and 1952,
respectively), within thirty (30) days from the date the decision becomes final, plus the
corresponding surcharges and interest in case of delinquency, pursuant to section 51(e), Int.
Revenue Code. With costs against petitioner.
Having failed to obtain a reconsideration of the decision, Mariano Zamora appealed (L-15290),
alleging that the Court of Tax Appeals erred
(1) In dissallowing P10,478.50, as promotion expenses incurred by his wife for the
promotion of the Bay View Hotel and Farmacia Zamora (which is of P20,957.00,
supposed business expenses):
(2) In disallowing 3-% per annum as the rate of depreciation of the Bay View Hotel
Building;
(3) In disregarding the price stated in the deed of sale, as the costs of a Manila property,
for the purpose of determining alleged capital gains; and
(4) In applying the Ballantyne scale of values in determining the cost of said property.
The Collector of Internal Revenue (L-15280) also appealed, claiming that the Court of Tax
Appeals erred
55

(1) In giving credence to the uncorroborated testimony of Mariano Zamora that he


bought the said real property in question during the Japanese occupation, partly in
Philippine currency and partly in Japanese war notes, and
(2) In not holding that Mariano Zamora is liable for the payment of the sums of
P43,758.00 and P7,625.00 as deficiency income taxes, for the years 1951 and 1952,
plus the 5% surcharge and 1% monthly interest, from the date said amounts became
due to the date of actual payment.
Wherefore, the parties respectfully pray that the foregoing stipulation of facts be
admitted and approved by this Honorable Court, without prejudice to the parties
adducing other evidence to prove their case not covered by this stipulation of
facts. 1wph1.t
Cases Nos. L-15289 and L-15281
Mariano Zamora and his deceased sister Felicidad Zamora, bought a piece of land located in
Manila on May 16, 1944, for P132,000.00 and sold it for P75,000.00 on March 5, 1951. They
also purchased a lot located in Quezon City for P68,959.00 on January 19, 1944, which they
sold for P94,000 on February 9, 1951. The CTA ordered the estate of the late Felicidad Zamora
(represented by Esperanza A. Zamora, as special administratrix of her estate), to pay the sum
of P235.50, representing alleged deficiency income tax and surcharge due from said estate.
Esperanza A. Zamora appealed and alleged that the CTA erred:
The Commissioner of Internal Revenue likewise appealed from the decision, claiming that the
lower court erred:
(1) In giving credence to the uncorroborated testimony of Mariano Zamora that he
bought the real property involved during the Japanese occupation, partly in genuine
Philippine currency and partly in Japanese war notes; and
(2) In not holding that Esperanza A. Zamora, as administratrix, is liable for the payment
of the sum of P613.00 as deficiency income tax and 50% surcharge for 1951, plus 50%
surcharge and 1% monthly interest from the date said amount became due, to the date
of actual payment.
It is alleged by Mariano Zamora that the CTA erred in disallowing P10,478.50 as promotion
expenses incurred by his wife for the promotion of the Bay View Hotel and Farmacia Zamora.
He contends that the whole amount of P20,957.00 as promotion expenses in his 1951 income
tax returns, should be allowed and not merely one-half of it or P10,478.50, on the ground that,
while not all the itemized expenses are supported by receipts, the absence of some supporting
receipts has been sufficiently and satisfactorily established. For, as alleged, the said amount of
P20,957.00 was spent by Mrs. Esperanza A. Zamora (wife of Mariano), during her travel to
Japan and the United States to purchase machinery for a new Tiki-Tiki plant, and to observe
hotel management in modern hotels. The CTA, however, found that for said trip Mrs. Zamora
obtained only the sum of P5,000.00 from the Central Bank and that in her application for dollar
allocation, she stated that she was going abroad on a combined medical and business trip,
which facts were not denied by Mariano Zamora. No evidence had been submitted as to where
Mariano had obtained the amount in excess of P5,000.00 given to his wife which she spent
abroad. No explanation had been made either that the statement contained in Mrs. Zamora's
application for dollar allocation that she was going abroad on a combined medical and business
trip, was not correct. The alleged expenses were not supported by receipts. Mrs. Zamora could
56

not even remember how much money she had when she left abroad in 1951, and how the
alleged amount of P20,957.00 was spent.
Section 30, of the Tax Code, provides that in computing net income, there shall be allowed as
deductions all the ordinary and necessary expenses paid or incurred during the taxable year, in
carrying on any trade or business (Vol. 4, Mertens, Law of Federal Income Taxation, sec. 25.03,
p. 307). Since promotion expenses constitute one of the deductions in conducting a business,
same must testify these requirements. Claim for the deduction of promotion expenses or
entertainment expenses must also be substantiated or supported by record showing in detail the
amount and nature of the expenses incurred (N.H. Van Socklan, Jr. v. Comm. of Int. Rev.; 33
BTA 544). Considering, as heretofore stated, that the application of Mrs. Zamora for dollar
allocation shows that she went abroad on a combined medical and business trip, not all of her
expenses came under the category of ordinary and necessary expenses; part thereof
constituted her personal expenses. There having been no means by which to ascertain which
expense was incurred by her in connection with the business of Mariano Zamora and which was
incurred for her personal benefit, the Collector and the CTA in their decisions, considered 50%
of the said amount of P20,957.00 as business expenses and the other 50%, as her personal
expenses. We hold that said allocation is very fair to Mariano Zamora, there having been no
receipt whatsoever, submitted to explain the alleged business expenses, or proof of the
connection which said expenses had to the business or the reasonableness of the said amount
of P20,957.00. While in situations like the present, absolute certainty is usually no possible, the
CTA should make as close an approximation as it can, bearing heavily, if it chooses, upon the
taxpayer whose inexactness is of his own making.
In the case of Visayan Cebu Terminal Co., Inc. v. Collector of Int. Rev., G.R. No. L-12798, May
30, 1960, it was declared that representation expenses fall under the category of business
expenses which are allowable deductions from gross income, if they meet the conditions
prescribed by law, particularly section 30 (a) [1], of the Tax Code; that to be deductible, said
business expenses must be ordinary and necessary expenses paid or incurred in carrying on
any trade or business; that those expenses must also meet the further test of reasonableness in
amount; that when some of the representation expenses claimed by the taxpayer were
evidenced by vouchers or chits, but others were without vouchers or chits, documents or
supporting papers; that there is no more than oral proof to the effect that payments have been
made for representation expenses allegedly made by the taxpayer and about the general nature
of such alleged expenses; that accordingly, it is not possible to determine the actual amount
covered by supporting papers and the amount without supporting papers, the court should
determine from all available data, the amount properly deductible as representation expenses.
In view hereof, We are of the opinion that the CTA, did not commit error in allowing as promotion
expenses of Mrs. Zamora claimed in Mariano Zamora's 1951 income tax returns, merely onehalf or P10,478.50.
Petitioner Mariano Zamora alleges that the CTA erred in disallowing 3-% per annum as the
rate of depreciation of the Bay View Hotel Building but only 2-%. In justifying depreciation
deduction of 3-%, Mariano Zamora contends that (1) the Ermita District, where the Bay View
Hotel is located, is now becoming a commercial district; (2) the hotel has no room for
improvement; and (3) the changing modes in architecture, styles of furniture and decorative
designs, "must meet the taste of a fickle public". It is a fact, however, that the CTA, in estimating
the reasonable rate of depreciation allowance for hotels made of concrete and steel at 2-%,
the three factors just mentioned had been taken into account already. Said the CTA
Normally, an average hotel building is estimated to have a useful life of 50 years, but
inasmuch as the useful life of the building for business purposes depends to a large
57

extent on the suitability of the structure to its use and location, its architectural quality,
the rate of change in population, the shifting of land values, as well as the extent and
maintenance and rehabilitation. It is allowed a depreciation rate of 2-% corresponding
to a normal useful life of only 40 years (1955 PH Federal Taxes, Par 14 160-K).
Consequently, the stand of the petitioners can not be sustained.
As the lower court based its findings on Bulletin F, petitioner Zamora, argues that the same
should have been first proved as a law, to be subject to judicial notice. Bulletin F, is a publication
of the US Federal Internal Revenue Service, which was made after a study of the lives of the
properties. In the words of the lower court: "It contains the list of depreciable assets, the
estimated average useful lives thereof and the rates of depreciation allowable for each kind of
property. (See 1955 PH Federal Taxes, Par. 14, 160 to Par. 14, 163-0). It is true that Bulletin F
has no binding force, but it has a strong persuasive effect considering that the same has been
the result of scientific studies and observation for a long period in the United States after whose
Income Tax Law ours is patterned." Verily, courts are permitted to look into and investigate the
antecedents or the legislative history of the statutes involved (Director of Lands v. Abaya, et al.,
63 Phil. 559). Zamora also contends that his basis for applying the 3-% rate is the testimony of
its witness Mariano Katipunan, who cited a book entitled "Hotel Management Principles and
Practice" by Lucius Boomer, President, Hotel Waldorf Astoria Corporation. As well commented
by the Solicitor General, "while the petitioner would deny us the right to use Bulletin F, he would
insist on using as authority, a book in Hotel management written by a man who knew more
about hotels than about taxation. All that the witness did (Katipunan) . . . is to read excerpts from
the said book (t.s.n. pp. 99-101), which admittedly were based on the decision of the U.S. Tax
Courts, made in 1928 (t.s.n. p. 106)". In view hereof, We hold that the 2-% rate of depreciation
of the Bay View Hotel building, is approximately correct.
The next items in dispute are the undeclared capital gains derived from the sales in 1951 of
certain real properties in Malate, Manila and in Quezon City, acquired during the Japanese
occupation.
The Manila property (Esperanza Zamora v. Coll. of Int. Rev., Case No. L-15289). The CTA held
in this case, that the cost basis of property acquired in Japanese war notes is the equivalent of
the war notes in genuine Philippine currency in accordance with the Ballantyne Scale of values,
and that the determination of the gain derived or loss sustained in the sale of such property is
not affected by the decline at the time of sale, in the purchasing power of the Philippine
currency. It was found by the CTA that the purchase price of P132,000.00 was not entirely paid
in Japanese War notes but thereof or P66,000.00 was in Philippine currency, and that during
certain periods of the enemy occupation, the value of the Japanese war notes was very much
less than the value of the genuine Philippine currency. On this point, the CTA declared
Finally, it is alleged that the purchase price of P132,000.00 was not entirely paid in
Japanese war notes, Mariano Zamora, co-owner of the property in question, testified
that P66,000.00 was paid in Philippine currency and the other P66,000.00 was paid in
Japanese war notes. No evidence was presented by respondent to rebut the testimony
of Mariano Zamora; it is assailed merely as being improbable. We have examined this
question thoroughly and we are inclined to give credence to the allegation that a portion
of the purchase price of the property was paid in Philippine money. In the first place, it
appears that the Zamoras owned the Farmacia Zamora which continued to engage in
business during the war years and that a considerable portion of its sales was paid for in
genuine Philippine currency. This circumstance enabled the Zamoras to accumulate
Philippine money which they used in acquiring the property in question and another
property in Quezon City. In the second place, P132,000.00 in Japanese war notes in
May, 1944 is equivalent to only P11,000.00. The property in question had at the time an
58

assessed value of P27,031.00 (in Philippine currency). Considering the well known fact
that the assessed value of real property is very much below the fair market value, it is
incredible that said property should have been sold by the owner thereof for less than
one-half of its assessed value. These facts have convinced us of the veracity of the
allegation that of the purchase price of P132,000.00 the sum of P66,000.00 was paid in
Philippine currency, so that only the sum of P66,000.00 was paid in Japanese War
notes.
This being the case, the Ballantyne Scale of values, which was the result of an impartial
scientific study, adopted and given judicial recognition, should be applied. As the value of the
Japanese war notes in May, 1944 when the Manila property was bought, was 1 of the
genuine Philippine Peso (Ballantyne Scale), and since the gain derived or loss sustained in the
disposition of this property is to reckoned in terms of Philippine Peso, the value of the Japanese
war notes used in the purchase of the property, must be reduced in terms of the genuine
Philippine Peso to determine the cost of acquisition. It, therefore, results that since the sum of
P66,000.00 in Japanese war notes in May, 1944 is equivalent to P5,500.00 in Philippine
currency (P66,000.00 divided by 12), the acquisition cost of the property in question is
P66,000.00 plus P5,500.00 or P71,500.00 and that as the property was sold for P75,000.00 in
1951, the owners thereof Mariano and Felicidad Zamora derived a capital gain of P3,500.00 or
P1,750.00 each.
The Quezon City Property (Mariano Zamora v. Coll. of Customs, Case No. 15290). The
Zamoras alleged that the entire purchase price of P68,959.00 was paid in Philippine currency.
The collector, on the other hand, contends that the purchase price of P68,959.00 was paid in
Japanese war notes. The CTA, however, giving credence to Zamora's version, said
. . . If , as contended by respondent, the purchase price of P68,959.00 was paid in
Japanese war notes, the purchase price in Philippine currency would be only
P17,239.75 (P68,959.00 divided by 4, 34.00 in war notes being equivalent to P1.00 in
Philippine currency). The assessed value of said property in Philippine currency at the
time of acquisition was P46,910.00. It is quite incredible that real property with an
assessed value of P46,910.00 should have been sold by the owner thereof in Japanese
war notes with an equivalent value in Philippine currency of only P17,239.75. We are
more inclined to believe the allegation that it was purchased for P68,959.00 in genuine
Philippine currency. Since the property was sold for P94,000.00 on February 9, 1951,
the gain derived from the sale is P15,361.75, after deducting from the selling price the
cost of acquisition in the sum of P68,959.00 and the expense of sale in the sum of
P9,679.25.
The above appraisal is correct, and We have no plausible reason to disturb the same.
Consequently, the total undeclared income of petitioners derived from the sales of the Manila
and Quezon City properties in 1951 is P17,111.75 (P1,750.00 plus P15,361.75), 50% of which in
the sum of P8,555.88 is taxable, the said properties being capital assets held for more than one
year.
IN VIEW HEREOF, the petition in each of the above-entitled cases is dismissed, and the
decision appealed from is affirmed, without special pronouncement as to costs.

59

Republic of the Philippines


SUPREME COURT
Manila
EN BANC
G.R. No. L-24059

November 28, 1969

C. M. HOSKINS & CO., INC., petitioner,


vs.
COMMISSIONER OF INTERNAL REVENUE, respondent.
Ross, Salcedo, Del Rosario, Bito and Misa for petitioner.
Office of the Solicitor General Arturo A. Alafriz, Assistant Solicitor General Felicisimo R. Rosete
and Special Attorney Michaelina R. Balasbas for respondent.
TEEHANKEE, J.:
We uphold in this taxpayer's appeal the Tax Court's ruling that payment by the taxpayer to its
controlling stockholder of 50% of its supervision fees or the amount of P99,977.91 is not a
deductible ordinary and necessary expense and should be treated as a distribution of earnings
and profits of the taxpayer.
Petitioner, a domestic corporation engaged in the real estate business as brokers, managing
agents and administrators, filed its income tax return for its fiscal year ending September 30,
1957 showing a net income of P92,540.25 and a tax liability due thereon of P18,508.00, which it
paid in due course. Upon verification of its return, respondent Commissioner of Internal
Revenue, disallowed four items of deduction in petitioner's tax returns and assessed against it
an income tax deficiency in the amount of P28,054.00 plus interests. The Court of Tax Appeals
60

upon reviewing the assessment at the taxpayer's petition, upheld respondent's disallowance of
the principal item of petitioner's having paid to Mr. C. M. Hoskins, its founder and controlling
stockholder the amount of P99,977.91 representing 50% of supervision fees earned by it and
set aside respondent's disallowance of three other minor items. The Tax Court therefore
determined petitioner's tax deficiency to be in the amount of P27,145.00 and on November 8,
1964 rendered judgment against it, as follows:
WHEREFORE, premises considered, the decision of the respondent is hereby modified.
Petitioner is ordered to pay to the latter or his representative the sum of P27,145.00,
representing deficiency income tax for the year 1957, plus interest at 1/2% per month
from June 20, 1959 to be computed in accordance with the provisions of Section 51(d) of
the National Internal Revenue Code. If the deficiency tax is not paid within thirty (30)
days from the date this decision becomes final, petitioner is also ordered to pay
surcharge and interest as provided for in Section 51 (e) of the Tax Code, without costs.
Petitioner questions in this appeal the Tax Court's findings that the disallowed payment to
Hoskins was an inordinately large one, which bore a close relationship to the recipient's
dominant stockholdings and therefore amounted in law to a distribution of its earnings and
profits.
We find no merit in petitioner's appeal.
As found by the Tax Court, "petitioner was founded by Mr. C. M. Hoskins in 1937, with a capital
stock of 1,000 shares at a par value of P1.00 each share; that of these 1,000 shares, Mr. C. M.
Hoskins owns 996 shares (the other 4 shares being held by the other four officers of the
corporation), which constitute exactly 99.6% of the total authorized capital stock (p. 92, t.s.n.);
that during the first four years of its existence, Mr. C. M. Hoskins was the President, but during
the taxable period in question, that is, from October 1, 1956 to September 30, 1957, he was the
chairman of the Board of Directors and salesman-broker for the company (p. 93, t.s.n.); that as
chairman of the Board of Directors, he received a salary of P3,750.00 a month, plus a salary
bonus of about P40,000.00 a year (p. 94, t.s.n.); that he was also a stockholder and officer of
the Paradise Farms, Inc. and Realty Investments, Inc., from which petitioner derived a large
portion of its income in the form of supervision fees and commissions earned on sales of lots
(pp. 97-99, t.s.n.; Financial Statements, attached to Exhibit '1', p. 11, BIR rec.); that as chairman
of the Board of Directors of petitioner, his duties were: "To act as a salesman; as a director,
preside over meetings and to get all of the real estate business I could for the company by
negotiating sales, purchases, making appraisals, raising funds to finance real estate operations
where that was necessary' (p. 96, t.s.n.); that he was familiar with the contract entered into by
the petitioner with the Paradise Farms, Inc. and the Realty Investments, Inc. by the terms of
which petitioner was 'to program the development, arrange financing, plan the proposed
subdivision as outlined in the prospectus of Paradise Farms, Inc., arrange contract for road
constructions, with the provision of water supply to all of the lots and in general to serve as
managing agents for the Paradise Farms, Inc. and subsequently for the Realty Investment, Inc."
(pp. 96-97. t.s.n.)
Considering that in addition to being Chairman of the board of directors of petitioner corporation,
which bears his name, Hoskins, who owned 99.6% of its total authorized capital stock while the
four other officers-stockholders of the firm owned a total of four-tenths of 1%, or one-tenth of 1%
each, with their respective nominal shareholdings of one share each was also salesman-broker
for his company, receiving a 50% share of the sales commissions earned by petitioner, besides
his monthly salary of P3,750.00 amounting to an annual compensation of P45,000.00 and an
annual salary bonus of P40,000.00, plus free use of the company car and receipt of other
similar allowances and benefits, the Tax Court correctly ruled that the payment by petitioner to
61

Hoskins of the additional sum of P99,977.91 as his equal or 50% share of the 8% supervision
fees received by petitioner as managing agents of the real estate, subdivision projects of
Paradise Farms, Inc. and Realty Investments, Inc. was inordinately large and could not be
accorded the treatment of ordinary and necessary expenses allowed as deductible items within
the purview of Section 30 (a) (i) of the Tax Code.
If such payment of P99,977.91 were to be allowed as a deductible item, then Hoskins would
receive on these three items alone (salary, bonus and supervision fee) a total of P184,977.91,
which would be double the petitioner's reported net income for the year of P92,540.25. As
correctly observed by respondent. If independently, a one-time P100,000.00-fee to plan and lay
down the rules for supervision of a subdivision project were to be paid to an experienced realtor
such as Hoskins, its fairness and deductibility by the taxpayer could be conceded; but here 50%
of the supervision fee of petitioner was being paid by it to Hoskins every year since 1955 up to
1963 and for as long as its contract with the subdivision owner subsisted, regardless of whether
services were actually rendered by Hoskins, since his services to petitioner included such
planning and supervision and were already handsomely paid for by petitioner.
The fact that such payment was authorized by a standing resolution of petitioner's board of
directors, since "Hoskins had personally conceived and planned the project" cannot change the
picture. There could be no question that as Chairman of the board and practically an absolutely
controlling stockholder of petitioner, holding 99.6% of its stock, Hoskins wielded tremendous
power and influence in the formulation and making of the company's policies and decisions.
Even just as board chairman, going by petitioner's own enumeration of the powers of the office,
Hoskins, could exercise great power and influence within the corporation, such as directing the
policy of the corporation, delegating powers to the president and advising the corporation in
determining executive salaries, bonus plans and pensions, dividend policies, etc.1
Petitioner's invoking of its policy since its incorporation of sharing equally sales commissions
with its salesmen, in accordance with its board resolution of June 18, 1946, is equally
untenable. Petitioner's Sales Regulations provide:
Compensation of Salesmen
8. Schedule I In the case of sales to prospects discovered and worked by a
salesman, even though the closing is done by or with the help of the Sales Manager or
other members of the staff, the salesmen get one-half (1/2) of the total commission
received by the Company, but not exceeding five percent (5%). In the case of
subdivisions, when the office commission covers general supervision, the 1/2-rule does
not apply, the salesman's share being stipulated in the case of each subdivision. In most
cases the salesman's share is 4%. (Exh. "N-1").2
It will be readily seen therefrom that when the petitioner's commission covers general
supervision, it is provided that the 1/2 rule of equal sharing of the sales commissions does not
apply and that the salesman's share is stipulated in the case of each subdivision. Furthermore,
what is involved here is not Hoskins' salesman's share in the petitioner's 12% sales
commission, which he presumably collected also from petitioner without respondent's
questioning it, but a 50% share besides in petitioner's planning and supervision fee of 8% of the
gross sales, as mentioned above. This is evident from petitioner's board's resolution of July 14,
1953 (Exhibit 7), wherein it is recited that in addition to petitioner's sales commission of 12% of
gross sales, the subdivision owners were paying to petitioner 8% of gross sales as supervision
fee, and a collection fee of 5% of gross collections, or total fees of 25% of gross sales.

62

The case before us is similar to previous cases of disallowances as deductible items of officers'
extra fees, bonuses and commissions, upheld by this Court as not being within the purview of
ordinary and necessary expenses and not passing the test of reasonable
compensation.3 In Kuenzle & Streiff, Inc. vs. Commissioner of Internal Revenue decided by this
Court on May 29, 1969,4 we reaffirmed the test of reasonableness, enunciated in the earlier
1967 case involving the same parties, that: "It is a general rule that 'Bonuses to employees
made in good faith and as additional compensation for the services actually rendered by the
employees are deductible, provided such payments, when added to the stipulated salaries, do
not exceed a reasonable compensation for the services rendered' (4 Mertens Law of Federal
Income Taxation, Sec. 25.50, p. 410). The conditions precedent to the deduction of bonuses to
employees are: (1) the payment of the bonuses is in fact compensation; (2) it must be for
personal services actually rendered; and (3) the bonuses, when added to the salaries, are
'reasonable . . . when measured by the amount and quality of the services performed with
relation to the business of the particular taxpayer' (Idem., Sec. 25, 44, p. 395).
"There is no fixed test for determining the reasonableness of a given bonus as compensation.
This depends upon many factors, one of them being 'the amount and quality of the services
performed with relation to the business.' Other tests suggested are: payment must be 'made in
good faith'; 'the character of the taxpayer's business, the volume and amount of its net earnings,
its locality, the type and extent of the services rendered, the salary policy of the corporation'; 'the
size of the particular business'; 'the employees' qualifications and contributions to the business
venture'; and 'general economic conditions' (4 Mertens, Law of Federal Income Taxation, Secs.
25.44, 25.49, 25.50, 25.51, pp. 407-412). However, 'in determining whether the particular salary
or compensation payment is reasonable, the situation must be considered as whole. Ordinarily,
no single factor is decisive. . . . it is important to keep in mind that it seldom happens that the
application of one test can give satisfactory answer, and that ordinarily it is the interplay of
several factors, properly weighted for the particular case, which must furnish the final answer."
Petitioner's case fails to pass the test. On the right of the employer as against respondent
Commissioner to fix the compensation of its officers and employees, we there held further that
while the employer's right may be conceded, the question of the allowance or disallowance
thereof as deductible expenses for income tax purposes is subject to determination by
respondent Commissioner of Internal Revenue. Thus: "As far as petitioner's contention that as
employer it has the right to fix the compensation of its officers and employees and that it was in
the exercise of such right that it deemed proper to pay the bonuses in question, all that We need
say is this: that right may be conceded, but for income tax purposes the employer cannot legally
claim such bonuses as deductible expenses unless they are shown to be reasonable. To hold
otherwise would open the gate of rampant tax evasion.
"Lastly, We must not lose sight of the fact that the question of allowing or disallowing as
deductible expenses the amounts paid to corporate officers by way of bonus is determined by
respondent exclusively for income tax purposes. Concededly, he has no authority to fix the
amounts to be paid to corporate officers by way of basic salary, bonus or additional
remuneration a matter that lies more or less exclusively within the sound discretion of the
corporation itself. But this right of the corporation is, of course, not absolute. It cannot exercise it
for the purpose of evading payment of taxes legitimately due to the State."
Finally, it should be noted that we have here a case practically of a sole proprietorship of C. M.
Hoskins, who however chose to incorporate his business with himself holding virtually absolute
control thereof with 99.6% of its stock with four other nominal shareholders holding one share
each. Having chosen to use the corporate form with its legal advantages of a separate
corporate personality as distinguished from his individual personality, the corporation so
created, i.e., petitioner, is bound to comport itself in accordance with corporate norms and
63

comply with its corporate obligations. Specifically, it is bound to pay the income tax imposed by
law on corporations and may not legally be permitted, by way of corporate resolutions
authorizing payment of inordinately large commissions and fees to its controlling stockholder, to
dilute and diminish its corresponding corporate tax liability.
ACCORDINGLY, the decision appealed from is hereby affirmed, with costs in both instances
against petitioner.
Concepcion, C.J., Reyes, J.B.L., Dizon, Makalintal, Zaldivar, Sanchez, Castro, Fernando and
Barredo, JJ.,concur.

Republic of the Philippines


SUPREME COURT
Manila
EN BANC
G.R. Nos. 106949-50 December 1, 1995
PAPER INDUSTRIES CORPORATION OF THE PHILIPPINES (PICOP), petitioner,
vs.
COURT OF APPEALS, COMMISSIONER OF INTERNAL REVENUE and COURT OF TAX
APPEALS, respondents.
G.R. Nos. 106984-85 December 1, 1995
COMMISSIONER INTERNAL REVENUE, petitioner,
vs.
PAPER INDUSTRIES CORPORATION OF THE PHILIPPINES, THE COURT OF APPEALS
and THE COURT OF TAX APPEALS, respondents.
FELICIANO, J.:
The Paper Industries Corporation of the Philippines ("Picop"), which is petitioner in G.R. Nos.
106949-50 and private respondent in G.R. Nos. 106984-85, is a Philippine corporation
registered with the Board of Investments ("BOI") as a preferred pioneer enterprise with respect
to its integrated pulp and paper mill, and as a preferred non-pioneer enterprise with respect to
its integrated plywood and veneer mills.

64

On 21 April 1983, Picop received from the Commissioner of Internal Revenue ("CIR") two (2)
letters of assessment and demand both dated 31 March 1983: (a) one for deficiency transaction
tax and for documentary and science stamp tax; and (b) the other for deficiency income tax for
1977, for an aggregate amount ofP88,763,255.00. These assessments were computed as
follows:
Transaction Tax
Interest payments on
money market
borrowings P 45,771,849.00

35% Transaction tax due


thereon 16,020,147.00
Add: 25% surcharge 4,005,036.75

T o t a l P 20,025,183.75
Add:
14% int. fr.
1-20-78 to
7-31-80 P 7,093,302.57
20% int, fr.
8-1-80 to
3-31-83 10,675,523.58

17,768,826.15

P 37,794,009.90
Documentary and Science Stamps Tax
Total face value of
65

debentures P100,000,000.00
Documentary Stamps
Tax Due
(P0.30 x P100,000.000 )
( P200 ) P 150,000.00
Science Stamps Tax Due
(P0.30 x P100,000,000 )
( P200 ) P 150,000.00

T o t a l P 300,000.00
Add: Compromise for
non-affixture 300.00

300,300.00

TOTAL AMOUNT DUE AND COLLECTIBLE P 38,094,309.90


===========
Deficiency Income Tax for 1977
Net income per return P 258,166.00
Add: Unallowable deductions
1) Disallowed deductions
availed of under
R.A. No. 5186 P 44,332,980.00
2) Capitalized interest
expenses on funds
used for acquisition
66

of machinery & other


equipment 42,840,131.00
3) Unexplained financial
guarantee expense 1,237,421.00
4) Understatement
of sales 2,391,644.00
5) Overstatement of
cost of sales 604,018.00

P91,406,194.00
Net income per investigation P91,664,360.00
Income tax due thereon 34,734,559.00
Less: Tax already assessed per return 80,358.00

Deficiency P34,654,201.00
Add:
14% int. fr.
4-15-78 to
7-31-81 P 11,128,503.56
20% int. fr.
8-1-80 to
4-15-81 4,886,242.34

P16,014,745.90

TOTAL AMOUNT DUE AND COLLECTIBLE P 50,668,946.90 1


67

===========
On 26 April 1983, Picop protested the assessment of deficiency transaction tax and
documentary and science stamp taxes. Picop also protested on 21 May 1983 the deficiency
income tax assessment for 1977. These protests were not formally acted upon by respondent
CIR. On 26 September 1984, the CIR issued a warrant of distraint on personal property and a
warrant of levy on real property against Picop, to enforce collection of the contested
assessments; in effect, the CIR denied Picop's protests.
Thereupon, Picop went before the Court of Tax Appeals ("CTA") appealing the assessments.
After trial, the CTA rendered a decision dated 15 August 1989, modifying the findings of the CIR
and holding Picop liable for the reduced aggregate amount of P20,133,762.33, which was
itemized in the dispositive portion of the decision as follows:
35% Transaction Tax P 16,020,113.20
Documentary & Science
Stamp Tax 300,300.00
Deficiency Income Tax Due 3,813,349.33

TOTAL AMOUNT DUE AND PAYABLE P 20,133,762.53 2


===========
Picop and the CIR both went to the Supreme Court on separate Petitions for Review of the
above decision of the CTA. In two (2) Resolutions dated 7 February 1990 and 19 February
1990, respectively, the Court referred the two (2) Petitions to the Court of Appeals. The Court of
Appeals consolidated the two (2) cases and rendered a decision, dated 31 August 1992, which
further reduced the liability of Picop to P6,338,354.70. The dispositive portion of the Court of
Appeals decision reads as follows:
WHEREFORE, the appeal of the Commissioner of Internal Revenue is denied for
lack of merit. The judgment against PICOP is modified, as follows:
1. PICOP is declared liable for the 35% transaction tax in the amount of
P3,578,543.51;
2. PICOP is absolved from the payment of documentary and science stamp tax
of P300,000.00 and the compromise penalty of P300.00;
3. PICOP shall pay 20% interest per annum on the deficiency income tax of
P1,481,579.15, for a period of three (3) years from 21 May 1983, or in the total
amount of P888,947.49, and a surcharge of 10% on the latter amount, or
P88,984.75.
No pronouncement as to costs.
SO ORDERED.
Picop and the CIR once more filed separate Petitions for Review before the Supreme Court.
These cases were consolidated and, on 23 August 1993, the Court resolved to give due course
68

to both Petitions in G.R. Nos. 106949-50 and 106984-85 and required the parties to file their
Memoranda.
Picop now maintains that it is not liable at all to pay any of the assessments or any part thereof.
It assails the propriety of the thirty-five percent (35%) deficiency transaction tax which the Court
of Appeals held due from it in the amount of P3,578,543.51. Picop also questions the imposition
by the Court of Appeals of the deficiency income tax of P1,481,579.15, resulting from
disallowance of certain claimed financial guarantee expenses and claimed year-end
adjustments of sales and cost of sales figures by Picop's external auditors. 3
The CIR, upon the other hand, insists that the Court of Appeals erred in finding Picop not liable
for surcharge and interest on unpaid transaction tax and for documentary and science stamp
taxes and in allowing Picop to claim as deductible expenses:
(a) the net operating losses of another corporation (i.e., Rustan Pulp and Paper
Mills, Inc.); and
(b) interest payments on loans for the purchase of machinery and equipment.
The CIR also claims that Picop should be held liable for interest at fourteen percent
(14%) per annum from 15 April 1978 for three (3) years, and interest at twenty percent
(20%) per annum for a maximum of three (3) years; and for a surcharge of ten percent
(10%), on Picop's deficiency income tax. Finally, the CIR contends that Picop is liable for
the corporate development tax equivalent to five percent (5%) of its correct 1977 net
income.
The issues which we must here address may be sorted out and grouped in the following
manner:
I. Whether Picop is liable for:
(1) the thirty-five percent (35%) transaction tax;
(2) interest and surcharge on unpaid transaction tax; and
(3) documentary and science stamp taxes;
II. Whether Picop is entitled to deductions against income of:
(1) interest payments on loans for the purchase of machinery
and equipment;
(2) net operating losses incurred by the Rustan Pulp and
Paper Mills, Inc.; and
(3) certain claimed financial guarantee expenses; and
III. (1) Whether Picop had understated its sales and overstated its cost of sales
for 1977; and
(2) Whether Picop is liable for the corporate development tax of five percent (5%)
of its net income for 1977.
69

We will consider these issues in the foregoing sequence.I.


(1) Whether Picop is liable
for the thirty-five percent
(35%) transaction tax.
With the authorization of the Securities and Exchange Commission, Picop issued commercial
paper consisting of serially numbered promissory notes with the total face value of
P229,864,000.00 and a maturity period of one (1) year, i.e., from 24 December 1977 to 23
December 1978. These promissory notes were purchased by various commercial banks and
financial institutions. On these promissory notes, Picop paid interest in the aggregate amount of
P45,771,849.00. In respect of these interest payments, the CIR required Picop to pay the thirtyfive percent (35%) transaction tax.
The CIR based this assessment on Presidential Decree No. 1154 dated 3 June 1977, which
reads in part as follows:
Sec. 1. The National Internal Revenue Code, as amended, is hereby further
amended by adding a new section thereto to read as follows:
Sec. 195-C. Tax on certain interest. There shall be levied, assessed, collected
and paid on every commercial paper issued in the primary market as principal
instrument, a transaction tax equivalent to thirty-five percent (35%) based on the
gross amount of interest thereto as defined hereunder, which shall be paid by the
borrower/issuer: Provided, however, that in the case of a long-term commercial
paper whose maturity exceeds more than one year, the borrower shall pay the
tax based on the amount of interest corresponding to one year, and thereafter
shall pay the tax upon accrual or actual payment (whichever is earlier) of the
untaxed portion of the interest which corresponds to a period not exceeding one
year.
The transaction tax imposed in this section shall be a final tax to be paid by the
borrower and shall be allowed as a deductible item for purposes of computing the
borrower's taxable income.
For purposes of this tax
(a) "Commercial paper" shall be defined as an instrument evidencing
indebtedness of any person or entity, including banks and non-banks performing
quasi-banking functions, which is issued, endorsed, sold, transferred or in any
manner conveyed to another person or entity, either with or without recourse and
irrespective of maturity. Principally, commercial papers are promissory
notes and/or similar instruments issued in the primary market and shall not
include repurchase agreements, certificates of assignments, certificates of
participations, and such other debt instruments issued in the secondary market.
(b) The term "interest" shall mean the difference between what the principal
borrower received and the amount it paid upon maturity of the commercial paper
which shall, in no case, be lower than the interest rate prevailing at the time of
the issuance or renewal of the commercial paper. Interest shall be deemed
synonymous with discount and shall include all fees, commissions, premiums
and other payments which form integral parts of the charges imposed as a
consequence of the use of money.
70

In all cases, where no interest rate is stated or if the rate stated is lower than the
prevailing interest rate at the time of the issuance or renewal of commercial
paper, the Commissioner of Internal Revenue, upon consultation with the
Monetary Board of the Central Bank of the Philippines, shall adjust the interest
rate in accordance herewith, and assess the tax on the basis thereof.
The tax herein imposed shall be remitted by the borrower to the Commissioner of
Internal Revenue or his Collection Agent in the municipality where such borrower
has its principal place of business within five (5) working days from the issuance
of the commercial paper. In the case of long term commercial paper, the tax upon
the untaxed portion of the interest which corresponds to a period not exceeding
one year shall be paid upon accrual payment, whichever is earlier. (Emphasis
supplied)
Both the CTA and the Court of Appeals sustained the assessment of transaction tax.
In the instant Petition, Picop reiterates its claim that it is exempt from the payment of the
transaction tax by virtue of its tax exemption under R.A. No. 5186, as amended, known as the
Investment Incentives Act, which in the form it existed in 1977-1978, read in relevant part as
follows:
Sec. 8. Incentives to a Pioneer Enterprise. In addition to the incentives provided
in the preceding section, pioneer enterprises shall be granted the following
incentive benefits:
(a) Tax Exemption. Exemption from all taxes under the National Internal Revenue
Code, except income tax, from the date the area of investment is included in the
Investment Priorities Plan to the following extent:
(1) One hundred per cent (100%) for the first five years;
(2) Seventy-five per cent (75%) for the sixth through the eighth years;
(3) Fifty per cent (50%) for the ninth and tenth years;
(4) Twenty per cent (20%) for the eleventh and twelfth years; and
(5) Ten per cent (10%) for the thirteenth through the fifteenth year.
xxx xxx xxx 4
We agree with the CTA and the Court of Appeals that Picop's tax exemption under R.A. No.
5186, as amended, does not include exemption from the thirty-five percent (35%) transaction
tax. In the first place, the thirty-five percent (35%) transaction tax 5 is an income tax, that is, it is
a tax on the interest income of the lenders or creditors. In Western Minolco
Corporation v. Commissioner of Internal Revenue, 6 the petitioner corporation borrowed funds
from several financial institutions from June 1977 to October 1977 and paid the corresponding
thirty-five (35%) transaction tax thereon in the amount of P1,317,801.03, pursuant to Section
210 (b) of the 1977 Tax Code. Western Minolco applied for refund of that amount alleging it was
exempt from the thirty-five (35%) transaction tax by reason of Section 79-A of C.A. No. 137, as
amended, which granted new mines and old mines resuming operation "five (5) years complete
71

tax exemptions, except income tax, from the time of its actual bonafide orders for equipment for
commercial production." In denying the claim for refund, this Court held:
The petitioner's contentions deserve scant consideration. The 35% transaction
tax is imposed on interest income from commercial papers issued in the primary
money market. Being a tax on interest, it is a tax on income.
As correctly ruled by the respondent Court of Tax Appeals:
Accordingly, we need not and do not think it necessary to discuss
further the nature of the transaction tax more than to say that the
incipient scheme in the issuance of Letter of Instructions No. 340
on November 24, 1975 (O.G. Dec. 15, 1975), i.e., to achieve
operational simplicity and effective administration in capturing the
interest-income "windfall" from money market operations as a new
source of revenue, has lost none of its animating principle in
parturition of amendatory Presidential Decree No. 1154, now
Section 210 (b) of the Tax Code. The tax thus imposed is actually
a tax on interest earnings of the lenders or placers who are
actually the taxpayers in whose income is imposed. Thus "the
borrower withholds the tax of 35% from the interest he would have
to pay the lender so that he (borrower) can pay the 35% of the
interest to the Government." (Citation omitted) . . . . Suffice it to
state that the broad consensus of fiscal and monetary authorities
is that "even if nominally, the borrower is made to pay the tax,
actually, the tax is on the interest earning of the immediate and all
prior lenders/placers of the money. . . ." (Rollo, pp. 36-37)
The 35% transaction tax is an income tax on interest earnings to the lenders or
placers. The latter are actually the taxpayers. Therefore, the tax cannot be a tax
imposed upon the petitioner. In other words, the petitioner who borrowed funds
from several financial institutions by issuing commercial papers merely withheld
the 35% transaction tax before paying to the financial institutions the interests
earned by them and later remitted the same to the respondent Commissioner of
Internal Revenue. The tax could have been collected by a different procedure but
the statute chose this method. Whatever collecting procedure is adopted does
not change the nature of the tax.
xxx xxx xxx 7
(Emphasis supplied)
Much the same issue was passed upon in Marinduque Mining Industrial
Corporation v. Commissioner of Internal Revenue 8 and resolved in the same
way:
It is very obvious that the transaction tax, which is a tax on interest derived from
commercial paper issued in the money market, is not a tax contemplated in the
above-quoted legal provisions. The petitioner admits that it is subject to income
tax. Its tax exemption should be strictly construed.
We hold that petitioner's claim for refund was justifiably denied. The transaction
tax, although nominally categorized as a business tax, is in reality a withholding
72

tax as positively stated in LOI No. 340. The petitioner could have shifted the tax
to the lenders or recipients of the interest. It did not choose to do so. It cannot be
heard now to complain about the tax. LOI No. 340 is an extraneous or extrinsic
aid to the construction of section 210 (b).
xxx xxx xxx 9
(Emphasis supplied)
It is thus clear that the transaction tax is an income tax and as such, in any event, falls outside
the scope of the tax exemption granted to registered pioneer enterprises by Section 8 of R.A.
No. 5186, as amended. Picop was the withholding agent, obliged to withhold thirty-five percent
(35%) of the interest payable to its lenders and to remit the amounts so withheld to the Bureau
of Internal Revenue ("BIR"). As a withholding agent, Picop is made personally liable for the
thirty-five percent (35%) transaction tax 10 and if it did not actually withhold thirty-five percent
(35%) of the interest monies it had paid to its lenders, Picop had only itself to blame.
Picop claims that it had relied on a ruling, dated 6 October 1977, issued by the CIR, which held
that Picop was not liable for the thirty-five (35%) transaction tax in respect of debenture bonds
issued by Picop. Prior to the issuance of the promissory notes involved in the instant case,
Picop had also issued debenture bonds P100,000,000.00 in aggregate face value. The
managing underwriter of this debenture bond issue, Bancom Development Corporation,
requested a formal ruling from the Bureau of Internal Revenue on the liability of Picop for the
thirty-five percent (35%) transaction tax in respect of such bonds. The ruling rendered by the
then Acting Commissioner of Internal Revenue, Efren I. Plana, stated in relevant part:
It is represented that PICOP will be offering to the public primary bonds in the
aggregate principal sum of one hundred million pesos (P100,000,000.00); that
the bonds will be issued as debentures in denominations of one thousand pesos
(P1,000.00) or multiples, to mature in ten (10) years at 14% interest per
annum payable semi-annually; that the bonds are convertible into common stock
of the issuer at the option of the bond holder at an agreed conversion price;
that the issue will be covered by a "Trust Indenture" with a duly authorized trust
corporation as required by the Securities and Exchange Commission, which
trustee will act for and in behalf of the debenture bond holders as beneficiaries;
that once issued, the bonds cannot be preterminated by the holder and cannot
be redeemed by the issuer until after eight (8) years from date of issue; that the
debenture bonds will besubordinated to present and future debts of PICOP; and
that said bonds are intended to be listed in the stock exchanges, which will place
them alongside listed equity issues.
In reply, I have the honor to inform you that although the bonds hereinabove
described are commercial papers which will be issued in the primary market,
however, it is clear from the abovestated facts that said bonds will not be issued
as money market instruments. Such being the case, and considering that the
purposes of Presidential Decree No. 1154, as can be gleaned from Letter of
Instruction No. 340, dated November 21, 1975, are (a) to regulate money market
transactions and (b) to ensure the collection of the tax on interest derived from
money market transactions by imposing a withholding tax thereon, said bonds do
not come within the purview of the"commercial papers" intended to be subjected
to the 35% transaction tax prescribed in Presidential Decree No. 1154, as
implemented by Revenue Regulations No. 7-77. (See Section 2 of said
Regulation) Accordingly, PICOP is not subject to 35% transaction tax on its
73

issues of the aforesaid bonds. However, those investing in said bonds should be
made aware of the fact that the transaction tax is not being imposed on the
issuer of said bonds by printing or stamping thereon, in bold letters, the following
statement: "ISSUER NOT SUBJECT TO TRANSACTION TAX UNDER P.D.
1154. BONDHOLDER SHOULD DECLARE INTEREST EARNING FOR INCOME
TAX." 11 (Emphases supplied)
In the above quoted ruling, the CIR basically held that Picop's debenture bonds did not
constitute "commercial papers" within the meaning of P.D. No. 1154, and that, as such, those
bonds were not subject to the thirty-five percent (35%) transaction tax imposed by P.D. No.
1154.
The above ruling, however, is not applicable in respect of the promissory notes which are the
subject matter of the instant case. It must be noted that the debenture bonds which were the
subject matter of Commissioner Plana's ruling were long-term bonds maturing in ten (10) years
and which could not be pre-terminated and could not be redeemed by Picop until after eight (8)
years from date of issue; the bonds were moreover subordinated to present and future debts of
Picop and convertible into common stock of Picop at the option of the bondholder. In contrast,
the promissory notes involved in the instant case are short-term instruments bearing a one-year
maturity period. These promissory notes constitute the very archtype of money market
instruments. For money market instruments are precisely, by custom and usage of the financial
markets, short-term instruments with a tenor of one (1) year or less. 12 Assuming, therefore,
(without passing upon) the correctness of the 6 October 1977 BIR ruling, Picop's short-term
promissory notes must be distinguished, and treated differently, from Picop's long-term
debenture bonds.
We conclude that Picop was properly held liable for the thirty-five percent (35%) transaction tax
due in respect of interest payments on its money market borrowings.
At the same time, we agree with the Court of Appeals that the transaction tax may be levied only
in respect of the interest earnings of Picop's money market lenders accruing after P.D. No. 1154
went into effect, and not in respect of all the 1977 interest earnings of such lenders. The Court
of Appeals pointed out that:
PICOP, however contends that even if the tax has to be paid, it should be
imposed only for the interests earned after 20 September 1977 when PD 1154
creating the tax became effective. We find merit in this contention. It appears that
the tax was levied on interest earnings from January to October, 1977. However,
as found by the lower court, PD 1154 was published in the Official Gazette only
on 5 September 1977, and became effective only fifteen (15) days after the
publication, or on 20 September 1977, no other effectivity date having been
provided by the PD. Based on the Worksheet prepared by the Commissioner's
office, the interests earned from 20 September to October 1977 was
P10,224,410.03. Thirty-five (35%) per cent of this is P3,578,543.51 which is all
PICOP should pay as transaction tax. 13 (Emphasis supplied)
P.D. No. 1154 is not, in other words, to be given retroactive effect by imposing the thirty-five
percent (35%) transaction tax in respect of interest earnings which accrued before the effectivity
date of P.D. No. 1154, there being nothing in the statute to suggest that the legislative authority
intended to bring about such retroactive imposition of the tax.

74

(2) Whether Picop is liable


for interest and surcharge
on unpaid transaction tax.
With respect to the transaction tax due, the CIR prays that Picop be held liable for a twenty-five
percent (25%) surcharge and for interest at the rate of fourteen percent (14%) per annum from
the date prescribed for its payment. In so praying, the CIR relies upon Section 10 of Revenue
Regulation 7-77 dated 3 June 1977, 14 issued by the Secretary of Finance. This Section reads:
Sec. 10. Penalties. Where the amount shown by the taxpayer to be due on its
return or part of such payment is not paid on or before the date prescribed for its
payment, the amount of the tax shall be increased by twenty-five (25%) per
centum, the increment to be a part of the tax and the entire amount shall be
subject to interest at the rate of fourteen (14%) per centum per annum from the
date prescribed for its payment.
In the case of willful neglect to file the return within the period prescribed herein
or in case a false or fraudulent return is willfully made, there shall be added to the
tax or to the deficiency tax in case any payment has been made on the basis of
such return before the discovery of the falsity or fraud, a surcharge of fifty (50%)
per centum of its amount. The amount so added to any tax shall be collected at
the same time and in the same manner and as part of the tax unless the tax has
been paid before the discovery of the falsity or fraud, in which case the amount
so added shall be collected in the same manner as the tax.
In addition to the above administrative penalties, the criminal and civil
penalties as provided for under Section 337 of the Tax Code of 1977 shall be
imposed for violation of any provision of Presidential Decree No.
1154. 15 (Emphases supplied)
The 1977 Tax Code itself, in Section 326 in relation to Section 4 of the same Code,
invoked by the Secretary of Finance in issuing Revenue Regulation 7-77, set out, in
comprehensive terms, the rule-making authority of the Secretary of Finance:
Sec. 326. Authority of Secretary of Finance to Promulgate Rules and
Regulations. The Secretary of Finance, upon recommendation of the
Commissioner of Internal Revenue, shall promulgate all needful rules and
regulations for the effective enforcement of the provisions of this Code.
(Emphasis supplied)
Section 4 of the same Code contains a list of subjects or areas to be dealt with by the
Secretary of Finance through the medium of an exercise of his quasi-legislative or rulemaking authority. This list, however, while it purports to be open-ended, does not include
the imposition of administrative or civil penalties such as the payment of amounts
additional to the tax due. Thus, in order that it may be held to be legally effective in
respect of Picop in the present case, Section 10 of Revenue Regulation 7-77 must
embody or rest upon some provision in the Tax Code itself which imposes surcharge and
penalty interest for failure to make a transaction tax payment when due.
P.D. No. 1154 did not itself impose, nor did it expressly authorize the imposition of, a surcharge
and penalty interest in case of failure to pay the thirty-five percent (35%) transaction tax when
due. Neither did Section 210 (b) of the 1977 Tax Code which re-enacted Section 195-C inserted
into the Tax Code by P.D. No. 1154.
75

The CIR, both in its petition before the Court of Appeals and its Petition in the instant case,
points to Section 51 (e) of the 1977 Tax Code as its source of authority for assessing a
surcharge and penalty interest in respect of the thirty-five percent (35%) transaction tax due
from Picop. This Section needs to be quoted in extenso:
Sec. 51. Payment and Assessment of Income Tax.
(c) Definition of deficiency. As used in this Chapter in respect of a tax imposed
by this Title, the term "deficiency" means:
(1) The amount by which the tax imposed by this Title exceeds the amount
shown as the tax by the taxpayer upon his return; but the amount so shown on
the return shall first be increased by the amounts previously assessed (or
collected without assessment) as a deficiency, and decreased by the amount
previously abated, credited, returned, or otherwise in respect of such tax; . . .
xxx xxx xxx
(e) Additions to the tax in case of non-payment.
(1) Tax shown on the return. Where the amount determined by the taxpayer
as the tax imposed by this Title or any installment thereof, or any part of such
amount or installment is not paid on or before the date prescribed for its
payment, there shall be collected as a part of the tax, interest upon such unpaid
amount at the rate of fourteen per centum per annum from the date prescribed
for its payment until it is paid: Provided, That the maximum amount that may be
collected as interest on deficiency shall in no case exceed the amount
corresponding to a period of three years, the present provisions regarding
prescription to the contrary notwithstanding.
(2) Deficiency. Where a deficiency, or any interest assessed in connection
therewith under paragraph (d) of this section, or any addition to the taxes
provided for in Section seventy-two of this Code is not paid in full within thirty
days from the date of notice and demand from the Commissioner of Internal
Revenue, there shall be collected upon the unpaid amount as part of the tax,
interest at the rate of fourteen per centum per annum from the date of such
notice and demand until it is paid: Provided, That the maximum amount that may
be collected as interest on deficiency shall in no case exceed the amount
corresponding to a period of three years, the present provisions regarding
prescription to the contrary notwithstanding.
(3) Surcharge. If any amount of tax included in the notice and demand from
the Commissioner of Internal Revenue is not paid in full within thirty days after
such notice and demand, there shall be collected in addition to the interest
prescribed herein and in paragraph (d) above and as part of the tax a surcharge
of five per centum of the amount of tax unpaid. (Emphases supplied)
Section 72 of the 1977 Tax Code referred to in Section 51 (e) (2) above, provides:
Sec. 72. Surcharges for failure to render returns and for rendering false and
fraudulent returns. In case of willful neglect to file the return or list required by
this Title within the time prescribed by law, or in case a false or fraudulent return
or list is wilfully made, the Commissioner of Internal Revenue shall add to the tax
76

or to the deficiency tax, in case any payment has been made on the basis of
such return before the discovery of the falsity or fraud, as surcharge of fifty per
centum of the amount of such tax or deficiency tax. In case of any failure to make
and file a return or list within the time prescribed by law or by the Commissioner
or other Internal Revenue Officer, not due to willful neglect, the Commissioner of
Internal Revenue shall add to the tax twenty-five per centum of its amount,
except that, when a return is voluntarily and without notice from the
Commissioner or other officer filed after such time, and it is shown that the failure
to file it was due to a reasonable cause, no such addition shall be made to the
tax. The amount so added to any tax shall be collected at the same time, in the
same manner and as part of the tax unless the tax has been paid before the
discovery of the neglect, falsity, or fraud, in which case the amount so added
shall be collected in the same manner as the tax. (Emphases supplied)
It will be seen that Section 51 (c) (1) and (e) (1) and (3), of the 1977 Tax Code, authorize the
imposition of surcharge and interest only in respect of a "tax imposed by this Title," that is to
say, Title II on "Income Tax." It will also be seen that Section 72 of the 1977 Tax Code imposes a
surcharge only in case of failure to file a return or list "required by this Title," that is, Title II
on "Income Tax." The thirty-five percent (35%) transaction tax is, however, imposed in the 1977
Tax Code by Section 210 (b) thereof which Section is embraced in Title V on "Taxes on
Business" of that Code. Thus, while the thirty-five percent (35%) transaction tax is in truth a tax
imposed on interest income earned by lenders or creditors purchasing commercial paper on the
money market, the relevant provisions, i.e., Section 210 (b), were not inserted in Title II of the
1977 Tax Code. The end result is that the thirty-five percent (35%) transaction tax is not one of
the taxes in respect of which Section 51 (e) authorized the imposition of surcharge and interest
and Section 72 the imposition of a fraud surcharge.
It is not without reluctance that we reach the above conclusion on the basis of what may well
have been an inadvertent error in legislative draftsmanship, a type of error common enough
during the period of Martial Law in our country. Nevertheless, we are compelled to adopt this
conclusion. We consider that the authority to impose what the present Tax Code calls (in
Section 248) civil penalties consisting of additions to the tax due, must be expressly given in the
enabling statute, in language too clear to be mistaken. The grant of that authority is not lightly to
be assumed to have been made to administrative officials, even to one as highly placed as the
Secretary of Finance.
The state of the present law tends to reinforce our conclusion that Section 51 (c) and (e) of the
1977 Tax Code did not authorize the imposition of a surcharge and penalty interest for failure to
pay the thirty-five percent (35%) transaction tax imposed under Section 210 (b) of the same
Code. The corresponding provision in the current Tax Code very clearly embraces failure to pay
all taxes imposed in the Tax Code, without any regard to the Title of the Code where provisions
imposing particular taxes are textually located. Section 247 (a) of the NIRC, as amended, reads:
Title X
Statutory Offenses and Penalties
Chapter I
Additions to the Tax
Sec. 247. General Provisions. (a) The additions to the tax or deficiency tax
prescribed in this Chapter shall apply to all taxes, fees and charges imposed in
this Code. The amount so added to the tax shall be collected at the same time, in
the same manner and as part of the tax. . . .
77

Sec. 248. Civil Penalties. (a) There shall be imposed, in addition to the tax
required to be paid, penalty equivalent to twenty-five percent (25%) of the
amount due, in the following cases:
xxx xxx xxx
(3) failure to pay the tax within the time prescribed for its payment;
or
xxx xxx xxx
(c) the penalties imposed hereunder shall form part of the tax and the entire
amount shall be subject to the interest prescribed in Section 249.
Sec. 249. Interest. (a) In General. There shall be assessed and
collected on any unpaid amount of tax, interest at the rate of twenty
percent (20%) per annum or such higher rate as may be prescribed by
regulations, from the date prescribed for payment until the amount is fully paid. . .
(Emphases supplied)
In other words, Section 247 (a) of the current NIRC supplies what did not exist back in
1977 when Picop's liability for the thirty-five percent (35%) transaction tax became fixed.
We do not believe we can fill that legislative lacuna by judicial fiat. There is nothing to
suggest that Section 247 (a) of the present Tax Code, which was inserted in 1985, was
intended to be given retroactive application by the legislative authority. 16
(3) Whether Picop is Liable
for Documentary and
Science Stamp Taxes.
As noted earlier, Picop issued sometime in 1977 long-term subordinated convertible debenture
bonds with an aggregate face value of P100,000,000.00. Picop stated, and this was not
disputed by the CIR, that the proceeds of the debenture bonds were in fact utilized to finance
the BOI-registered operations of Picop. The CIR assessed documentary and science stamp
taxes, amounting to P300,000.00, on the issuance of Picop's debenture bonds. It is claimed by
Picop that its tax exemption "exemption from all taxes under the National Internal Revenue
Code, except income tax" on a declining basis over a certain period of time includes
exemption from the documentary and science stamp taxes imposed under the NIRC.
The CIR, upon the other hand, stresses that the tax exemption under the Investment Incentives
Act may be granted or recognized only to the extent that the claimant Picop was engaged in
registered operations, i.e., operations forming part of its integrated pulp and paper
project. 17 The borrowing of funds from the public, in the submission of the CIR, was not an
activity included in Picop's registered operations. The CTA adopted the view of the CIR and held
that "the issuance of convertible debenture bonds [was] not synonymous [with] the
manufactur[ing] operations of an integrated pulp and paper mill." 18
The Court of Appeals took a less rigid view of the ambit of the tax exemption granted to
registered pioneer enterprises. Said the Court of Appeals:
. . . PICOP's explanation that the debenture bonds were issued to finance its
registered operation is logical and is unrebutted. We are aware that tax
78

exemptions must be applied strictly against the beneficiary in order to deter their
abuse. It would indeed be altogether a different matter if there is a showing that
the issuance of the debenture bonds had no bearing whatsoever on the
registered operations PICOP and that they were issued in connection with a
totally different business undertaking of PICOP other than its registered
operation. There is, however, a dearth of evidence in this regard. It cannot be
denied that PICOP needed funds for its operations. One of the means it used to
raise said funds was to issue debenture bonds. Since the money raised thereby
was to be used in its registered operation, PICOP should enjoy the incentives
granted to it by R.A. 5186, one of which is the exemption from payment of all
taxes under the National Internal Revenue Code, except income taxes, otherwise
the purpose of the incentives would be defeated. Documentary and science
stamp taxes on debenture bonds are certainly not income taxes. 19 (Emphasis
supplied)
Tax exemptions are, to be sure, to be "strictly construed," that is, they are not to be extended
beyond the ordinary and reasonable intendment of the language actually used by the legislative
authority in granting the exemption. The issuance of debenture bonds is certainly conceptually
distinct from pulping and paper manufacturing operations. But no one contends that issuance of
bonds was a principal or regular business activity of Picop; only banks or other financial
institutions are in the regular business of raising money by issuing bonds or other instruments to
the general public. We consider that the actual dedication of the proceeds of the bonds to the
carrying out of Picop's registered operations constituted a sufficient nexus with such registered
operations so as to exempt Picop from stamp taxes ordinarily imposed upon or in connection
with issuance of such bonds. We agree, therefore, with the Court of Appeals on this matter that
the CTA and the CIR had erred in rejecting Picop's claim for exemption from stamp taxes.
It remains only to note that after commencement of the present litigation before the CTA, the
BIR took the position that the tax exemption granted by R.A. No. 5186, as
amended, does include exemption from documentary stamp taxes on transactions entered into
by BOI-registered enterprises. BIR Ruling No. 088, dated 28 April 1989, for instance, held that a
registered preferred pioneer enterprise engaged in the manufacture of integrated circuits,
magnetic heads, printed circuit boards, etc., is exempt from the payment of documentary stamp
taxes. The Commissioner said:
You now request a ruling that as a preferred pioneer enterprise, you are exempt
from the payment of Documentary Stamp Tax (DST).
In reply, please be informed that your request is hereby granted. Pursuant to
Section 46 (a) of Presidential Decree No. 1789, pioneer enterprises registered
with the BOI are exempt from all taxes under the National Internal Revenue
Code, except from all taxes under the National Internal Revenue Code, except
income tax, from the date the area of investment is included in the Investment
Priorities Plan to the following extent:
xxx xxx xxx
Accordingly, your company is exempt from the payment of documentary stamp
tax to the extent of the percentage aforestated on transactions connected with
the registered business activity. (BIR Ruling No. 111-81) However, if said
transactions conducted by you require the execution of a taxable document with
other parties, said parties who are not exempt shall be the one directly liable for
the tax. (Sec. 173, Tax Code, as amended; BIR Ruling No. 236-87) In other
79

words, said parties shall be liable to the same percentage corresponding to your
tax exemption. (Emphasis supplied)
Similarly, in BIR Ruling No. 013, dated 6 February 1989, the Commissioner held that a
registered pioneer enterprise producing polyester filament yarn was entitled to
exemption "from the documentary stamp tax on [its] sale of real property in Makati up to
December 31, 1989." It appears clear to the Court that the CIR, administratively at least,
no longer insists on the position it originally took in the instant case before the CTA.
II
(1) Whether Picop is entitled
to deduct against current
income interest payments
on loans for the purchase
of machinery and equipment.
In 1969, 1972 and 1977, Picop obtained loans from foreign creditors in order to finance the
purchase of machinery and equipment needed for its operations. In its 1977 Income Tax Return,
Picop claimed interest payments made in 1977, amounting to P42,840,131.00, on these loans
as a deduction from its 1977 gross income.
The CIR disallowed this deduction upon the ground that, because the loans had been incurred
for the purchase of machinery and equipment, the interest payments on those loans should
have been capitalized instead and claimed as a depreciation deduction taking into account the
adjusted basis of the machinery and equipment (original acquisition cost plus interest charges)
over the useful life of such assets.
Both the CTA and the Court of Appeals sustained the position of Picop and held that the interest
deduction claimed by Picop was proper and allowable. In the instant Petition, the CIR insists on
its original position.
We begin by noting that interest payments on loans incurred by a taxpayer (whether BOIregistered or not) are allowed by the NIRC as deductions against the taxpayer's gross income.
Section 30 of the 1977 Tax Code provided as follows:
Sec. 30. Deduction from Gross Income. The following may be deducted from
gross income:
(a) Expenses:
xxx xxx xxx
(b) Interest:
(1) In general. The amount of interest paid within the taxable
year on indebtedness, except on indebtedness incurred or
continued to purchase or carry obligations the interest upon which
is exempt from taxation as income under this Title: . . . (Emphasis
supplied)

80

Thus, the general rule is that interest expenses are deductible against gross income and
this certainly includes interest paid under loans incurred in connection with the carrying
on of the business of the taxpayer. 20 In the instant case, the CIR does not dispute that
the interest payments were made by Picop on loansincurred in connection with the
carrying on of the registered operations of Picop, i.e., the financing of the purchase of
machinery and equipment actually used in the registered operations of Picop. Neither
does the CIR deny that such interest payments were legally due and demandable under
the terms of such loans, and in fact paid by Picop during the tax year 1977.
The CIR has been unable to point to any provision of the 1977 Tax Code or any other Statute
that requires the disallowance of the interest payments made by Picop. The CIR invokes
Section 79 of Revenue Regulations No. 2 as amended which reads as follows:
Sec. 79. Interest on Capital. Interest calculated for cost-keeping or other
purposes on account of capital or surplus invested in the business, which does
not represent a charge arising under an interest-bearing obligation,
is not allowable deduction from gross income. (Emphases supplied)
We read the above provision of Revenue Regulations No. 2 as referring to so called
"theoretical interest," that is to say, interest "calculated" or computed (and
not incurred or paid) for the purpose of determining the "opportunity cost" of investing
funds in a given business. Such "theoretical" or imputed interest does notarise from a
legally demandable interest-bearing obligation incurred by the taxpayer who however
wishes to find out, e.g., whether he would have been better off by lending out his funds
and earning interest rather than investing such funds in his business. One thing that
Section 79 quoted above makes clear is that interest which does constitute a charge
arising under an interest-bearing obligation is an allowable deduction from gross income.
It is claimed by the CIR that Section 79 of Revenue Regulations No. 2 was "patterned after"
paragraph 1.266-1 (b), entitled "Taxes and Carrying Charges Chargeable to Capital Account and
Treated as Capital Items" of the U.S. Income Tax Regulations, which paragraph reads as
follows:
(B) Taxes and Carrying Charges. The items thus chargeable to capital
accounts are
(11) In the case of real property, whether improved or unimproved and whether
productive or nonproductive.
(a) Interest on a loan (but not theoretical interest of a taxpayer using his own
funds). 21
The truncated excerpt of the U.S. Income Tax Regulations quoted by the CIR needs to be
related to the relevant provisions of the U.S. Internal Revenue Code, which provisions deal with
the general topic of adjusted basis for determining allowable gain or loss on sales or exchanges
of property and allowable depreciation and depletion of capital assets of the taxpayer:
Present Rule. The Internal Revenue Code, and the Regulations promulgated
thereunder provide that "No deduction shall be allowed for amounts paid or
accrued for such taxes and carrying charges as, under regulations prescribed by
the Secretary or his delegate, are chargeable to capital account with respect to
property, if the taxpayer elects, in accordance with such regulations, to treat
such taxes orcharges as so chargeable."
81

At the same time, under the adjustment of basis provisions which have just been
discussed, it is provided that adjustment shall be made for all "expenditures,
receipts, losses, or other items" properly chargeable to a capital account, thus
including taxes and carrying charges; however, an exception exists, in which
event such adjustment to the capital account is not made, with respect to taxes
and carrying charges which the taxpayer has not elected to capitalize but for
which a deduction instead has been taken. 22 (Emphasis supplied)
The "carrying charges" which may be capitalized under the above quoted provisions of
the U.S. Internal Revenue Code include, as the CIR has pointed out, interest on a loan
"(but not theoretical interest of a taxpayer using his own funds)." What the CIR failed to
point out is that such "carrying charges" may, at the election of the taxpayer, either be
(a) capitalized in which case the cost basis of the capital assets, e.g., machinery and
equipment, will be adjusted by adding the amount of such interest
payments or alternatively, be (b) deducted from gross income of the taxpayer. Should
the taxpayer elect to deduct the interest payments against its gross income, the taxpayer
cannot at the same time capitalize the interest payments. In other words, the taxpayer
is not entitled to both the deduction from gross income and the adjusted (increased)
basis for determining gain or loss and the allowable depreciation charge. The U.S.
Internal Revenue Code does not prohibit the deduction of interest on a loan obtained for
purchasing machinery and equipment against gross income, unless the taxpayer
has also or previously capitalized the same interest payments and thereby adjusted the
cost basis of such assets.
We have already noted that our 1977 NIRC does not prohibit the deduction of interest on a loan
incurred for acquiring machinery and equipment. Neither does our 1977 NIRC compel the
capitalization of interest payments on such a loan. The 1977 Tax Code is simply silent on a
taxpayer's right to elect one or the other tax treatment of such interest payments. Accordingly,
the general rule that interest payments on a legally demandable loan are deductible from gross
income must be applied.
The CIR argues finally that to allow Picop to deduct its interest payments against its gross
income would be to encourage fraudulent claims to double deductions from gross income:
[t]o allow a deduction of incidental expense/cost incurred in the purchase of fixed
asset in the year it was incurred would invite tax evasion through fraudulent
application of double deductions from gross income. 23 (Emphases supplied)
The Court is not persuaded. So far as the records of the instant cases show, Picop has
not claimed to be entitled to double deduction of its 1977 interest payments. The CIR
has neither alleged nor proved that Picop had previously adjusted its cost basis for the
machinery and equipment purchased with the loan proceeds by capitalizing the interest
payments here involved. The Court will not assume that the CIR would be unable or
unwilling to disallow "a double deduction" should Picop, having deducted its interest cost
from its gross income, also attempt subsequently to adjust upward the cost basis of the
machinery and equipment purchased and claim, e.g., increased deductions for
depreciation.
We conclude that the CTA and the Court of Appeals did not err in allowing the deductions of
Picop's 1977 interest payments on its loans for capital equipment against its gross income for
1977.

82

(2) Whether Picop is entitled


to deduct against current
income net operating losses
incurred by Rustan Pulp
and Paper Mills, Inc.
On 18 January 1977, Picop entered into a merger agreement with the Rustan Pulp and Paper
Mills, Inc. ("RPPM") and Rustan Manufacturing Corporation ("RMC"). Under this agreement, the
rights, properties, privileges, powers and franchises of RPPM and RMC were to be transferred,
assigned and conveyed to Picop as the surviving corporation. The entire subscribed and
outstanding capital stock of RPPM and RMC would be exchanged for 2,891,476 fully paid up
Class "A" common stock of Picop (with a par value of P10.00) and 149,848 shares of preferred
stock of Picop (with a par value of P10.00), to be issued by Picop, the result being that Picop
would wholly own both RPPM and RMC while the stockholders of RPPM and RMC would join
the ranks of Picop's shareholders. In addition, Picop paid off the obligations of RPPM to the
Development Bank of the Philippines ("DBP") in the amount of P68,240,340.00, by issuing
6,824,034 shares of preferred stock (with a par value of P10.00) to the DBP. The merger
agreement was approved in 1977 by the creditors and stockholders of Picop, RPPM and RMC
and by the Securities and Exchange Commission. Thereupon, on 30 November 1977,
apparently the effective date of merger, RPPM and RMC were dissolved. The Board of
Investments approved the merger agreement on 12 January 1978.
It appears that RPPM and RMC were, like Picop, BOI-registered companies. Immediately
before merger effective date, RPPM had over preceding years accumulated losses in the total
amount of P81,159,904.00. In its 1977 Income Tax Return, Picop claimed P44,196,106.00 of
RPPM's accumulated losses as a deduction against Picop's 1977 gross income. 24
Upon the other hand, even before the effective date of merger, on 30 August 1977, Picop sold
all the outstanding shares of RMC stock to San Miguel Corporation for the sum of
P38,900,000.00, and reported a gain of P9,294,849.00 from this transaction. 25
In claiming such deduction, Picop relies on section 7 (c) of R.A. No. 5186 which provides as
follows:
Sec. 7. Incentives to Registered Enterprise. A registered enterprise, to the
extent engaged in a preferred area of investment, shall be granted the following
incentive benefits:
xxx xxx xxx
(c) Net Operating Loss Carry-over. A net operating loss incurred in any of the
first ten years of operations may be carried over as a deduction from taxable
income for the six years immediately following the year of such loss. The entire
amount of the loss shall be carried over to the first of the six taxable years
following the loss, and any portion of such loss which exceeds the taxable
income of such first year shall be deducted in like manner from the taxable
income of the next remaining five years. The net operating loss shall be
computed in accordance with the provisions of the National Internal Revenue
Code, any provision of this Act to the contrary notwithstanding, except that
income not taxable either in whole or in part under this or other laws shall be
included in gross income. (Emphasis supplied)

83

Picop had secured a letter-opinion from the BOI dated 21 February 1977 that is, after the
date of the agreement of merger but before the merger became effective relating to the
deductibility of the previous losses of RPPM under Section 7 (c) of R.A. No. 5186 as amended.
The pertinent portions of this BOI opinion, signed by BOI Governor Cesar Lanuza, read as
follows:
2) PICOP will not be allowed to carry over the losses of Rustan prior to the legal
dissolution of the latter because at that time the two (2) companies still had
separate legal personalities;
3) After BOI approval of the merger, PICOP can no longer apply for the
registration of the registered capacity of Rustan because with the approved
merger, such registered capacity of Rustan transferred to PICOP will have the
same registration date as that of Rustan. In this case, the previous losses of
Rustan may be carried over by PICOP, because with the merger, PICOP
assumes all the rights and obligations of Rustan subject, however, to the period
prescribed for carrying over of such losses. 26 (Emphasis supplied)
Curiously enough, Picop did not also seek a ruling on this matter, clearly a matter of tax law,
from the Bureau of Internal Revenue. Picop chose to rely solely on the BOI letter-opinion.
The CIR disallowed all the deductions claimed on the basis of RPPM's losses, apparently on
two (2) grounds. Firstly, the previous losses were incurred by "another taxpayer," RPPM, and
not by Picop in connection with Picop's own registered operations. The CIR took the view that
Picop, RPPM and RMC were merged into one (1) corporate personality only on 12 January
1978, upon approval of the merger agreement by the BOI. Thus, during the taxable year 1977,
Picop on the one hand and RPPM and RMC on the other, still had their separate juridical
personalities. Secondly, the CIR alleged that these losses had been incurred by RPPM "from
the borrowing of funds" and not from carrying out of RPPM's registered operations. We focus on
the first ground. 27
The CTA upheld the deduction claimed by Picop; its reasoning, however, is less than crystal
clear, especially in respect of its view of what the U.S. tax law was on this matter. In any event,
the CTA apparently fell back on the BOI opinion of 21 February 1977 referred to above. The
CTA said:
Respondent further averred that the incentives granted under Section 7 of R.A.
No. 5186 shall be available only to the extent in which they are engaged in
registered operations, citing Section 1 of Rule IX of the Basic Rules and
Regulations to Implement the Intent and Provisions of the Investment Incentives
Act, R.A. No. 5186.
We disagree with respondent. The purpose of the merger was to rationalize the
container board industry and not to take advantage of the net losses incurred by
RPPMI prior to the stock swap. Thus, when stock of a corporation is purchased
in order to take advantage of the corporation's net operating loss incurred in
years prior to the purchase, the corporation thereafter entering into a trade or
business different from that in which it was previously engaged, the net operating
loss carry-over may be entirely lost. [IRC (1954), Sec. 382(a), Vol. 5, Mertens,
Law of Federal Income Taxation, Chap. 29.11a, p. 103]. 28 Furthermore, once the
BOI approved the merger agreement, the registered capacity of Rustan shall be
transferred to PICOP, and the previous losses of Rustan may be carried over by
PICOP by operation of law. [BOI ruling dated February 21, 1977 (Exh. J-1)] It is
84

clear therefrom, that the deduction availed of under Section 7(c) of R.A. No. 5186
was only proper." (pp. 38-43,Rollo of SP No. 20070) 29 (Emphasis supplied)
In respect of the above underscored portion of the CTA decision, we must note that the CTA in
fact overlooked the statement made by petitioner's counsel before the CTA that:
Among the attractions of the merger to Picop was the accumulated net operating
loss carry-over of RMC that it might possibly use to relieve it (Picop) from its
income taxes, under Section 7 (c) of R.A.5186. Said section provides:
xxx xxx xxx
With this benefit in mind, Picop addressed three (3) questions to the BOI in a
letter dated November 25, 1976. The BOI replied on February 21, 1977 directly
answering the three (3) queries. 30 (Emphasis supplied)
The size of RPPM's accumulated losses as of the date of the merger more than
P81,000,000.00 must have constituted a powerful attraction indeed for Picop.
The Court of Appeals followed the result reached by the CTA. The Court of Appeals, much like
the CTA, concluded that since RPPM was dissolved on 30 November 1977, its accumulated
losses were appropriately carried over by Picop in the latter's 1977 Income Tax Return "because
by that time RPPMI and Picop were no longer separate and different taxpayers." 31
After prolonged consideration and analysis of this matter, the Court is unable to agree with the
CTA and Court of Appeals on the deductibility of RPPM's accumulated losses against Picop's
1977 gross income.
It is important to note at the outset that in our jurisdiction, the ordinary rule that is, the rule
applicable in respect of corporations not registered with the BOI as a preferred pioneer
enterprise is that net operating losses cannot be carried over. Under our Tax Code, both in
1977 and at present, losses may be deducted from gross income only if such losses were
actually sustained in the same year that they are deducted or charged off. Section 30 of the
1977 Tax Code provides:
Sec. 30. Deductions from Gross Income. In computing net income, there shall
be allowed as deduction
xxx xxx xxx
(d) Losses:
(1) By Individuals. In the case of an individual, losses actually sustained
during the taxable year and not compensated for by an insurance or otherwise
(A) If incurred in trade or business;
xxx xxx xxx
(2) By Corporations. In a case of a corporation, all losses actually sustained
and charged off within the taxable year and not compensated for by insurance or
otherwise.
85

(3) By Non-resident Aliens or Foreign Corporations. In the case of a nonresident alien individual or a foreign corporation, the losses deductible are
those actually sustained during the year incurred in business or trade
conducted within the Philippines, . . . 32 (Emphasis supplied)
Section 76 of the Philippine Income Tax Regulations (Revenue Regulation No. 2, as
amended) is even more explicit and detailed:
Sec. 76. When charges are deductible. Each year's return, so far as
practicable, both as to gross income and deductions therefrom should be
complete in itself, and taxpayers are expected to make every reasonable effort to
ascertain the facts necessary to make a correct return. The expenses,
liabilities, or deficit of one year cannot be used to reduce the income of a
subsequent year. A taxpayer has the right to deduct all authorized allowances
and it follows that if he does not within any year deduct certain of his
expenses, losses,
interests,
taxes,
or
other
charges,
he can not deduct them from the income of the next or any succeeding year. . . .
xxx xxx xxx
. . . . If subsequent to its occurrence, however, a taxpayer first ascertains the
amount of a loss sustained during a prior taxable year which has not been
deducted from gross income, he may render an amended return for such
preceding taxable year including such amount of loss in the deduction from gross
income and may in proper cases file a claim for refund of the excess paid by
reason of the failure to deduct such loss in the original return. A loss from theft or
embezzlement occurring in one year and discovered in another is ordinarily
deductible for the year in which sustained. (Emphases supplied)
It is thus clear that under our law, and outside the special realm of BOI-registered
enterprises, there is no such thing as a carry-over of net operating loss. To the
contrary, losses must be deducted against current income in the taxable year when such
losses were incurred. Moreover, such losses may be charged offonly against income
earned in the same taxable year when the losses were incurred.
Thus it is that R.A. No. 5186 introduced the carry-over of net operating losses as a very special
incentive to be granted only to registered pioneer enterprises and only with respect to their
registered operations. The statutory purpose here may be seen to be the encouragement of the
establishment and continued operation of pioneer industries by allowing the registered
enterprise to accumulate its operating losses which may be expected during the early years of
the enterprise and to permit the enterprise to offset such losses against income earned by it in
later years after successful establishment and regular operations. To promote its economic
development goals, the Republic foregoes or defers taxing the income of the pioneer enterprise
until after that enterprise has recovered or offset its earlier losses. We consider that the statutory
purpose can be served only if the accumulated operating losses are carried over and charged
off against income subsequently earned and accumulated by the same enterprise engaged in
the same registered operations.
In the instant case, to allow the deduction claimed by Picop would be to permit one corporation
or enterprise, Picop, to benefit from the operating losses accumulated by another corporation or
enterprise, RPPM. RPPM far from benefiting from the tax incentive granted by the BOI statute,
in fact gave up the struggle and went out of existence and its former stockholders joined the
much larger group of Picop's stockholders. To grant Picop's claimed deduction would be to
86

permit Picop to shelter its otherwise taxable income (an objective which Picop had from the very
beginning) which had not been earned by the registered enterprise which had suffered the
accumulated losses. In effect, to grant Picop's claimed deduction would be to permit Picop to
purchase a tax deduction and RPPM to peddle its accumulated operating losses. Under the
CTA and Court of Appeals decisions, Picop would benefit by immunizing P44,196,106.00 of its
income from taxation thereof although Picop had not run the risks and incurred the losses which
had been encountered and suffered by RPPM. Conversely, the income that would be shielded
from taxation is not income that was, after much effort, eventually generated by the same
registered operations which earlier had sustained losses. We consider and so hold that there is
nothing in Section 7 (c) of R.A. No. 5186 which either requires or permits such a result. Indeed,
that result makes non-sense of the legislative purpose which may be seen clearly to be
projected by Section 7 (c), R.A. No. 5186.
The CTA and the Court of Appeals allowed the offsetting of RPPM's accumulated operating
losses against Picop's 1977 gross income, basically because towards the end of the taxable
year 1977, upon the arrival of the effective date of merger, only one (1) corporation, Picop,
remained. The losses suffered by RPPM's registered operations and the gross income
generated by Picop's own registered operations now came under one and the same corporate
roof. We consider that this circumstance relates much more to form than to substance. We do
not believe that that single purely technical factor is enough to authorize and justify the
deduction claimed by Picop. Picop's claim for deduction is not only bereft of statutory basis; it
does violence to the legislative intent which animates the tax incentive granted by Section 7 (c)
of R.A. No. 5186. In granting the extraordinary privilege and incentive of a net operating loss
carry-over to BOI-registered pioneer enterprises, the legislature could not have intended to
require the Republic to forego tax revenues in order to benefit a corporation which had run no
risks and suffered no losses, but had merely purchased another's losses.
Both the CTA and the Court of Appeals appeared much impressed not only with corporate
technicalities but also with the U.S. tax law on this matter. It should suffice, however, simply to
note that in U.S. tax law, the availability to companies generally of operating loss carry-overs
and of operating loss carry-backs is expressly provided and regulated in great detail by
statute. 33 In our jurisdiction, save for Section 7 (c) of R.A. No. 5186, no statute recognizes or
permits loss carry-overs and loss carry-backs. Indeed, as already noted, our tax law expressly
rejects the very notion of loss carry-overs and carry-backs.
We conclude that the deduction claimed by Picop in the amount of P44,196,106.00 in its 1977
Income Tax Return must be disallowed.
(3) Whether Picop is entitled
to deduct against current
income certain claimed
financial guarantee expenses.
In its Income Tax Return for 1977, Picop also claimed a deduction in the amount of
P1,237,421.00 as financial guarantee expenses.
This deduction is said to relate to chattel and real estate mortgages required from Picop by the
Philippine National Bank ("PNB") and DBP as guarantors of loans incurred by Picop from
foreign creditors. According to Picop, the claimed deduction represents registration fees and
other expenses incidental to registration of mortgages in favor of DBP and PNB.
In support of this claimed deduction, Picop allegedly showed its own vouchers to BIR Examiners
to prove disbursements to the Register of Deeds of Tandag, Surigao del Sur, of particular
87

amounts. In the proceedings before the CTA, however, Picop did not submit in evidence such
vouchers and instead presented one of its employees to testify that the amount claimed had
been disbursed for the registration of chattel and real estate mortgages.
The CIR disallowed this claimed deduction upon the ground of insufficiency of evidence. This
disallowance was sustained by the CTA and the Court of Appeals. The CTA said:
No records are available to support the abovementioned expenses. The
vouchers merely showed that the amounts were paid to the Register of Deeds
and simply cash account. Without the supporting papers such as the invoices or
official receipts of the Register of Deeds, these vouchers standing alone cannot
prove that the payments made were for the accrued expenses in question.The
best evidence of payment is the official receipts issued by the Register of Deeds.
The testimony of petitioner's witness that the official receipts and cash vouchers
were shown to the Bureau of Internal Revenue will not suffice if no records could
be presented in court for proper marking and identification. 34 Emphasis supplied)
The Court of Appeals added:
The mere testimony of a witness for PICOP and the cash vouchers do not suffice
to establish its claim that registration fees were paid to the Register of Deeds for
the registration of real estate and chattel mortgages in favor of Development
Bank of the Philippines and the Philippine National Bank as guarantors of
PICOP's loans. The witness could very well have been merely repeating what he
was instructed to say regardless of the truth, while the cash vouchers, which we
do not find on file, are not said to provide the necessary details regarding the
nature and purpose of the expenses reflected therein. PICOP should have
presented, through the guarantors, its owner's copy of the registered titles with
the lien inscribed thereon as well as an official receipt from the Register of Deeds
evidencing payment of the registration fee. 35 (Emphasis supplied)
We must support the CTA and the Court of Appeals in their foregoing rulings. A taxpayer has the
burden of proving entitlement to a claimed deduction. 36 In the instant case, even Picop's own
vouchers were not submitted in evidence and the BIR Examiners denied that such vouchers
and other documents had been exhibited to them. Moreover, cash vouchers can only confirm
the fact of disbursement but not necessarily the purpose thereof. 37 The best evidence that
Picop should have presented to support its claimed deduction were the invoices and official
receipts issued by the Register of Deeds. Picop not only failed to present such documents; it
also failed to explain the loss thereof, assuming they had existed before. 38 Under the best
evidence rule, 39 therefore, the testimony of Picop's employee was inadmissible and was in any
case entitled to very little, if any, credence.
We consider that entitlement to Picop's claimed deduction of P1,237,421.00 was not adequately
shown and that such deduction must be disallowed.
III
(1) Whether Picop had understated
its sales and overstated its
cost of sales for 1977.
In its assessment for deficiency income tax for 1977, the CIR claimed that Picop had
understated its sales by P2,391,644.00 and, upon the other hand, overstated its cost of sales by
88

P604,018.00. Thereupon, the CIR added back both sums to Picop's net income figure per its
own return.
The 1977 Income Tax Return of Picop set forth the following figures:
Sales (per Picop's Income Tax Return):
Paper P 537,656,719.00
Timber P 263,158,132.00

Total Sales P 800,814,851.00


============
Upon the other hand, Picop's Books of Accounts reflected higher sales figures:
Sales (per Picop's Books of Accounts):
Paper P 537,656,719.00
Timber P 265,549,776.00

Total Sales P 803,206,495.00


============
The above figures thus show a discrepancy between the sales figures reflected in Picop's Books
of Accounts and the sales figures reported in its 1977 Income Tax Return, amounting to:
P2,391,644.00.
The CIR also contended that Picop's cost of sales set out in its 1977 Income Tax Return, when
compared with the cost figures in its Books of Accounts, was overstated:
Cost of Sales
(per Income Tax Return) P607,246,084.00
Cost of Sales
(per Books of Accounts) P606,642,066.00

Discrepancy P 604,018.00
============
Picop did not deny the existence of the above noted discrepancies. In the proceedings before
the CTA, Picop presented one of its officials to explain the foregoing discrepancies. That
explanation is perhaps best presented in Picop's own words as set forth in its Memorandum
before this Court:
. . . that the adjustment discussed in the testimony of the witness, represent the
best and most objective method of determining in pesos the amount of the
correct and actual export sales during the year. It was this correct and actual
export sales and costs of sales that were reflected in the income tax return and in
the audited financial statements. These corrections did not result in realization of
income and should not give rise to any deficiency tax.
xxx xxx xxx
What are the facts of this case on this matter? Why were adjustments necessary
at the year-end?
89

Because of PICOP's procedure of recording its export sales (reckoned in U.S.


dollars) on the basis of a fixed rate, day to day and month to month, regardless of
the actual exchange rate and without waiting when the actual proceeds are
received. In other words, PICOP recorded its export sales at a pre-determined
fixed exchange rate. That pre-determined rate was decided upon at the
beginning of the year and continued to be used throughout the year.
At the end of the year, the external auditors made an examination. In that
examination, the auditors determined with accuracy the actual dollar proceeds of
the export sales received. What exchange rate was used by the auditors to
convert these actual dollar proceeds into Philippine pesos? They used the
average of the differences between (a) the recorded fixed exchange rate and (b)
the exchange rate at the time the proceeds were actually received. It was this
rate at time of receipt of the proceeds that determined the amount of pesos
credited by the Central Bank (through the agent banks) in favor of PICOP. These
accumulated differences were averaged by the external auditors and this was
what was used at the year-end for income tax and other government-report
purposes. (T.s.n., Oct. 17/85, pp. 20-25) 40
The above explanation, unfortunately, at least to the mind of the Court, raises more questions
than it resolves. Firstly, the explanation assumes that all of Picop's sales were export sales for
which U.S. dollars (or other foreign exchange) were received. It also assumes that the expenses
summed up as "cost of sales" were all dollar expenses and that no peso expenses had been
incurred. Picop's explanation further assumes that a substantial part of Picop's dollar proceeds
for its export sales were not actually surrendered to the domestic banking system and
seasonably converted into pesos; had all such dollar proceeds been converted into pesos, then
the peso figures could have been simply added up to reflect the actual peso value of Picop's
export sales. Picop offered no evidence in respect of these assumptions, no explanation why
and how a "pre-determined fixed exchange rate" was chosen at the beginning of the year and
maintained throughout. Perhaps more importantly, Picop was unable to explain why its Books of
Accounts did not pick up the same adjustments that Picop's External Auditors were alleged to
have made for purposes of Picop's Income Tax Return. Picop attempted to explain away the
failure of its Books of Accounts to reflect the same adjustments (no correcting entries,
apparently) simply by quoting a passage from a case where this Court refused to ascribe much
probative value to the Books of Accounts of a corporate taxpayer in a tax case. 41 What appears
to have eluded Picop, however, is that its Books of Accounts, which are kept by its own
employees and are prepared under its control and supervision, reflect what may be deemed to
be admissions against interest in the instant case. For Picop's Books of Accounts precisely
show higher sales figures andlower cost of sales figures than Picop's Income Tax Return.
It is insisted by Picop that its Auditors' adjustments simply present the "best and most objective"
method of reflecting in pesos the "correct and ACTUAL export sales" 42 and that the adjustments
or "corrections" "did not result in realization of [additional] income and should not give rise to
any deficiency tax." The correctness of this contention is not self-evident. So far as the record of
this case shows, Picop did not submit in evidence the aggregate amount of its U.S. dollar
proceeds of its export sales; neither did it show the Philippine pesos it had actually received or
been credited for such U.S. dollar proceeds. It is clear to this Court that the testimonial evidence
submitted by Picop fell far short of demonstrating the correctness of its explanation.
Upon the other hand, the CIR has made out at least a prima facie case that Picop had
understated its sales and overstated its cost of sales as set out in its Income Tax Return. For the
CIR has a right to assume that Picop's Books of Accounts speak the truth in this case since, as
already noted, they embody what must appear to be admissions against Picop's own interest.
90

Accordingly, we must affirm the findings of the Court of Appeals and the CTA.
(2) Whether Picop is liable for
the corporate development
tax of five percent (5%)
of its income for 1977.
The five percent (5%) corporate development tax is an additional corporate income tax imposed
in Section 24 (e) of the 1977 Tax Code which reads in relevant part as follows:
(e) Corporate development tax. In addition to the tax imposed in subsection
(a) of this section, an additional tax in an amount equivalent to 5 per cent of the
same taxable net income shall be paid by a domestic or a resident foreign
corporation; Provided, That this additional tax shall be imposed only if the net
income exceeds 10 per cent of the net worth, in case of a domestic corporation,
or net assets in the Philippines in case of a resident foreign corporation: . . . .
The additional corporate income tax imposed in this subsection shall be collected
and paid at the same time and in the same manner as the tax imposed in
subsection (a) of this section.
Since this five percent (5%) corporate development tax is an income tax, Picop is not exempted
from it under the provisions of Section 8 (a) of R.A. No. 5186.
For purposes of determining whether the net income of a corporation exceeds ten percent
(10%) of its net worth, the term "net worth" means the stockholders' equity represented by the
excess of the total assets over liabilities as reflected in the corporation's balance sheet provided
such balance sheet has been prepared in accordance with generally accepted accounting
principles employed in keeping the books of the corporation. 43
The adjusted net income of Picop for 1977, as will be seen below, is P48,687,355.00. Its net
worth figure or total stockholders' equity as reflected in its Audited Financial Statements for
1977 is P464,749,528.00. Since its adjusted net income for 1977 thus exceeded ten percent
(10%) of its net worth, Picop must be held liable for the five percent (5%) corporate
development tax in the amount of P2,434,367.75.
Recapitulating, we hold:
(1) Picop is liable for the thirty-five percent (35%) transaction tax in the amount of
P3,578,543.51.
(2) Picop is not liable for interest and surcharge on unpaid transaction tax.
(3) Picop is exempt from payment of documentary and science stamp taxes in the amount of
P300,000.00 and the compromise penalty of P300.00.
(4) Picop is entitled to its claimed deduction of P42,840,131.00 for interest payments on loans
for, among other things, the purchase of machinery and equipment.
(5) Picop's claimed deduction in the amount of P44,196,106.00 for the operating losses
previously incurred by RPPM, is disallowed for lack of merit.
91

(6) Picop's claimed deduction for certain financial guarantee expenses in the amount
P1,237,421.00 is disallowed for failure adequately to prove such expenses.
(7) Picop has understated its sales by P2,391,644.00 and overstated its cost of sales by
P604,018.00, for 1977.
(8) Picop is liable for the corporate development tax of five percent (5%) of its adjusted net
income for 1977 in the amount of P2,434,367.75.
Considering conclusions nos. 4, 5, 6, 7 and 8, the Court is compelled to hold Picop liable for
deficiency income tax for the year 1977 computed as follows:
Deficiency Income Tax
Net Income Per Return P 258,166.00
Add:
Unallowable Deductions
(1) Deduction of net
operating losses
incurred by RPPM P 44,196,106.00
(2) Unexplained financial
guarantee expenses P 1,237,421.00
(3) Understatement of
Sales P 2,391,644.00
(4) Overstatement of
Cost of Sales P 604,018.00

Total P 48,429,189.00

Net Income as Adjusted P 48,687,355.00


===========
Income Tax Due Thereon 44 P 17,030,574.00
Less:
Tax Already Assessed per
Return 80,358.00

92

Deficiency Income Tax P 16,560,216.00


Add:
Five percent (5%) Corporate
Development Tax P 2,434,367.00
Total Deficiency Income Tax P 18,994,583.00
===========
Add:
Five percent (5%) surcharge 45 P 949,729.15

Total Deficiency Income Tax


with surcharge P 19,944,312.15
Add:
Fourteen percent (14%)
interest from 15 April
1978 to 14 April 1981 46 P 8,376,610.80
Fourteen percent (14%)
interest from 21 April
1983 to 20 April 1986 47 P 11,894,787.00

Total Deficiency Income Tax


Due and Payable P 40,215,709.00
===========
WHEREFORE, for all the foregoing, the Decision of the Court of Appeals is hereby MODIFIED
and Picop is hereby ORDERED to pay the CIR the aggregate amount of P43,794,252.51
itemized as follows:
(1) Thirty-five percent (35%)
transaction tax P 3,578,543.51
93

(2) Total Deficiency Income


Tax Due 40,215,709.00

Aggregate Amount Due and Payable P 43,794,252.51


============
No pronouncement as to costs.
SO ORDERED.
Narvasa, C.J., Regalado, Davide, Jr., Romero, Bellosillo, Melo, Puno, Kapunan, Mendoza,
Francisco, Hermosisima, Jr. and Panganiban, JJ., concur.
Padilla, J., took no part.

Separate Opinions
VITUG, J., concurring and dissenting:
In usual erudite manner, Mr. Justice Florentino P. Feliciano has written for the Court
the ponencia that presents in clear and logical sequence the issues, the facts and the law
involved. While I share, in most part, the conclusions expressed in the opinion, I regrettably find
it difficult, nevertheless, not to propose a re-examination of the Court's holding in Western
Minolco Corporation vs. Commissioner of Internal Revenue (124 SCRA 121), reiterated
inMarinduque Mining and Industrial Corporation vs. Commissioner of Internal Revenue (137
SCRA 88), that has taken the 35% transaction tax on commercial papers issued in the primary
market under the 1977 Revenue Code, in relation to Republic Act ("R.A.") 5186, to be an
income tax.
R.A. No. 5186, also known as the Investment Incentives Act, has provided for incentives by,
among other things, granting to registered pioneer enterprises an exemption from all taxes,
except income tax, under the National Internal Revenue Code. The income tax, referred to, in
my view, is that imposed in Title II, entitled "Income Tax," of the Revenue Code. Nowhere under
that title is there a 35% transaction tax.
There was, to be sure, a 35% transaction tax still in effect in 1977 but it was a tax not on the
investor-lender in whose favor the interest income on the commercial paper accrues. The tax
was, instead, levied on the borrower-issuer of commercial papers transacted in the primary
market. Being the principal taxpayer, the borrower-issuer could not have been likewise
contemplated to be a mere tax withholding agent. The tax was conceived as a tax on business
transaction, and so it was rightly incorporated in Title V, entitled "Privilege Taxes on Business
and Occupation" of the Tax Code.

94

The fact that a taxpayer on whom the tax is imposed can shift, characteristic of indirect taxes,
the burden thereof to another does not make the latter the taxpayer and the former the
withholding agent. Indeed, the facility of shifting the burden of the tax is opposed to the idea of
a direct tax to which class the income tax actually belongs.
Accordingly, I vote to so reduce the tax liability of petitioners as adjudged by the amount
corresponding to the 35% transaction tax. In all other respects, I concur with the majority in the
judgment.
Separate Opinions
VITUG, J., concurring and dissenting:
In usual erudite manner, Mr. Justice Florentino P. Feliciano has written for the Court
the ponencia that presents in clear and logical sequence the issues, the facts and the law
involved. While I share, in most part, the conclusions expressed in the opinion, I regrettably find
it difficult, nevertheless, not to propose a re-examination of the Court's holding in Western
Minolco Corporation vs. Commissioner of Internal Revenue (124 SCRA 121), reiterated in
Marinduque Mining and Industrial Corporation vs. Commissioner of Internal Revenue (137
SCRA 88), that has taken the 35% transaction tax on commercial papers issued in the primary
market under the 1977 Revenue Code, in relation to Republic Act ("R.A.") 5186, to be an
income tax.
R.A. No. 5186, also known as the Investment Incentives Act, has provided for incentives by,
among other things, granting to registered pioneer enterprises an exemption from all taxes,
except income tax, under the National Internal Revenue Code. The income tax, referred to, in
my view, is that imposed in Title II, entitled "Income Tax," of the Revenue Code. Nowhere under
that title is there a 35% transaction tax.
There was, to be sure, a 35% transaction tax still in effect in 1977 but it was a tax not on the
investor-lender in whose favor the interest income on the commercial paper accrues. The tax
was, instead, levied on the borrower-issuer of commercial papers transacted in the primary
market. Being the principal taxpayer, the borrower-issuer could not have been likewise
contemplated to be a mere tax withholding agent. The tax was conceived as a tax on business
transaction, and so it was rightly incorporated in Title V, entitled "Privilege Taxes on Business
and Occupation" of the Tax Code.
The fact that a taxpayer on whom the tax is imposed can shift, characteristic of indirect taxes,
the burden thereof to another does not make the latter the taxpayer and the former the
withholding agent. Indeed, the facility of shifting the burden of the tax is opposed to the idea of
a direct tax to which class the income tax actually belongs.
Accordingly, I vote to so reduce the tax liability of petitioners as adjudged by the amount
corresponding to the 35% transaction tax. In all other respects, I concur with the majority in the
judgment.

95

Republic of the Philippines


SUPREME COURT
Manila
EN BANC
G.R. Nos. L-18169, L-18262 & L-21434

July 31, 1964

COMMISSIONER OF INTERNAL REVENUES, petitioner,


vs.
V.E. LEDNICKY and MARIA VALERO LEDNICKY, respondents.
Office of the Solicitor General for petitioner.
Ozaeta, Gibbs and Ozaeta for respondents.
REYES, J.B.L., J.:
The above-captioned cases were elevated to this Court under separate petitions by the
Commissioner for review of the corresponding decisions of the Court of Tax Appeals. Since
these cases involve the same parties and issues akin to each case presented, they are herein
decided jointly.
The respondents, V. E. Lednicky and Maria Valero Lednicky, are husband and wife, respectively,
both American citizens residing in the Philippines, and have derived all their income from
Philippine sources for the taxable years in question.
In compliance with local law, the aforesaid respondents, on 27 March 1957, filed their income
tax return for 1956, reporting therein a gross income of P1,017,287. 65 and a net income of
P733,809.44 on which the amount of P317,395.4 was assessed after deducting P4,805.59 as
96

withholding tax. Pursuant to the petitioner's assessment notice, the respondents paid the total
amount of P326,247.41, inclusive of the withheld taxes, on 15 April 1957.
On 17 March 1959, the respondents Lednickys filed an amended income tax return for 1956.
The amendment consists in a claimed deduction of P205,939.24 paid in 1956 to the United
States government as federal income tax for 1956. Simultaneously with the filing of the
amended return, the respondents requested the refund of P112,437.90.
When the petitioner Commissioner of Internal Revenue failed to answer the claim for refund, the
respondents filed their petition with the Tax Court on 11 April 1959 as CTA Case No. 646, which
is now G. R. No. L-18286 in the Supreme Court.
G. R. No. L-18169 (formerly CTA Case No. 570) is also a claim for refund in the amount of
P150,269.00, as alleged overpaid income tax for 1955, the facts of which are as follows:
On 28 February 1956, the same respondents-spouses filed their domestic income tax return for
1955, reporting a gross income of P1,771,124.63 and a net income of P1,052,550.67. On 19
April 1956, they filed an amended income tax return, the amendment upon the original being a
lesser net income of P1,012,554.51, and, on the basis of this amended return, they paid
P570,252.00, inclusive of withholding taxes. After audit, the petitioner determined a deficiency of
P16,116.00, which amount, the respondents paid on 5 December 1956.
Back in 1955, however, the Lednickys filed with the U.S. Internal Revenue Agent in Manila their
federal income tax return for the years 1947, 1951, 1952, 1953, and 1954 on income from
Philippine sources on a cash basis. Payment of these federal income taxes, including penalties
and delinquency interest in the amount of P264,588.82, were made in 1955 to the U.S. Director
of Internal Revenue, Baltimore, Maryland, through the National City Bank of New York, Manila
Branch. Exchange and bank charges in remitting payment totaled P4,143.91.
Wherefore, the parties respectfully pray that the foregoing stipulation of facts be admitted and
approved by this Honorable Court, without prejudice to the parties adducing other evidence to
prove their case not covered by this stipulation of facts.
On 11 August 1958, the said respondents amended their Philippine income tax return for 1955
to include the following deductions:
U.S. Federal income taxes
Interest accrued up to May 15, 1955
Exchange and bank charges
Total

P471,867.32
40,333.92
4,143.91
P516,345.15

and therewith filed a claim for refund of the sum of P166,384.00, which was later reduced to
P150,269.00.
The respondents Lednicky brought suit in the Tax Court, which was docketed therein as CTA
Case No. 570.
In G. R. No. 21434 (CTA Case No. 783), the facts are similar, but refer to respondents
Lednickys' income tax return for 1957, filed on 28 February 1958, and for which respondents
97

paid a total sum of P196,799.65. In 1959, they filed an amended return for 1957, claiming
deduction of P190,755.80, representing taxes paid to the U.S. Government on income derived
wholly from Philippine sources. On the strength thereof, respondents seek refund of P90 520.75
as overpayment. The Tax Court again decided for respondents.
The common issue in all three cases, and one that is of first impression in this jurisdiction, is
whether a citizen of the United States residing in the Philippines, who derives income wholly
from sources within the Republic of the Philippines, may deduct from his gross income the
income taxes he has paid to the United States government for the taxable year on the strength
of section 30 (C-1) of the Philippine Internal Revenue Code, reading as follows:
SEC. 30. Deduction from gross income. In computing net income there shall be
allowed as deductions
(a) ...
(b) ...
(c) Taxes:
(1) In general. Taxes paid or accrued within the taxable year, except
(A) The income tax provided for under this Title;
(B) Income, war-profits, and excess profits taxes imposed by the
authority of any foreign country; but this deduction shall be
allowed in the case of a taxpayer who does not signify in his
return his desire to have to any extent the benefits of paragraph
(3) of this subsection (relating to credit for foreign countries);
(C) Estate, inheritance and gift taxes; and
(D) Taxes assessed against local benefits of a kind tending to
increase the value of the property assessed. (Emphasis supplied)
The Tax Court held that they may be deducted because of the undenied fact that the
respondent spouses did not "signify" in their income tax return a desire to avail
themselves of the benefits of paragraph 3 (B) of the subsection, which reads:
Par. (c) (3) Credits against tax for taxes of foreign countries. If the taxpayer
signifies in his return his desire to have the benefits of this paragraph, the tax
imposed by this Title shall be credited with
(A) ...;
(B) Alien resident of the Philippines. In the case of an alien resident of
the Philippines, the amount of any such taxes paid or accrued during the
taxable year to any foreign country, if the foreign country of which such
alien resident is a citizen or subject, in imposing such taxes, allows a
similar credit to citizens of the Philippines residing in such country;

98

It is well to note that the tax credit so authorized is limited under paragraph 4 (A and B)
of the same subsection, in the following terms:
Par. (c) (4) Limitation on credit. The amount of the credit taken under this
section shall be subject to each of the following limitations:
(A) The amount of the credit in respect to the tax paid or accrued to any
country shall not exceed the same proportion of the tax against which
such credit is taken, which the taxpayer's net income from sources within
such country taxable under this Title bears to his entire net income for the
same taxable year; and
(B) The total amount of the credit shall not exceed the same proportion of
the tax against which such credit is taken, which the taxpayer's net
income from sources without the Philippines taxable under this Title bears
to his entire net income for the same taxable year.
We agree with appellant Commissioner that the Construction and wording of
Section 30 (c) (1) (B) of the Internal Revenue Act shows the law's intent that the
right to deduct income taxes paid to foreign government from the taxpayer's
gross income is given only as an alternative or substitute to his right to claim a
tax credit for such foreign income taxes under section 30 (c) (3) and (4); so that
unless the alien resident has a right to claim such tax credit if he so chooses, he
is precluded from deducting the foreign income taxes from his gross income. For
it is obvious that in prescribing that such deduction shall be allowed in the case
of a taxpayer who does not signify in his return his desire to have to any extent
the benefits of paragraph (3) (relating to credits for taxes paid to foreign
countries), the statute assumes that the taxpayer in question also may signify his
desire to claim a tax credit and waive the deduction; otherwise, the foreign taxes
would always be deductible, and their mention in the list of non-deductible items
in Section 30(c) might as well have been omitted, or at least expressly limited to
taxes on income from sources outside the Philippine Islands.
Had the law intended that foreign income taxes could be deducted from gross
income in any event, regardless of the taxpayer's right to claim a tax credit, it is
the latter right that should be conditioned upon the taxpayer's waiving the
deduction; in which Case the right to reduction under subsection (c-1-B) would
have been made absolute or unconditional (by omitting foreign taxes from the
enumeration of non-deductions), while the right to a tax credit under subsection
(c-3) would have been expressly conditioned upon the taxpayer's not claiming
any deduction under subsection (c-1). In other words, if the law had been
intended to operate as contended by the respondent taxpayers and by the Court
of Tax Appeals section 30 (subsection (c-1) instead of providing as at present:
SEC. 30. Deduction from gross income. In computing net income there shall be
allowed as deductions
(a) ...
(b) ...
(c) Taxes:
99

(1) In general. Taxes paid or accrued within the taxable year, except
(A) The income tax provided for under this Title;
(B) Income, war-profits, and excess profits taxes imposed by the
authority of any foreign country; but this deduction shall be
allowed in the case of a taxpayer who does not signify in his return
his desire to have to any extent the benefits of paragraph (3) of
this subsection (relating to credit for taxes of foreign countries);
(C) Estate, inheritance and gift taxes; and
(D) Taxes assessed against local benefits of a kind tending to
increase the value of the property assessed.
would have merely provided:
SEC. 30. Decision from grow income. In computing net income there shall be allowed
as deductions:
(a) ...
(b) ...
(c) Taxes paid or accrued within the taxable year, EXCEPT
(A) The income tax provided for in this Title;
(B) Omitted or else worded as follows:
Income, war profits and excess profits taxes imposed by authority of any foreign
country on income earned within the Philippines if the taxpayer does not claim
the benefits under paragraph 3 of this subsection;
(C) Estate, inheritance or gift taxes;
(D) Taxes assessed against local benefits of a kind tending to increase the value
of the property assessed.
while subsection (c-3) would have been made conditional in the following or equivalent terms:
(3) Credits against tax for taxes of foreign countries. If the taxpayer has not deducted
such taxes from his gross income but signifies in his return his desire to have the
benefits of this paragraph, the tax imposed by Title shall be credited with ... (etc.).
Petitioners admit in their brief that the purpose of the law is to prevent the taxpayer from
claiming twice the benefits of his payment of foreign taxes, by deduction from gross income
(subs. c-1) and by tax credit (subs. c-3). This danger of double credit certainly can not exist if
the taxpayer can not claim benefit under either of these headings at his option, so that he must
be entitled to a tax credit (respondent taxpayers admittedly are not so entitled because all their
income is derived from Philippine sources), or the option to deduct from gross income
disappears altogether.
100

Much stress is laid on the thesis that if the respondent taxpayers are not allowed to deduct the
income taxes they are required to pay to the government of the United States in their return for
Philippine income tax, they would be subjected to double taxation. What respondents fail to
observe is that double taxation becomes obnoxious only where the taxpayer is taxed twice for
the benefit of the same governmental entity (cf. Manila vs. Interisland Gas Service, 52 Off. Gaz.
6579; Manuf. Life Ins. Co. vs. Meer, 89 Phil. 357). In the present case, while the taxpayers
would have to pay two taxes on the same income, the Philippine government only receives the
proceeds of one tax. As between the Philippines, where the income was earned and where the
taxpayer is domiciled, and the United States, where that income was not earned and where the
taxpayer did not reside, it is indisputable that justice and equity demand that the tax on the
income should accrue to the benefit of the Philippines. Any relief from the alleged double
taxation should come from the United States, and not from the Philippines, since the former's
right to burden the taxpayer is solely predicated on his citizenship, without contributing to the
production of the wealth that is being taxed.
Aside from not conforming to the fundamental doctrine of income taxation that the right of a
government to tax income emanates from its partnership in the production of income, by
providing the protection, resources, incentive, and proper climate for such production, the
interpretation given by the respondents to the revenue law provision in question operates, in its
application, to place a resident alien with only domestic sources of income in an equal, if not in a
better, position than one who has both domestic and foreign sources of income, a situation
which is manifestly unfair and short of logic.
Finally, to allow an alien resident to deduct from his gross income whatever taxes he pays to his
own government amounts to conferring on the latter the power to reduce the tax income of the
Philippine government simply by increasing the tax rates on the alien resident. Everytime the
rate of taxation imposed upon an alien resident is increased by his own government, his
deduction from Philippine taxes would correspondingly increase, and the proceeds for the
Philippines diminished, thereby subordinating our own taxes to those levied by a foreign
government. Such a result is incompatible with the status of the Philippines as an independent
and sovereign state.
IN VIEW OF THE FOREGOING, the decisions of the Court of Tax Appeals are reversed, and,
the disallowance of the refunds claimed by the respondents Lednicky is affirmed, with costs
against said respondents-appellees.
Bengzon, C.J., Padilla, Bautista Angelo, Labrador, Concepcion, Paredes, Regala and
Makalintal, JJ., concur.

101

Republic of the Philippines


SUPREME COURT
Manila
EN BANC
G.R. No. L-19537

May 20, 1965

The late LINO GUTIERREZ substituted by ANDREA C. VDA. DE GUTIERREZ, ANTONIO D.


GUTIERREZ, GUILLERMO D. GUTIERREZ, SANTIAGO D. GUTIERREZ and TOMAS D.
GUTIERREZ, petitioners,
vs.
COLLECTOR (now COMMISSIONER) OF INTERNAL REVENUE, respondent.
Rosendo J. Tansinsin, Sr., Rosendo Tansinsin, Jr. and Juan C. Nabong, Jr.for petitioners.
Office of the Solicitor General for respondent.
BENGZON, J.P., J.:
Lino Gutierrez was primarily engaged in the business of leasing real property for which he paid
estate broker's privilege tax. He filed his income tax returns for the years 1951, 1952, 1953 and
1954 on the following dates:
Year

Date Filed

1951

March 1, 1952

1952

February 28, 1953

1953

February 22, 1954

1954

February 23, 1955

and paid the corresponding tax declared therein.


102

On July 10, 1956 the Commissioner (formerly Collector) of Internal Revenue assessed against
Gutierrez the following defiency income tax:
1951 . . . . . . . . . . . . . .

P 1,400.00

1952 . . . . . . . . . . . . . .

672.00

1953 . . . . . . . . . . . . . .

5,161.00

1954 . . . . . . . . . . . . . .

4,608.00
Total . . . . . . . . . . . . . .

P 11,841.00
==========

The above defiency tax came about by the disallowance of deductions from gross income
representing depreciation, expenses Gutierrez allegely incurred in carrying on his business, and
the addition to gross income of receipts which he did not report in his income tax returns. The
disallowed business expenses which were considered by the Commissioner either as personal
or capital expenditures consisted of:

1951
Personal expenses:
Transportation expenses to attend funeral of various
persons
Repair of car and salary of driver
Expenses in attending National Convention of Filipino
Businessmen in Baguio
Alms to indigent family

P 96.50
59.80
121.35
15.00

Capital expenditures:
Electrical fixtures and supplies

100.00

Transportation and other expenses to watch laborers in


construction work

516.00

Realty tax not paid by former owner of property acquired


by Gutierrez

350.00

Litigation expenses to collect rental and eject lessee

702.65

Other disallowed deductions:


Fines and penalties for late payment of taxes

64.48

1952
103

Personal expenses:
Car expenses, salary of driver and car depreciation
Contribution to Lydia Samson and G. Trinidad
Officers' jewels and aprons donated to Biak-na-Bato
Lodge No. 7, Free Masons
Luncheon of Homeowners' Association
Ticket to opera "Aida"

P1,454.37
52.00
280.00
5.50
15.00

1953
Personal expenses:
Car expenses, salary of driver, car depreciation

P 1,409.24

Cruise to Corregidor with Homeowners' Association

43.00

Contribution to alms to various individuals

70.00

Tickets to operas

28.00

Capital expenditures:
Cost of one set of Comments on the Rules of Court by
Moran

P 145.00

1954
Personal expenses:
Car expenses, salary of driver and car depreciation
Furniture given as commission in connection with
business transaction
Cost of iron door of Gutierrez' residence

P
1,413.67
115.00
55.00

Capital expenditures:
Painting of rental apartments

P 908.00

Carpentry and lumber for rental apartments

335.83

Tinsmith and plumbing for rental apartments

605.25

Cement, tiles, gravel, sand and masonry for rental


apartments

199.48

Iron bars, venetian blind, water pumps for rental

1,340.00
104

apartments
Relocation and registration of property used in
taxpayer's business

1,758.12

He also claimed the depreciation of his residence as follows:


1952 . . . . . .

P 992.22

1953 . . . . . .

942.61

1954 . . . . . .

895.45

The following are the items of income which Gutierrez did not declare in his
income tax returns:
1951
Income of wife (admitted by Gutierrez)

P 2,749.90.

1953
Overstatement of purchase price of real estate
Understatement of profits from sale of real estate

P 8,476.92
5,803.74

1954
Understatement of profits from sale of real estate

P 5,444.24

The overstatement of purchase price of real estate refers to the sale of two pieces of property in
1953. In 1943 Gutierrez bought a parcel of land situated along Padre Faura St. in Manila for
P35,000.00. Sometime in 1953, he sold the same for P30,400.00. Expenses of sale amounted
to P631.80. In his return he claimed a loss of P5,231.80. 1 However, the Commissioner,
including the said property was bought in Japanese military notes, converting the buying price to
its equivalent in Philippine Commonwealth peso by the use of the Ballantyne Scale of Values. At
P1.30 Japanese military notes per Commonwealth peso, the acquisition cost of P35,000.00
Japanese military notes was valued at P26,923.00 Philippine Commonwealth peso. Accordingly,
the Commissioner determined a profit of P3,476.92 after restoring to Gutierrez' gross income
the P5,231.80 deduction for loss.
In another transaction, Gutierrez sold a piece of land for P1,200.00. Alleging the said property
was purchased for P1,200.00, he reported no profit hereunder. However, after verifying the deed
of acquisition, the Commissioner discovered the purchase price to be only P800.00.
Consequently, he determined a profit of P400.00 which was added to the gross income for
1953.1wph1.t
The understatement of profit from the sale of real estate may be explained thus: In 1953 and
1954 Gutierrez sold four other properties upon which he made substantial profits.2Convinced
that said properties were capital assets, he declared only 50% of the profits from their sale.
However, treating said properties as ordinary assets (as property held and used byGutierrez in
105

his business), the Commissioner taxed 100% of the profits from their disposition pursuant to
Section 35 of the Tax Code.
Having unsuccessfully questioned the legality and correctness of the aforesaid assessment,
Gutierrez instituted on February 17, 1958, the Commissioner issued a warrant of distraint and
levy on one of Gutierrez' real properties but desisted from enforcing the same when Gutierrez
filed a bond to assure payment of his tax liability.
In a decision dated January 28, 1962, the Court of Tax Appeals upheld in toto the assessment of
the Commissioner of Internal Revenue. Hence, this appeal.
On October 18, 1962, Lino Guttierrez died and he was substituted by Andrea C. Vda. de
Gutierrez, Antonio D. Gutierrez, Santiago D. Gutierrez, Guillermo D. Gutierrez and Tomas D.
Gutierrez, his heirs, as party petitioners.
The issues are: (1) Are the taxpayer's aforementioned claims for deduction proper and
allowable? (2) May the Ballantyne Scale of Values be applied in determining the acquisition cost
in 1943 of a real property sold in 1953, for income tax purposes? (3) Are real properties used in
the trade or business of the taxpayer capital or ordinary assets? (4) Has the right of the
Commissioner of Internal Revenue to collect the deficiency income tax for the years 1951 and
1952 prescribed? (5) Has the right of the Commissioner of Internal Revenue to collect by
distraint and levy the deficiency income tax for 1953 prescribed? If not, may the taxpayer's rea
lproperty be distrained and levied upon without first exhausting his personal property?
We come first to question whether or not the deductions claimed by Gutierrez are allowable.
Section 30(a) of the Tax Code allows business expenses tobe deducted from gross income. We
quote:
SEC. 30. Deductions from gross income. In computing net income there shall be
allowed as deductions
(a) Expenses:
(1) In general. All the ordinary and necessary expenses paid or incurred during the
taxable year in carrying on any trade or business, including a reasonable allowance for
salaries or other compensation for personal services actually rendered; travelling
expenses while away from home in the pursuit of a trade or business; and rentals or
other payments required to be made as a condition to be continued use of possession,
for the purposes of the trade or business, or property to which the taxpayer has not
taken or is not taking title or in which he has no equity.
To be deductible, therefore, an expense must be (1) ordinary and necessary;(2) paid or incurred
within the taxable year; and, (3) paid or incurred in carrying on a trade or business. 3
The transportation expenses which petitioner incurred to attend the funeral of his friends and the
cost of admission tickets to operas were expenses relative to his personal and social activities
rather than to his business of leasing real estate. Likewise, the procurement and installation of
an iron door to is residence is purely a personal expense. Personal, living, or family expenses
are not deductible. 4
On the other hand, the cost of furniture given by the taxpayer as commission in furtherance of a
business transaction, the expenses incurred in attending the National Convention of Filipino
106

Businessmen, luncheon meeting and cruise to Corregidor of the Homeowners' Association were
shown to have been made in the pursuit of his business. Commissions given in consideration
for bringing about a profitable transaction are part of the cost of the business transaction and
are deductible.
The record shows that Gutierrez was an officer of the Junior Chamber of Commerce which
sponsored the National Convention of Filipino Businessmen. He was also the president of the
Homeowners' Association, an organization established by those engaged in the real estate
trade. Having proved that his membership thereof and activities in connection therewith were
solely to enhance his business, the expenses incurred thereunder are deductible as ordinary
and necessary business expenses.
With respect to the taxpayer's claim for deduction for car expenses, salary of his driver and car
depreciation, one-third of the same was disallowed by the Commissioner on the ground that the
taxpayer used his car and driver both for personal and business purposes. There is no clear
showing, however, that the car was devoted more for the taxpayer's business than for his
personal and business needs. 5 According to the evidence, the taxpayer's car was utilized both
for personal and business needs. We therefore find it reasonable to allow as deduction one-half
of the driver's salary, car expenses and depreciation.
The electrical supplies, paint, lumber, plumbing, cement, tiles, gravel, masonry and labor used
to repair the taxpayer's rental apartments did not increase the value of such apartments, or
prolong their life. They merely kept the apartments in an ordinary operating condition. Hence,
the expenses incurred therefor are deductible as necessary expenditures for the maintenance of
the taxpayer's business.
Similarly, the litigation expenses defrayed by Gutierrez to collect apartment rentals and to eject
delinquent tenants are ordinary and necessary expenses in pursuing his business. It is routinary
and necessary for one in the leasing business to collect rentals and to eject tenants who refuse
to pay their accounts.
The following are not deductible business expenses but should be integrated into the cost of the
capital assets for which they were incurred and depreciated yearly: (1) Expenses in watching
over laborers in construction work. Watching over laborers is an activity more akin to the
construction work than to running the taxpayer's business. Hence, the expenses incurred
therefor should form part of the construction cost. (2) Real estate tax which remained unpaid by
the former owner of Gutierrez' rental property but which the latter paid, is an additional cost to
acquire such property and ought therefore to be treated as part of the property's purchase price.
(3) The iron bars, venetian blind and water pump augmented the value of the, apartments where
they were installed. Their cost is not a maintenance charge, 6 hence, not deductible.. 7 (4)
Expenses for the relocation, survey and registration of property tend to strengthen title over the
property, hence, they should be considered as addition to the costs of such property. (5) The set
of "Comments on the Rules of Court" having a life span of more than one year should be
depreciated ratably during its whole life span instead of its total cost being deducted in one year.
Coming to the claim for depreciation of Gutierrez' residence, we find the same not deductible. A
taxpayer may deduct from gross income a reasonable allowance for deterioration of property
arising out of its use or employment in business or trade. 8 Gutierrez' residence was not used in
his trade or business.
Gutierrez also claimed for deduction the fines and penalties which he paid for late payment of
taxes. While Section 30 allows taxes to be deducted from gross income, it does not specifically
allow fines and penalties to be so deducted. Deductions from gross income are matters of
107

legislative grace; what is not expressly granted by Congress is withheld. Moreover, when acts
are condemned, by law and their commission is made punishable by fines or forfeitures, to allow
them to be deducted from the wrongdoer's gross income, reduces, and so in part defeats, the
prescribed punishment..9
As regards the alms to an indigent family and various individuals, contributions to Lydia Yamson
and G. Trinidad and a donation consisting of officers' jewels and aprons to Biak-na-Bato Lodge
No. 7, the same are not deductible from gross income inasmuch as their recipients have not
been shown to be among those specified by law. Contributions are deductible when given to the
Government of the Philippines, or any of its political subdivisions for exclusively public
purposes, to domestic corporations or associations organized and operated exclusively for
religious, charitable, scientific, athletic, cultural or educational purposes, or for the rehabilitation
of veterans, or to societies for the prevention of cruelty to children or animals, no part of the net
income of which inures to the benefit of any private stockholder or individual. 10
We come to the question of whether or not the Ballantyne Scale of Values can be applied to tax
cases.
Sometime in 1943 Gutierrez bought a piece of real estate in Manila for a price of P35,000.00. In
1953 he sold said property for P30,400.00, thereby incurring a loss which he claimed as
deduction in his income tax return for 1953. The Commissioner of Internal Revenue, convinced
that the purchase price of the property in 1943 was in Japanese military notes, converted said
purchase price into Philippine Commonwealth pesos by the use of the Ballantyne Scale of
Values. As a result, the Commissioner found Gutierrez to have profited, instead of lost in the
sale.
Firstly, Gutierrez maintains that the purchase price was paid for in Commonwealth pesos. On
the other hand the Commissioner insists that inasmuch as the prevailing currency in the City of
Manila in 1943 was the Japanese military issue, the transaction could have been in said military
notes. The evidence offered by Gutierrez, consisting of the testimony of his son to the effect that
it was he who carried the bundle of Commonwealth pesos and Japanese military notes when
his father purchased the property, did not convince the Tax Court. No cogent reason to alter the
court a quo's finding of fact in this regard has been given. There is no definite showing that
Gutierrez paid for the property in Commonwealth pesos. Considering that in 1943 the medium
of exchange in Manila was the Japanese military notes, the use of which the Japanese Military
Government enforced with stringent measures, we are inclined to concur with the finding that
the purchase price was in Japanese military notes. We are specifically mindful of the fact that
Gutierrez sold the property in 1953 for only P30,400.00 at a time when the price of real estate in
the City of Manila was much greater than in 1943.
It is further contended by Gutierrez that the money he used to pay for the purchase of the
property in question came from the proceeds of merchandise acquired prior to World War II but
which he sold after Manila was occupied by the Japanese military forces, hence, the purchase
price should be deemed to have been made in Commonwealth pesos inasmuch as the
aforesaid merchandise was purchased in Commonwealth pesos. This contention, if true,
strengthens our conclusion that the real estate in question was bought in Japanese military
notes. For, at the time Gutierrez sold his merchandise, the prevailing currency in the City of
Manila was the Japanese military money. Consequently, the proceeds therefrom, which were
used to buy the real estate in question, were Japanese military notes.
Gutierrez assails the use of the Ballantyne Scale of Values in converting the purchase price of
the real estate in question from Japanese military notes to Philippine Commonwealth pesos on
the ground that (1) the Ballantyne Scale of Values was intended only for transactions entered
108

into by parties voluntarily during the Japanese occupation, wherein a portion of the contract was
left unperformed until liberation of the Philippines by the Americans; (2) that such Scale of
Values cannot be the basis of a tax, for it is not a law.
In determining the gain or loss from the sale of property the purchase price and the selling price
ought to be in the same currency. Since in this case the purchase price was in Japanese military
notes and the selling price was in our present legal tender, the Japanese military notes should
be converted to the present currency. Since the only standard scale recognized by courts for the
purpose is the Ballantyne Scale of Values, we find it compelling to use such table of values
rather than adopt an arbitrary scale. It may not be amiss to state in this connection that the
Ballantyne Scale of Values is not being used herein as the authority to impose the tax, but only
as a medium of computing the tax base upon which the tax is to be imposed.
It is furthermore proffered by the taxpayer that in determining gain or loss, the real value of the
Commonwealth peso at the time the property was purchased and the value of the Republic
peso at the time. the same property was sold should be considered. The Commonwealth peso
and the Republic peso are the same currency, with the same intrinsic value, sanctioned by the
same authorities. Both are legal tender and accepted at face value regardless of fluctuation in
their buying power. The 1941 Commonwealth peso when used to buy in 1963 or in 1965 is
accorded the same value: one peso.
In his income tax returns for 1953 and 1954, Gutierrez reported only 50% of profits he realized
from the sale of real properties during the years 1953 and 1954 on the ground that said
properties were capital assets. Profits from the sale of capital assets are taxable to the extent of
50% thereof pursuant to Section 34 of the Tax Code.
Section 34 provides:
SEC. 34. Capital gains and losses. (a) Definitions. As used in this title
(1) Capital assets. The term "capital assets" means property held by the taxpayer
(whether or not connected with his trade or business), but does not include stock in trade
of the taxpayer or other property of a kind which would properly be included in the
inventory of the taxpayer if on hand at the close of the taxable year, or property held by
the taxpayer primarily for sale to customers in the ordinary course of his trade or
business, or property used in the trade or business, of a character which is subject to the
allowance for depreciation provided in subsection (f) of section thirty; or real property
used in the trade or business of the taxpayer.
xxx

xxx

xxx

(b) Percentage taken into account. In the case of a taxpayer, other than a corporation,
only the following percentages of the gain or loss recognized upon the sale or exchange
of capital asset hall be taken into account in computing net capital gain, net capital loss,
and net income:
(1) One hundred per centum if the capital asset has been held for not more than twelve
months;
(2) Fifty per centum if the capital asset has been held for not more than twelve months.

109

Section 34, before it was amended by Republic Act 82 in 1947, considered as capital assets
real property used in the trade or business of a taxpayer. However, with the passage of Republic
Act 82, Congress classified "real property used in the trade or business of the taxpayer" is
ordinary asset. The explanatory note to Republic Act 82 says "... the words "or real property
used in the trade or business of the taxpayer" have been included among the non-capital
assets. This has the effect of withdrawing the gain or loss from the sale or exchange of real
property used in the trade or business of the taxpayer from the operation of the capital gains
and losses provisions. As such real property is used in the trade or business of the taxpayer, it is
logical that the gain or loss from the sale or exchange thereof should be treated as ordinary
income or loss. 11 Accordingly, the real estate, admittedly used by Gutierrez in his business,
which he sold in 1953 and 1954 should be treated as ordinary assets and the gain from the sale
thereof, as ordinary gain, hence, fully taxable. 12
With regard to the issue of the prescription of the Commissioner's right to collect deficiency tax
for 1951 and 1952, Gutierrez claims that the counting of the 5-year period to collect income tax
should start from the time the income tax returns were filed. He, therefore, urges us to declare
the Commissioner's right to collect the deficiency tax for 1951 and 1952 to have prescribed, the
income tax returns for 1951 and 1952 having been filed in March 1952 and on February 28,
1953, respectively, and the action to collect the tax having been instituted on March 5, 1958
when the Commissioner filed his answer to the petition for review in C.T.A. Case No. 504. On
the other hand, the Commissioner argues that the running of the prescriptive period to collect
commences from the time of assessment. Inasmuch as the tax for 1951 and 1952 were
assessed only on July 10, 1956, less than five years lapsed when he filed his answer on March
5, 1958.
The period of limitation to collect income tax is counted from the assessment of the tax as
provided for in paragraph (c) of Section 332 quoted below:
SEC. 332(c). Where the assessment of any internal revenue tax has been made within
the period of limitation above prescribed such tax may be collected by distraint or levy or
by a proceeding in court, but only if begun (1) within five years after the assessment of
the tax, or (2) prior to the expiration of any period for collection agreed upon in writing by
the Collector of Internal Revenue and the taxpayer before the expiration of such fiveyear period. The period so agreed upon may be extended by subsequent agreements in
writing made before the expiration of the period previously agreed upon.
Inasmuch as the assessment for deficiency income tax was made on July 10, 1956 which is 7
months and 25 days prior to the action for collection, the right of the Commissioner to collect
such tax has not prescribed.
The next issue relates to the prescription of the right of the Commissioner of Internal Revenue
to collect the deficiency tax for 1954 by distraint and levy.
The pertinent provision of the Tax Code states:
SEC. 51(d). Refusal or neglect to make returns; fraudulent returns, etc. In cases of
refusal or neglect to make a return and in cases of erroneous, false, or fraudulent
returns, the Collector of Internal Revenue shall, upon the discovery thereof, at any time
within three years after said return is due or has been made, make a return upon
information obtained as provided for in this code or by existing law, or require the
necessary corrections to be made, and the assessment made by the Collector of Internal
Revenue thereon shall be paid by such person or corporation immediately upon
notification of the amount of such assessment.
110

On February 23, 1955 Gutierrez filed his income tax return for 1954 and on February 24, 1958
the Commissioner of Internal Revenue issued a warrant of distraint and levy to collect the tax
due thereunder. Gutierrez contends that the Commissioner's right to issue said warrant is
barred, for the same was issued more than 3 years from the time he filed his income tax return.
On the other hand, the Commissioner of Internal Revenue maintains that his right did not lapse
inasmuch as from the last day prescribed by law for the filing of the 1954 return to the date
when he issued the warrant of distraint and levy, less than 3 years passed. The question now is:
should the counting of the prescriptive period commence from the actual filing of the return or
from the last day prescribed by law for the filing thereof?
We observe that Section 51(d) speaks of erroneous, false or fraudulent returns, and refusal or
neglect of the taxpayer to file a return. It also provides for two dates from which to count the
three-year prescriptive period, namely, the date when the return is due and the date the return
has been made. We are inclined to conclude that the date when the return is due refers to
cases where the taxpayer refused or neglected to file a return, and the date when the return has
been made refers to instances where the taxpayer filed erroneous, false or fraudulent returns.
Since Gutierrez filed an income tax return, the three-year prescriptive period should be counted
from the time he filed such return. From February 23, 1955 when the income tax return for 1954
was filed, to February 24, 1958, when the warrant of distraint and levy was issued, 3 years and
2 days elapsed. The right of the Commissioner to issue said warrant of distraint and levy having
lapsed by two days, the warrant issued is null and void.
The above finding has made academic the question of whether or not the warrant of distraint
and levy can be enforced against the taxpayer's real property without first exhausting his
personal properties.
In resume the tax liability of Lino Gutierrez for 1951, 1952, 1953 and 1954 may be computed as
follows:

1951
Net income per investigation
Add: Disallowed deductions for salary of
driver and car expenses

P29,471.81
29.90
P29,501.81

Less: Allowable deductions:


Expenses in attending
National
Convention of Filipino
Businessmen
P 121.35
Repair of rental apartments
802.65

924.00

Net income
Less: Personal exemption

P30,425.71
3,600.00

Amount subject to tax

P26,825.71

Tax due thereon


Less tax already paid
Deficiency income tax due

P 5,668.00
3,981.00

111

P 1,687.00
==========
1952
Net income per investigation
Add: Disallowed deductions:
Salary of driver
Car expenses
Car depreciation

P21,632.22
P 260.67
401.51
65.00

727.18
P22,359.40

Less Allowable deduction:


Luncheon, Homeowners' Association

5.50

Net income
Less: Personal exemption

P22,364.90
3,600.00

Amount subject to tax

P18,764.90

Tax due thereon


Less tax already paid

P 3,324.00
2,476.00

Deficiency income tax due


1953
Net income per investigation
Add: Disallowed deductions:
Salary of driver
Car expenses
Car depreciation

848.00
==========
P69,180.91
P 140.00
406.00
58.50

604.50
P69,785.40

Less: Allowable deduction:


Cruise to Corregidor with
Homeowners'
Association

42.00

Net Income
Less: Personal exemption

P69,828 40
3,600.00

Amount subject to tax

P66,228.40

Tax due thereon


Less tax already paid

P15,179.00
9,805.00

Deficiency income tax due


1954
Net income per investigation
Add: Disallowed deductions:

P 5,374.00
==========
P43,881.92

112

Salary of driver
Car expenses
Car depreciation

P 140.00
414.18
72.65

626.83
P44,508.75

Less: Allowable deductions:


Furniture given in
connection with business
transaction
P 115.00
Repairs of rental apartments 2,048.56

2,163.56

Net income
Less: Personal exemption

P42,345.19
3,000.00

Amount subject to tax

P39,345.19

Tax due thereon


Less tax already paid

P 9,984.00
5,964.00

Deficiency income tax due

P 4,020.00
==========

SUMMARY
1951 . . . . . . . . . . . . . . . .

P 1,687.00

1952 . . . . . . . . . . . . . . . .

848.00

1953 . . . . . . . . . . . . . . . .

5,374.00

1954 . . . . . . . . . . . . . . . .

4,020.00
TOTAL . . . . . . . . . .

P 11,929.00
=========

WHEREFORE, the decision appealed from is modified and Lino Gutierrez and/or his heirs,
namely, Andrea C. Vda. de Gutierrez, Antonio D. Gutierrez, Santiago D. Gutierrez, Guillermo D.
Gutierrez and Tomas D. Gutierrez, are ordered to pay the sums of P1,687.00, P848.00,
P5,374.00, and P4,020.00, as deficiency income tax for the years 1951, 1952, 1953 and 1954,
respectively, or a total of P11,929.00, plus the statutory penalties in case of delinquency. No
costs. So ordered.

Republic of the Philippines


SUPREME COURT
Manila
113

EN BANC
G.R. No. L-21520

December 11, 1967

PLARIDEL SURETY and INSURANCE COMPANY, petitioner,


vs.
COMMISSIONER OF INTERNAL REVENUE, respondent.
Gil R. Carlos and Associates for petitioner.
Office of the Solicitor General for respondent.
BENGZON, J.P., J.:
Petitioner Plaridel Surety & Insurance Co., is a domestic corporation engaged in the bonding
business. On November 9, 1950, petitioner, as surety, and Constancio San Jose, as principal,
solidarily executed a performance bond in the penal sum of P30,600.00 in favor of the P. L.
Galang Machinery Co., Inc., to secure the performance of San Jose's contractual obligation to
produce and supply logs to the latter.
To afford itself adequate protection against loss or damage on the performance bond, petitioner
required San Jose and one Ramon Cuervo to execute an indemnity agreement obligating
themselves, solidarily, to indemnify petitioner for whatever liability it may incur by reason of said
performance bond. Accordingly, San Jose constituted a chattel mortgage on logging
machineries and other movables in petitioner's favor1 while Ramon Cuervo executed a real
estate mortgage.2
San Jose later failed to deliver the logs to Galang Machinery 3 and the latter sued on the
performance bond. On October 1, 1952, the Court of First Instance adjudged San Jose and
petitioner liable; it also directed San Jose and Cuervo to reimburse petitioner for whatever
amount it would pay Galang Machinery. The Court of Appeals, on June 17, 1955, affirmed the
judgment of the lower court. The same judgment was likewise affirmed by this Court 4 on
January 11, 1957 except for a slight modification apropos the award of attorney's fees.
On February 19 and March 20, 1957, petitioner effected payment in favor of Galang Machinery
in the total sum of P44,490.00 pursuant to the final decision.
In its income tax return for the year 1957, petitioner claimed the said amount of P44,490.00 as
deductible loss from its gross income and, accordingly, paid the amount of P136.00 as its
income tax for 1957.
The Commissioner of Internal Revenue disallowed the claimed deduction of P44,490.00 and
assessed against petitioner the sum of P8,898.00, plus interest, as deficiency income tax for the
year 1957. Petitioner filed its protest which was denied. Whereupon, appeal was taken to the
Tax Court, petitioner insisting that the P44,490.00 which it paid to Galang Machinery was a
deductible loss.
The Tax Court dismissed the appeal, ruling that petitioner was duly compensated for otherwise
than by insurance thru the mortgages in its favor executed by San Jose and Cuervo and it
had not yet exhausted all its available remedies, especially as against Cuervo, to minimize its
loss. When its motion to reconsider was denied, petitioner elevated the present appeal.

114

Of the sum of P44,490.00, the amount of P30,600.00 which is the principal sum stipulated in
the performance bond is being claimed as loss deduction under Sec. 30 (d) (2) of the Tax
Code and P10,000.00 which is the interest that had accrued on the principal sum is now
being claimed as interest deduction under Sec. 30 (b) (1).
Loss is deductible only in the taxable year it actually happens or is sustained. However, if it is
compensable by insurance or otherwise, deduction for the loss suffered is postponed to a
subsequent year, which, to be precise, is that year in which it appears that no compensation at
all can be had, or that there is a remaining or net loss, i.e., no full compensation.5
There is no question that the year in which the petitioner Insurance Co. effected payment to
Galang Machinery pursuant to a final decision occurred in 1957. However, under the same court
decision, San Jose and Cuervo were obligated to reimburse petitioner for whatever payments it
would make to Galang Machinery. Clearly, petitioner's loss is compensable otherwise (than by
insurance).itc-alf It should follow, then, that the loss deduction can not be claimed in 1957.
Now, petitioner's submission is that its case is an exception. Citing Cu Unjieng Sons, Inc. v.
Board of Tax Appeals,6 and American cases also, petitioner argues that even if there is a right to
compensation by insurance or otherwise, the deduction can be taken in the year of actual
loss where the possibility of recovery is remote. The pronouncement, however to this effect in
the Cu Unjieng case is not as authoritative as petitioner would have it since it was there found
that the taxpayer had no legal right to compensation either by insurance or otherwise.7And the
American cases cited8 are not in point. None of them involved a taxpayer who had, as in the
present case, obtained a final judgment against third persons for reimbursement of payments
made. In those cases, there was either no legally enforceable right at all or such claimed right
was still to be, or being, litigated.
On the other hand, the rule is that loss deduction will be denied if there is a measurable right to
compensation for the loss, with ultimate collection reasonably clear. So where there is
reasonable ground for reimbursement, the taxpayer must seek his redress and may not secure
a loss deduction until he establishes that no recovery may be had.9 In other words, as the Tax
Court put it, the taxpayer (petitioner) must exhaust his remedies first to recover or reduce his
loss.
It is on record that petitioner had not exhausted its remedies, especially against Ramon Cuervo
who was solidarily liable with San Jose for reimbursement to it. Upon being prodded by the Tax
Court to go after Cuervo, Hermogenes Dimaguiba, president of petitioner corporation, said that
they would10 but no evidence was submitted that anything was really done on the matter.
Moreover, petitioner's evidence on remote possibility of recovery is fatally wanting. Its right to
reimbursement is not only secured by the mortgages executed by San Jose and Cuervo but
also by a final and executory judgment in the civil case itself. Thus, other properties of San Jose
and Cuervo were subject to levy and execution. But no writ of execution, satisfied or unsatisfied,
was ever submitted. Neither has it been established that Cuervo was insolvent. The only
evidence on record on the point is Dimaguiba's testimony that he does not really know if Cuervo
has other properties.11 This is not substantial proof of insolvency.itc-alf Thus, it was too
premature for petitioner to claim a loss deduction.
But assuming that there was no reasonable expectation of recovery, still no loss deduction can
be had. Sec. 30 (d) (2) of the Tax Code requires a charge-off as one of the conditions for loss
deduction:
In the case of a corporation, all losses actually sustained and charged-off within the
taxable year and not compensated for by insurance or otherwise. (Emphasis supplied)
115

Mertens12 states only four (4) requisites because the United States Internal Revenue Code of
193913 has no charge-off requirement. itc-alf Sec. 23(f) thereof provides merely:
In the case of a corporation, losses sustained during the taxable year and not
compensated for by insurance or otherwise.
Petitioner, who had the burden of proof14 failed to adduce evidence that there was a charge-off
in connection with the P44,490.00or P30,600.00 which it paid to Galang Machinery.
In connection with the claimed interest deduction of P10,000.00, the Solicitor General correctly
points out that this question was never raised before the Tax Court. Petitioner, thru counsel, had
admitted before said court15 and in the memorandum it filed16 that the only issue in the case was
whether the entire P44,490.00 paid by it was or was not a deductible loss under Sec. 30 (d) (2)
of the Tax Code. Even in petitioner's return, the P44,490.00 was claimed wholly as losses on its
bond.17 The alleged interest deduction not having been properly litigated as an issue before the
Tax Court, it is now too late to raise and assert it before this Court.
WHEREFORE, the appealed decision is, as it is hereby, affirmed. Costs against petitioner
Plaridel Surety & Insurance Co. So ordered.
Concepcion, C.J., Reyes, J.B.L., Dizon, Makalintal, Zaldivar, Sanchez, Castro, Angeles and
Fernando, JJ.,concur.

Republic of the Philippines


SUPREME COURT
Manila
116

EN BANC
G.R. No. L-21551

September 30, 1969

FERNANDEZ HERMANOS, INC., petitioner,


vs.
COMMISSIONER OF INTERNAL REVENUE and COURT OF TAX APPEALS, respondents.
----------------------------G.R. No. L-21557

September 30, 1969

COMMISSIONER OF INTERNAL REVENUE, petitioner,


vs.
FERNANDEZ HERMANOS, INC., and COURT OF TAX APPEALS, respondents.
----------------------------G.R. No. L-24972

September 30, 1969

COMMISSIONER OF INTERNAL REVENUE, petitioner,


vs.
FERNANDEZ HERMANOS INC., and the COURT OF TAX APPEALS, respondents.
----------------------------G.R. No. L-24978

September 30, 1969

FERNANDEZ HERMANOS, INC., petitioner,


vs.
THE COMMISSIONER OF INTERNAL REVENUE, and HON. ROMAN A. UMALI, COURT OF
TAX APPEALS,respondents.
L-21551:
Rafael Dinglasan for petitioner.
Office of the Solicitor General Arturo A. Alafriz, Solicitor Alejandro B. Afurong and Special
Attorney Virgilio G. Saldajeno for respondent.
L-21557:
Office of the Solicitor General for petitioner.
Rafael Dinglasan for respondent Fernandez Hermanos, Inc.
L-24972:
Office of the Solicitor General Antonio P. Barredo, Assistant Solicitor General Felicisimo R.
Rosete and Special Attorney Virgilio G. Saldajeno for petitioner.
Rafael Dinglasan for respondent Fernandez Hermanos, Inc.
L-24978:
117

Rafael Dinglasan for petitioner.


Office of the Solicitor General Antonio P. Barredo, Assistant Solicitor General Antonio G. Ibarra
and Special Attorney Virgilio G. Saldajeno for respondent.
TEEHANKEE, J.:
These four appears involve two decisions of the Court of Tax Appeals determining the
taxpayer's income tax liability for the years 1950 to 1954 and for the year 1957. Both the
taxpayer and the Commissioner of Internal Revenue, as petitioner and respondent in the cases
a quo respectively, appealed from the Tax Court's decisions, insofar as their respective
contentions on particular tax items were therein resolved against them. Since the issues raised
are interrelated, the Court resolves the four appeals in this joint decision.
Cases L-21551 and L-21557
The taxpayer, Fernandez Hermanos, Inc., is a domestic corporation organized for the principal
purpose of engaging in business as an "investment company" with main office at Manila. Upon
verification of the taxpayer's income tax returns for the period in question, the Commissioner of
Internal Revenue assessed against the taxpayer the sums of P13,414.00, P119,613.00,
P11,698.00, P6,887.00 and P14,451.00 as alleged deficiency income taxes for the years 1950,
1951, 1952, 1953 and 1954, respectively. Said assessments were the result of alleged
discrepancies found upon the examination and verification of the taxpayer's income tax returns
for the said years, summarized by the Tax Court in its decision of June 10, 1963 in CTA Case
No. 787, as follows:
1. Losses
a. Losses in Mati Lumber Co. (1950)

P 8,050.00

b. Losses in or bad debts of Palawan Manganese Mines, Inc. (1951)


353,134.25
c. Losses in Balamban Coal Mines
1950
1951

8,989.76
27,732.66

d. Losses in Hacienda Dalupiri


1950
1951
1952
1953
1954

17,418.95
29,125.82
26,744.81
21,932.62
42,938.56

e. Losses in Hacienda Samal


1951
1952

8,380.25
7,621.73

2. Excessive depreciation of Houses


118

1950
1951
1952
1953
1954

P 8,180.40
8,768.11
18,002.16
13,655.25
29,314.98

3. Taxable increase in net worth


1950
1951

P 30,050.00
1,382.85

4. Gain realized from sale of real property in 1950

P 11,147.2611

The Tax Court sustained the Commissioner's disallowances of Item 1, sub-items (b) and
(e) and Item 2 of the above summary, but overruled the Commissioner's disallowances
of all the remaining items. It therefore modified the deficiency assessments accordingly,
found the total deficiency income taxes due from the taxpayer for the years under review
to amount to P123,436.00 instead of P166,063.00 as originally assessed by the
Commissioner, and rendered the following judgment:
RESUME
1950
1951
1952
1953
1954

P2,748.00
108,724.00
3,600.00
2,501.00
5,863.00

Total

P123,436.00

WHEREFORE, the decision appealed from is hereby modified, and petitioner is ordered
to pay the sum of P123,436.00 within 30 days from the date this decision becomes final.
If the said amount, or any part thereof, is not paid within said period, there shall be
added to the unpaid amount as surcharge of 5%, plus interest as provided in Section 51
of the National Internal Revenue Code, as amended. With costs against petitioner. (Pp.
75, 76, Taxpayer's Brief as appellant)
Both parties have appealed from the respective adverse rulings against them in the Tax Court's
decision. Two main issues are raised by the parties: first, the correctness of the Tax Court's
rulings with respect to the disputed items of disallowances enumerated in the Tax Court's
summary reproduced above, and second, whether or not the government's right to collect the
deficiency income taxes in question has already prescribed.
On the first issue, we will discuss the disputed items of disallowances seriatim.
1. Re allowances/disallowances of losses.
(a) Allowance of losses in Mati Lumber Co. (1950). The Commissioner of Internal Revenue
questions the Tax Court's allowance of the taxpayer's writing off as worthless securities in its
1950 return the sum of P8,050.00 representing the cost of shares of stock of Mati Lumber Co.
acquired by the taxpayer on January 1, 1948, on the ground that the worthlessness of said
119

stock in the year 1950 had not been clearly established. The Commissioner contends that
although the said Company was no longer in operation in 1950, it still had its sawmill and
equipment which must be of considerable value. The Court, however, found that "the company
ceased operations in 1949 when its Manager and owner, a certain Mr. Rocamora, left for
Spain ,where he subsequently died. When the company eased to operate, it had no assets, in
other words, completely insolvent. This information as to the insolvency of the Company
reached (the taxpayer) in 1950," when it properly claimed the loss as a deduction in its 1950 tax
return, pursuant to Section 30(d) (4) (b) or Section 30 (e) (3) of the National Internal Revenue
Code. 2
We find no reason to disturb this finding of the Tax Court. There was adequate basis for the
writing off of the stock as worthless securities. Assuming that the Company would later
somehow realize some proceeds from its sawmill and equipment, which were still existing as
claimed by the Commissioner, and that such proceeds would later be distributed to its
stockholders such as the taxpayer, the amount so received by the taxpayer would then properly
be reportable as income of the taxpayer in the year it is received.
(b) Disallowance of losses in or bad debts of Palawan Manganese Mines, Inc. (1951). The
taxpayer appeals from the Tax Court's disallowance of its writing off in 1951 as a loss or bad
debt the sum of P353,134.25, which it had advanced or loaned to Palawan Manganese Mines,
Inc. The Tax Court's findings on this item follow:
Sometime in 1945, Palawan Manganese Mines, Inc., the controlling stockholders of
which are also the controlling stockholders of petitioner corporation, requested financial
help from petitioner to enable it to resume it mining operations in Coron, Palawan. The
request for financial assistance was readily and unanimously approved by the Board of
Directors of petitioner, and thereafter a memorandum agreement was executed on
August 12, 1945, embodying the terms and conditions under which the financial
assistance was to be extended, the pertinent provisions of which are as follows:
"WHEREAS, the FIRST PARTY, by virtue of its resolution adopted on August 10,
1945, has agreed to extend to the SECOND PARTY the requested financial help
by way of accommodation advances and for this purpose has authorized its
President, Mr. Ramon J. Fernandez to cause the release of funds to the
SECOND PARTY.
"WHEREAS, to compensate the FIRST PARTY for the advances that it has
agreed to extend to the SECOND PARTY, the latter has agreed to pay to the
former fifteen per centum (15%) of its net profits.
"NOW THEREFORE, for and in consideration of the above premises, the parties
hereto have agreed and covenanted that in consideration of the financial help to
be extended by the FIRST PARTY to the SECOND PARTY to enable the latter to
resume its mining operations in Coron, Palawan, the SECOND PARTY has
agreed and undertaken as it hereby agrees and undertakes to pay to the FIRST
PARTY fifteen per centum (15%) of its net profits." (Exh. H-2)
Pursuant to the agreement mentioned above, petitioner gave to Palawan Manganese Mines,
Inc. yearly advances starting from 1945, which advances amounted to P587,308.07 by the end
of 1951. Despite these advances and the resumption of operations by Palawan Manganese
Mines, Inc., it continued to suffer losses. By 1951, petitioner became convinced that those
advances could no longer be recovered. While it continued to give advances, it decided to write
off as worthless the sum of P353,134.25. This amount "was arrived at on the basis of the total of
120

advances made from 1945 to 1949 in the sum of P438,981.39, from which amount the sum of
P85,647.14 had to be deducted, the latter sum representing its pre-war assets. (t.s.n., pp. 136139, Id)." (Page 4, Memorandum for Petitioner.) Petitioner decided to maintain the advances
given in 1950 and 1951 in the hope that it might be able to recover the same, as in fact it
continued to give advances up to 1952. From these facts, and as admitted by petitioner itself,
Palawan Manganese Mines, Inc., was still in operation when the advances corresponding to the
years 1945 to 1949 were written off the books of petitioner. Under the circumstances, was the
sum of P353,134.25 properly claimed by petitioner as deduction in its income tax return for
1951, either as losses or bad debts?
It will be noted that in giving advances to Palawan Manganese Mine Inc., petitioner did not
expect to be repaid. It is true that some testimonial evidence was presented to show that there
was some agreement that the advances would be repaid, but no documentary evidence was
presented to this effect. The memorandum agreement signed by the parties appears to be very
clear that the consideration for the advances made by petitioner was 15% of the net profits of
Palawan Manganese Mines, Inc. In other words, if there were no earnings or profits, there was
no obligation to repay those advances. It has been held that the voluntary advances made
without expectation of repayment do not result in deductible losses. 1955 PH Fed. Taxes, Par.
13, 329, citing W. F. Young, Inc. v. Comm., 120 F 2d. 159, 27 AFTR 395; George B. Markle, 17
TC. 1593.
Is the said amount deductible as a bad debt? As already stated, petitioner gave advances to
Palawan Manganese Mines, Inc., without expectation of repayment. Petitioner could not sue for
recovery under the memorandum agreement because the obligation of Palawan Manganese
Mines, Inc. was to pay petitioner 15% of its net profits, not the advances. No bad debt could
arise where there is no valid and subsisting debt.
Again, assuming that in this case there was a valid and subsisting debt and that the debtor was
incapable of paying the debt in 1951, when petitioner wrote off the advances and deducted the
amount in its return for said year, yet the debt is not deductible in 1951 as a worthless debt. It
appears that the debtor was still in operation in 1951 and 1952, as petitioner continued to give
advances in those years. It has been held that if the debtor corporation, although losing money
or insolvent, was still operating at the end of the taxable year, the debt is not considered
worthless and therefore not deductible. 3
The Tax Court's disallowance of the write-off was proper. The Solicitor General has rightly
pointed out that the taxpayer has taken an "ambiguous position " and "has not definitely taken a
stand on whether the amount involved is claimed as losses or as bad debts but insists that it is
either a loss or a bad debt." 4 We sustain the government's position that the advances made by
the taxpayer to its 100% subsidiary, Palawan Manganese Mines, Inc. amounting to P587,308,07
as of 1951 were investments and not loans. 5 The evidence on record shows that the board of
directors of the two companies since August, 1945, were identical and that the only capital of
Palawan Manganese Mines, Inc. is the amount of P100,000.00 entered in the taxpayer's
balance sheet as its investment in its subsidiary company. 6 This fact explains the liberality with
which the taxpayer made such large advances to the subsidiary, despite the latter's admittedly
poor financial condition.
The taxpayer's contention that its advances were loans to its subsidiary as against the Tax
Court's finding that under their memorandum agreement, the taxpayer did not expect to be
repaid, since if the subsidiary had no earnings, there was no obligation to repay those
advances, becomes immaterial, in the light of our resolution of the question. The Tax Court
correctly held that the subsidiary company was still in operation in 1951 and 1952 and the
taxpayer continued to give it advances in those years, and, therefore, the alleged debt or
121

investment could not properly be considered worthless and deductible in 1951, as claimed by
the taxpayer. Furthermore, neither under Section 30 (d) (2) of our Tax Code providing for
deduction by corporations of losses actually sustained and charged off during the taxable year
nor under Section 30 (e) (1) thereof providing for deduction of bad debts actually ascertained to
be worthless and charged off within the taxable year, can there be a partial writing off of a loss
or bad debt, as was sought to be done here by the taxpayer. For such losses or bad debts must
be ascertained to be so and written off during the taxable year, are therefore deductible in full or
not at all, in the absence of any express provision in the Tax Code authorizing partial
deductions.
The Tax Court held that the taxpayer's loss of its investment in its subsidiary could not be
deducted for the year 1951, as the subsidiary was still in operation in 1951 and 1952. The
taxpayer, on the other hand, claims that its advances were irretrievably lost because of the
staggering losses suffered by its subsidiary in 1951 and that its advances after 1949 were "only
limited to the purpose of salvaging whatever ore was already available, and for the purpose of
paying the wages of the laborers who needed help." 7 The correctness of the Tax Court's ruling
in sustaining the disallowance of the write-off in 1951 of the taxpayer's claimed losses is borne
out by subsequent events shown in Cases L-24972 and L-24978 involving the taxpayer's 1957
income tax liability. (Infra, paragraph 6.) It will there be seen that by 1956, the obligation of the
taxpayer's subsidiary to it had been reduced from P587,398.97 in 1951 to P442,885.23 in 1956,
and that it was only on January 1, 1956 that the subsidiary decided to cease operations. 8
(c) Disallowance of losses in Balamban Coal Mines (1950 and 1951). The Court sustains the
Tax Court's disallowance of the sums of P8,989.76 and P27,732.66 spent by the taxpayer for
the operation of its Balamban coal mines in Cebu in 1950 and 1951, respectively, and claimed
as losses in the taxpayer's returns for said years. The Tax Court correctly held that the losses
"are deductible in 1952, when the mines were abandoned, and not in 1950 and 1951, when they
were still in operation." 9 The taxpayer's claim that these expeditions should be allowed as
losses for the corresponding years that they were incurred, because it made no sales of coal
during said years, since the promised road or outlet through which the coal could be transported
from the mines to the provincial road was not constructed, cannot be sustained. Some definite
event must fix the time when the loss is sustained, and here it was the event of actual
abandonment of the mines in 1952. The Tax Court held that the losses, totalling P36,722.42
were properly deductible in 1952, but the appealed judgment does not show that the taxpayer
was credited therefor in the determination of its tax liability for said year. This additional
deduction of P36,722.42 from the taxpayer's taxable income in 1952 would result in the
elimination of the deficiency tax liability for said year in the sum of P3,600.00 as determined by
the Tax Court in the appealed judgment.
(d) and (e) Allowance of losses in Hacienda Dalupiri (1950 to 1954) and Hacienda Samal (19511952). The Tax Court overruled the Commissioner's disallowance of these items of losses
thus:
Petitioner deducted losses in the operation of its Hacienda Dalupiri the sums of
P17,418.95 in 1950, P29,125.82 in 1951, P26,744.81 in 1952, P21,932.62 in 1953, and
P42,938.56 in 1954. These deductions were disallowed by respondent on the ground
that the farm was operated solely for pleasure or as a hobby and not for profit. This
conclusion is based on the fact that the farm was operated continuously at a
loss.1awphl.nt
From the evidence, we are convinced that the Hacienda Dalupiri was operated by
petitioner for business and not pleasure. It was mainly a cattle farm, although a few race
horses were also raised. It does not appear that the farm was used by petitioner for
122

entertainment, social activities, or other non-business purposes. Therefore, it is entitled


to deduct expenses and losses in connection with the operation of said farm. (See 1955
PH Fed. Taxes, Par. 13, 63, citing G.C.M. 21103, CB 1939-1, p.164)
Section 100 of Revenue Regulations No. 2, otherwise known as the Income Tax
Regulations, authorizes farmers to determine their gross income on the basis of
inventories. Said regulations provide:
"If gross income is ascertained by inventories, no deduction can be made for
livestock or products lost during the year, whether purchased for resale,
produced on the farm, as such losses will be reflected in the inventory by
reducing the amount of livestock or products on hand at the close of the year."
Evidently, petitioner determined its income or losses in the operation of said farm on the
basis of inventories. We quote from the memorandum of counsel for petitioner:
"The Taxpayer deducted from its income tax returns for the years from 1950 to
1954 inclusive, the corresponding yearly losses sustained in the operation of
Hacienda Dalupiri, which losses represent the excess of its yearly expenditures
over the receipts; that is, the losses represent the difference between the sales of
livestock and the actual cash disbursements or expenses." (Pages 21-22,
Memorandum for Petitioner.)
As the Hacienda Dalupiri was operated by petitioner for business and since it sustained
losses in its operation, which losses were determined by means of inventories
authorized under Section 100 of Revenue Regulations No. 2, it was error for respondent
to have disallowed the deduction of said losses. The same is true with respect to loss
sustained in the operation of the Hacienda Samal for the years 1951 and 1952. 10
The Commissioner questions that the losses sustained by the taxpayer were properly based on
the inventory method of accounting. He concedes, however, "that the regulations referred to
does not specify how the inventories are to be made. The Tax Court, however, felt satisfied with
the evidence presented by the taxpayer ... which merely consisted of an alleged physical count
of the number of the livestock in Hacienda Dalupiri for the years involved." 11 The Tax Court was
satisfied with the method adopted by the taxpayer as a farmer breeding livestock, reporting on
the basis of receipts and disbursements. We find no Compelling reason to disturb its findings.
2. Disallowance of excessive depreciation of buildings (1950-1954). During the years 1950 to
1954, the taxpayer claimed a depreciation allowance for its buildings at the annual rate of 10%.
The Commissioner claimed that the reasonable depreciation rate is only 3% per annum, and,
hence, disallowed as excessive the amount claimed as depreciation allowance in excess of 3%
annually. We sustain the Tax Court's finding that the taxpayer did not submit adequate proof of
the correctness of the taxpayer's claim that the depreciable assets or buildings in question had
a useful life only of 10 years so as to justify its 10% depreciation per annum claim, such finding
being supported by the record. The taxpayer's contention that it has many zero or one-peso
assets, 12representing very old and fully depreciated assets serves but to support the
Commissioner's position that a 10% annual depreciation rate was excessive.
3. Taxable increase in net worth (1950-1951). The Tax Court set aside the Commissioner's
treatment as taxable income of certain increases in the taxpayer's net worth. It found that:

123

For the year 1950, respondent determined that petitioner had an increase in net worth in
the sum of P30,050.00, and for the year 1951, the sum of P1,382.85. These amounts
were treated by respondent as taxable income of petitioner for said years.
It appears that petitioner had an account with the Manila Insurance Company, the
records bearing on which were lost. When its records were reconstituted the amount of
P349,800.00 was set up as its liability to the Manila Insurance Company. It was
discovered later that the correct liability was only 319,750.00, or a difference of
P30,050.00, so that the records were adjusted so as to show the correct liability. The
correction or adjustment was made in 1950. Respondent contends that the reduction of
petitioner's liability to Manila Insurance Company resulted in the increase of petitioner's
net worth to the extent of P30,050.00 which is taxable. This is erroneous. The principle
underlying the taxability of an increase in the net worth of a taxpayer rests on the theory
that such an increase in net worth, if unreported and not explained by the taxpayer,
comes from income derived from a taxable source. (See Perez v. Araneta, G.R. No. L9193, May 29, 1957; Coll. vs. Reyes, G.R. Nos. L- 11534 & L-11558, Nov. 25, 1958.) In
this case, the increase in the net worth of petitioner for 1950 to the extent of P30,050.00
was not the result of the receipt by it of taxable income. It was merely the outcome of the
correction of an error in the entry in its books relating to its indebtedness to the Manila
Insurance Company. The Income Tax Law imposes a tax on income; it does not tax any
or every increase in net worth whether or not derived from income. Surely, the said sum
of P30,050.00 was not income to petitioner, and it was error for respondent to assess a
deficiency income tax on said amount.
The same holds true in the case of the alleged increase in net worth of petitioner for the year
1951 in the sum of P1,382.85. It appears that certain items (all amounting to P1,382.85)
remained in petitioner's books as outstanding liabilities of trade creditors. These accounts were
discovered in 1951 as having been paid in prior years, so that the necessary adjustments were
made to correct the errors. If there was an increase in net worth of the petitioner, the increase in
net worth was not the result of receipt by petitioner of taxable income." 13 The Commissioner
advances no valid grounds in his brief for contesting the Tax Court's findings. Certainly, these
increases in the taxpayer's net worth were not taxable increases in net worth, as they were not
the result of the receipt by it of unreported or unexplained taxable income, but were shown to be
merely the result of the correction of errors in its entries in its books relating to its
indebtednesses to certain creditors, which had been erroneously overstated or listed as
outstanding when they had in fact been duly paid. The Tax Court's action must be affirmed.
4. Gain realized from sale of real property (1950). We likewise sustain as being in
accordance with the evidence the Tax Court's reversal of the Commissioner's assessment on all
alleged unreported gain in the sum of P11,147.26 in the sale of a certain real property of the
taxpayer in 1950. As found by the Tax Court, the evidence shows that this property was
acquired in 1926 for P11,852.74, and was sold in 1950 for P60,000.00, apparently, resulting in a
gain of P48,147.26. 14 The taxpayer reported in its return a gain of P37,000.00, or a discrepancy
of P11,147.26. 15 It was sufficiently proved from the taxpayer's books that after acquiring the
property, the taxpayer had made improvements totalling P11,147.26, 16 accounting for the
apparent discrepancy in the reported gain. In other words, this figure added to the original
acquisition cost of P11,852.74 results in a total cost of P23,000.00, and the gain derived from
the sale of the property for P60,000.00 was correctly reported by the taxpayer at P37,000.00.
On the second issue of prescription, the taxpayer's contention that the Commissioner's action to
recover its tax liability should be deemed to have prescribed for failure on the part of the
Commissioner to file a complaint for collection against it in an appropriate civil action, as
contradistinguished from the answer filed by the Commissioner to its petition for review of the
124

questioned assessments in the case a quo has long been rejected by this Court. This Court has
consistently held that "a judicial action for the collection of a tax is begun by the filing of a
complaint with the proper court of first instance, or where the assessment is appealed to the
Court of Tax Appeals, by filing an answer to the taxpayer's petition for review wherein payment
of the tax is prayed for." 17 This is but logical for where the taxpayer avails of the right to appeal
the tax assessment to the Court of Tax Appeals, the said Court is vested with the authority to
pronounce judgment as to the taxpayer's liability to the exclusion of any other court. In the
present case, regardless of whether the assessments were made on February 24 and 27, 1956,
as claimed by the Commissioner, or on December 27, 1955 as claimed by the taxpayer, the
government's right to collect the taxes due has clearly not prescribed, as the taxpayer's appeal
or petition for review was filed with the Tax Court on May 4, 1960, with the Commissioner filing
on May 20, 1960 his Answer with a prayer for payment of the taxes due, long before the
expiration of the five-year period to effect collection by judicial action counted from the date of
assessment.
Cases L-24972 and L-24978
These cases refer to the taxpayer's income tax liability for the year 1957. Upon examination of
its corresponding income tax return, the Commissioner assessed it for deficiency income tax in
the amount of P38,918.76, computed as follows:
Net income per return
Add: Unallowable deductions:
(1) Net loss claimed on Ha. Dalupiri
(2) Amortization of Contractual right claimed as
an expense under Mines Operations
Net income per investigation
Tax due thereon

P29,178.70

89,547.33
48,481.62
P167,297.65
38,818.00

Less: Amount already assessed


Balance
Add:
1/2% monthly interest from 6-20-59
to 6-20-62

5,836.00
P32,982.00

TOTAL AMOUNT DUE AND COLLECTIBLE

P38,918.76

5,936.76

The Tax Court overruled the Commissioner's disallowance of the taxpayer's losses in the
operation of its Hacienda Dalupiri in the sum of P89,547.33 but sustained the disallowance of
the sum of P48,481.62, which allegedly represented 1/5 of the cost of the "contractual right"
over the mines of its subsidiary, Palawan Manganese Mines, Inc. which the taxpayer had
acquired. It found the taxpayer liable for deficiency income tax for the year 1957 in the amount
of P9,696.00, instead of P32,982.00 as originally assessed, and rendered the following
judgment:
WHEREFORE, the assessment appealed from is hereby modified. Petitioner is hereby
ordered to pay to respondent the amount of P9,696.00 as deficiency income tax for the
year 1957, plus the corresponding interest provided in Section 51 of the Revenue Code.
If the deficiency tax is not paid in full within thirty (30) days from the date this decision
becomes final and executory, petitioner shall pay a surcharge of five per cent (5%) of the
unpaid amount, plus interest at the rate of one per cent (1%) a month, computed from
125

18

the date this decision becomes final until paid, provided that the maximum amount that
may be collected as interest shall not exceed the amount corresponding to a period of
three (3) years. Without pronouncement as to costs. 19
Both parties again appealed from the respective adverse rulings against them in the Tax Court's
decision.
5. Allowance of losses in Hacienda Dalupiri (1957). The Tax Court cited its previous decision
overruling the Commissioner's disallowance of losses suffered by the taxpayer in the operation
of its Hacienda Dalupiri, since it was convinced that the hacienda was operated for business
and not for pleasure. And in this appeal, the Commissioner cites his arguments in his appellant's
brief in Case No. L-21557. The Tax Court, in setting aside the Commissioner's principal
objections, which were directed to the accounting method used by the taxpayer found that:
It is true that petitioner followed the cash basis method of reporting income and
expenses in the operation of the Hacienda Dalupiri and used the accrual method with
respect to its mine operations. This method of accounting, otherwise known as the
hybrid method, followed by petitioner is not without justification.
... A taxpayer may not, ordinarily, combine the cash and accrual bases. The 1954
Code provisions permit, however, the use of a hybrid method of accounting,
combining a cash and accrual method, under circumstances and requirements to
be set out in Regulations to be issued. Also, if a taxpayer is engaged in more
than one trade or business he may use a different method of accounting for each
trade or business. And a taxpayer may report income from a business on accrual
basis and his personal income on the cash basis.' (See Mertens, Law of Federal
Income Taxation, Zimet & Stanley Revision, Vol. 2, Sec. 12.08, p. 26.) 20
The Tax Court, having satisfied itself with the adequacy of the taxpayer's accounting
method and procedure as properly reflecting the taxpayer's income or losses, and the
Commissioner having failed to show the contrary, we reiterate our ruling [supra,
paragraph 1 (d) and (e)] that we find no compelling reason to disturb its findings.
6. Disallowance of amortization of alleged "contractual rights." The reasons for sustaining this
disallowance are thus given by the Tax Court:
It appears that the Palawan Manganese Mines, Inc., during a special meeting of its
Board of Directors on January 19, 1956, approved a resolution, the pertinent portions of
which read as follows:
"RESOLVED, as it is hereby resolved, that the corporation's current assets
composed of ores, fuel, and oil, materials and supplies, spare parts and canteen
supplies appearing in the inventory and balance sheet of the Corporation as of
December 31, 1955, with an aggregate value of P97,636.98, contractual rights
for the operation of various mining claims in Palawan with a value of
P100,000.00, its title on various mining claims in Palawan with a value of
P142,408.10 or a total value of P340,045.02 be, as they are hereby ceded and
transferred to Fernandez Hermanos, Inc., as partial settlement of the
indebtedness of the corporation to said Fernandez Hermanos Inc. in the amount
of P442,895.23." (Exh. E, p. 17, CTA rec.)
On March 29, 1956, petitioner's corporation accepted the above offer of transfer, thus:
126

"WHEREAS, the Palawan Manganese Mines, Inc., due to its yearly substantial
losses has decided to cease operation on January 1, 1956 and in order to satisfy
at least a part of its indebtedness to the Corporation, it has proposed to transfer
its current assets in the amount of NINETY SEVEN THOUSAND SIX HUNDRED
THIRTY SIX PESOS & 98/100 (P97,636.98) as per its balance sheet as of
December 31, 1955, its contractual rights valued at ONE HUNDRED
THOUSAND PESOS (P100,000.00) and its title over various mining claims
valued at ONE HUNDRED FORTY TWO THOUSAND FOUR HUNDRED EIGHT
PESOS & 10/100 (P142,408.10) or a total evaluation of THREE HUNDRED
FORTY THOUSAND FORTY FIVE PESOS & 08/100 (P340,045.08) which shall
be applied in partial settlement of its obligation to the Corporation in the amount
of FOUR HUNDRED FORTY TWO THOUSAND EIGHT HUNDRED EIGHTY
FIVE PESOS & 23/100 (P442,885.23)," (Exh. E-1, p. 18, CTA rec.)
Petitioner determined the cost of the mines at P242,408.10 by adding the value of the
contractual rights (P100,000.00) and the value of its mining claims (P142,408.10).
Respondent disallowed the deduction on the following grounds: (1) that the Palawan
Manganese Mines, Inc. could not transfer P242,408.10 worth of assets to petitioner
because the balance sheet of the said corporation for 1955 shows that it had only
current as worth P97,636.96; and (2) that the alleged amortization of "contractual rights"
is not allowed by the Revenue Code.
The law in point is Section 30(g) (1) (B) of the Revenue Code, before its amendment by
Republic Act No. 2698, which provided in part:
"(g) Depletion of oil and gas wells and mines.:
"(1) In general. ... (B) in the case of mines, a reasonable allowance for
depletion thereof not to exceed the market value in the mine of the product
thereof, which has been mined and sold during the year for which the return and
computation are made. The allowances shall be made under rules and
regulations to be prescribed by the Secretary of Finance: Provided, That when
the allowances shall equal the capital invested, ... no further allowance shall be
made."
Assuming, arguendo, that the Palawan Manganese Mines, Inc. had assets worth
P242,408.10 which it actually transferred to the petitioner in 1956, the latter cannot just
deduct one-fifth (1/5) of said amount from its gross income for the year 1957 because
such deduction in the form of depletion charge was not sanctioned by Section 30(g) (1)
(B) of the Revenue Code, as above-quoted.
xxx

xxx

xxx

The sole basis of petitioner in claiming the amount of P48,481.62 as a deduction was the
memorandum of its mining engineer (Exh. 1, pp. 31-32, CTA rec.), who stated that the
ore reserves of the Busuange Mines (Mines transferred by the Palawan Manganese
Mines, Inc. to the petitioner) would be exhausted in five (5) years, hence, the claim for
P48,481.62 or one-fifth (1/5) of the alleged cost of the mines corresponding to the year
1957 and every year thereafter for a period of 5 years. The said memorandum merely
showed the estimated ore reserves of the mines and it probable selling price. No
evidence whatsoever was presented to show the produced mine and for how much they
were sold during the year for which the return and computation were made. This is
necessary in order to determine the amount of depletion that can be legally deducted
127

from petitioner's gross income. The method employed by petitioner in making an outright
deduction of 1/5 of the cost of the mines is not authorized under Section 30(g) (1) (B) of
the Revenue Code. Respondent's disallowance of the alleged "contractual rights"
amounting to P48,481.62 must therefore be sustained. 21
The taxpayer insists in this appeal that it could use as a method for depletion under the
pertinent provision of the Tax Code its "capital investment," representing the alleged value of its
contractual rights and titles to mining claims in the sum of P242,408.10 and thus deduct outright
one-fifth (1/5) of this "capital investment" every year. regardless of whether it had actually mined
the product and sold the products. The very authorities cited in its brief give the correct concept
of depletion charges that they "allow for the exhaustion of the capital value of the deposits by
production"; thus, "as the cost of the raw materials must be deducted from the gross income
before the net income can be determined, so the estimated cost of the reserve used up is
allowed." 22 The alleged "capital investment" method invoked by the taxpayer is not a method of
depletion, but the Tax Code provision, prior to its amendment by Section 1, of Republic Act No.
2698, which took effect on June 18, 1960, expressly provided that "when the allowances shall
equal the capital invested ... no further allowances shall be made;" in other words, the "capital
investment" was but the limitation of the amount of depletion that could be claimed. The outright
deduction by the taxpayer of 1/5 of the cost of the mines, as if it were a "straight line" rate of
depreciation, was correctly held by the Tax Court not to be authorized by the Tax Code.
ACCORDINGLY, the judgment of the Court of Tax Appeals, subject of the appeals in Cases Nos.
L-21551 and L-21557, as modified by the crediting of the losses of P36,722.42 disallowed in
1951 and 1952 to the taxpayer for the year 1953 as directed in paragraph 1 (c) of this decision,
is hereby affirmed. The judgment of the Court of Tax Appeals appealed from in Cases Nos. L24972 and L-24978 is affirmed in toto. No costs. So ordered.
Concepcion, C.J., Dizon, Makalintal, Zaldivar, Sanchez, Castro, Fernando, Capistrano and
Barredo, JJ., concur.

Republic of the Philippines


SUPREME COURT
Manila
EN BANC
G.R. No. L-22265

December 22, 1967

COLLECTOR OF INTERNAL REVENUE, petitioner,


vs.
GOODRICH INTERNATIONAL RUBBER CO., respondent.
128

Manuel O. Chan for respondent.


CONCEPCION, C.J.:
Appeal by the Government from a decision of the Court of Tax Appeals, setting aside the
assessments made by the Commissioner of Internal Revenue, in the sums of P14,128.00 and
P8,439.00, as deficiency income taxes allegedly due from respondent Goodrich International
Rubber Company hereinafter referred to as Goodrich for the years 1951 and 1952,
respectively.
These assessments were based on disallowed deductions, claimed by Goodrich, consisting of
several alleged bad debts, in the aggregate sum of P50,455.41, for the year 1951, and the sum
of P30,138.88, as representation expenses allegedly incurred in the year 1952. Goodrich had
appealed from said assessments to the Court of Tax Appeals, which, after appropriate
proceedings, rendered, on June 8, 1963, a decision allowing the deduction for bad debts, but
disallowing the alleged representation expenses. On motion for reconsideration and new trial,
filed by Goodrich, on November 19, 1963, the Court of Tax Appeals amended its
aforementioned decision and allowed said deductions for representation expenses. Hence, this
appeal by the Government.
The alleged representation expenses are:

1. Expenses at Elks Club

P10,959.21

2. Manila Polo Club

4,947.35

3. Army and Navy Club

2,812.95

4. Manila Golf Club

4,478.45

5. Wack Wack Golf Club, Casino


Espaol, etc.

6,940.92

T O TAL

P30,138.88

The claim for deduction thereof is based upon receipts issued, not by the entities in which the
alleged expenses had been incurred, but by the officers of Goodrich who allegedly paid them.

129

The claim must be rejected. If the expenses had really been incurred, receipts or chits would
have been issued by the entities to which the payments had been made, and it would have
been easy for Goodrich or its officers to produce such receipts.
These issued by said officers merely attest to their claim that they had incurred and paid said
expenses. They do not establish payment of said alleged expenses to the entities in which the
same are said to have been incurred. The Court of Tax Appeals erred, therefore, in allowing the
deduction thereof.
The alleged bad debts are:

1. Portillo's Auto Seat Cover

2. Visayan Rapid Transit

3. Bataan Auto Seat Cover

4. Tres Amigos Auto Supply

P630.31

17,810.26

373.13

1,370.31

5. P. C. Teodorolawphil

650.00

6. Ordnance Service, P.A.

386.42

7. Ordnance Service, P.C.

796.26

8. National land Settlement Administration

9. National Coconut Corporation

10. Interior Caltex Service Station

3,020.76

644.74

1,505.87

130

11. San Juan Auto Supply

4,530.64

12. P A C S A

45.36

13. Philippine Naval Patrol

14.18

14. Surplus Property Commission

277.68

15. Alverez Auto Supply

285.62

16. Lion Shoe Store

1,686.93

17. Ruiz Highway Transit

2,350.00

18. Esquire Auto Seat Cover

3,536.94

T O TAL

P50,455.41*

The issue, in connection with these debts is whether or not the same had been properly
deducted as bad debts for the year 1951. In this connection, we find:
Portillo's Auto Seat Cover (P730.00):
This debt was incurred in 1950. In 1951, the debtor paid P70.00, leaving a balance of P630.31.
That same year, the account was written off as bad debt (Exhibit 3-C-4). Counsel for Goodrich
had merely sent two (2) letters of demand in 1951 (Exh. B-14). In 1952, the debtor paid the full
balance (Exhibit A).
Visayan Rapid Transit (P17,810.26):
This debt was, also, incurred in 1950. In 1951, it was charged off as bad debt, after the debtor
had paid P275.21. No other payment had been made. Taxpayer's Accountant testified that,
according to its branch manager in Cebu, he had been unable to collect the balance. The debtor
131

had merely promised and kept on promising to pay. Taxpayer's counsel stated that the debtor
had gone out of business and became insolvent, but no proof to this effect. was introduced.
Bataan Auto Seat Cover (P373.13):
This is the balance of a debt of P474.13 contracted in 1949. In 1951, the debtor paid P100.00.
That same year, the balance of P373.13 was charged off as bad debt. The next year, the debtor
paid the additional sum of P50.00.
Tres Amigos Auto Supply (P1,370.31):
This account had been outstanding since 1949. Counsel for the taxpayer had merely sent
demand letters (Exh. B-13) without success.
P. C. Teodoro (P650.00):
In 1949, the account was P751.91. In 1951, the debtor paid P101.91, thus leaving a balance of
P650.00, which the taxpayer charged off as bad debt in the same year. In 1952, the debtor
made another payment of P150.00.
Ordinance Service, P.A. (P386.42):
In 1949, the outstanding account of this government agency was P817.55. Goodrich's counsel
sent demand letters (Exh. B-8). In 1951, it paid Goodrich P431.13. The balance of P386.42 was
written off as bad debt that same year.
Ordinance Service, P.C. (P796.26):
In 1950, the account was P796.26. It was referred to counsel for collection. In 1951, the account
was written off as a debt. In 1952, the debtor paid it in full.
National Land Settlement Administration (P3,020.76):
The outstanding account in 1949 was P7,041.51. Collection letters were sent (Exh. B-7). In
1951, the debtor paid P4,020.75, leaving a balance of P3,020.76, which was written off, that
same year, as a bad debt. This office was under liquidation, and its Board of Liquidators
promised to pay when funds shall become available.
National Coconut Corporation (P644.74):
This account had been outstanding since 1949. Collection letters were sent (Exh. B-12) without
success. It was written off as bad debt in 1951, while the corporation was under a Board of
Liquidators, which promised to pay upon availability of funds. In 1961, the debt was fully paid.
Interior Caltex Service Station (P1,505.87):
The original account was P2,705.87, when, in 1950, it was turned over for collection to counsel
for Goodrich (p. 156, CTA Records). Counsel began sending letters of collection in April 1950.
Interior Caltex made partial payments, so that as of December, 1951, the balance outstanding
was P1,505.87.lawphil.net The debtor paid P200, in 1952; P113.20, in 1954; P750.00, in 1961;
and P300.00.00 in 1962. The account had been written off as bad debt in 1951.
132

The claim for deduction of these ten (10) debts should be rejected. Goodrich has not
established either that the debts are actually worthless or that it had reasonable grounds to
believe them to be so in 1951. Our statute permits the deduction of debts "actually ascertained
to be worthless within the taxable year," obviously to prevent arbitrary action by the taxpayer, to
unduly avoid tax liability.
The requirement of ascertainment of worthlessness requires proof of two facts: (1) that the
taxpayer did in fact ascertain the debt to be worthlessness, in the year for which the deduction is
sought; and (2) that, in so doing, he acted in good faith.1
Good faith on the part of the taxpayer is not enough. He must show, also, that he had
reasonably investigated the relevant facts and had drawn a reasonable inference from the
information thus obtained by him.2 Respondent herein has not adequately made such showing.
The payments made, some in full, after some of the foregoing accounts had been characterized
as bad debts, merely stresses the undue haste with which the same had been written off. At any
rate, respondent has not proven that said debts were worthless. There is no evidence that the
debtors can not pay them.lawphil.net It should be noted also that, in violation of Revenue
Regulations No. 2, Section 102, respondent had not attached to its income tax returns a
statement showing the propriety of the deductions therein made for alleged bad debts.
Upon the other hand, we find that the following accounts were properly written off:
San Juan Auto Supply (P4,530.64):
This account was contracted in 1950. Referred, for collection, to respondent's counsel, the latter
secured no payment. In November, 1950, the corresponding suit for collection was filed (Exh.
C). The debtor's counsel was allowed to withdraw, as such, the debtor having failed to meet
him. In fact, the debtor did not appear at the hearing of the case.lawphil.net Judgment was
rendered in 1951 for the creditor (Exh. C-2). The corresponding writ of execution (Exh. C-3) was
returned unsatisfied, for no properties could be attached or levied upon.

PACSA

(P45.36),

Philippine Naval Patrol

(P14.18),

Surplus Property
Commission

(P277.68),

Alvarez Auto Supply

(P285.62):

These four (4) accounts were 2 or 3 years old in 1951. After the collectors of the creditor had
failed to collect the same, its counsel wrote letters of demand (Exhs. B-10, B-11, B-6 and B-2) to
no avail. Considering the small amounts involved in these accounts, the taxpayer was justified
133

in feeling that the unsuccessful efforts therefore exerted to collect the same sufficed to warrant
their being written off.3

Lion Shoe Store

(P11,686.93),

Ruiz Highway Transit

(P2,350.00),
and

Esquire Auto Seat Cover

(P3,536.94):

These three (3) accounts were among those referred to counsel for Goodrich for collection. Up
to 1951, when they were written off, counsel had sent 17 Letters of demand to Lion Shoe Store
(Exh. B); 16 demand letters to Ruiz Highway Transit (Exh. B-1); and 6 letters of demand to
Esquire Auto Seat Cover (Exit. B-5) In 1951, Lion Shoe Store, Ruiz Highway Transit, and
Esquire Auto Seat Cover had made partial payments in the sums of P1,050.00, P400.00, and
P300.00 respectively. Subsequent to the write-off, additional small payments were made and
accounted for as income of Goodrich. Counsel interviewed the debtors, investigated their ability
to pay and threatened law suits. He found that the debtors were in strained financial condition
and had no attachable or leviable property. Moreover, Lion Shoe Store was burned twice, in
1948 and 1949. Thereafter, it continued to do business on limited scale. Later; it went out of
business. Ruiz Highway Transit, had more debts than assets. Counsel, therefore, advised
respondent to write off these accounts as bad debts without going to court, for it would be
"foolish to spend good money after bad."
The deduction of these eight (8) accounts, aggregating P22,627.35, as bad debts should be
allowed.
WHEREFORE, the decision appealed from should be, as it is hereby, modified, in the sense
that respondent's alleged representation expenses are totally disallowed, and its claim for bad
debts allowed up to the sum of P22,627.35 only. Without special pronouncement as to costs. It
is so ordered.
Reyes, J.B.L., Dizon, Makalintal, Bengzon, JJ., Zaldivar, Sanchez, Castro, Angeles and
Fernando, JJ., concur.

134

Republic of the Philippines


SUPREME COURT
Manila
EN BANC
G.R. No. L-22492

September 5, 1967

BASILAN ESTATES, INC., petitioner,


vs.
THE COMMISSIONER OF INTERNAL REVENUE and THE COURT OF TAX
APPEALS, respondents.
Felix A. Gulfin and Antonio S. Alano for petitioner.
Office of the Solicitor General for respondents.
BENGZON, J.P., J.:
A Philippine corporation engaged in the coconut industry, Basilan Estates, Inc., with principal
offices in Basilan City, filed on March 24, 1954 its income tax returns for 1953 and paid an
income tax of P8,028. On February 26, 1959, the Commissioner of Internal Revenue, per
examiners' report of February 19, 1959, assessed Basilan Estates, Inc., a deficiency income tax
of P3,912 for 1953 and P86,876.85 as 25% surtax on unreasonably accumulated profits as of
1953 pursuant to Section 25 of the Tax Code. On non-payment of the assessed amount, a
warrant of distraint and levy was issued but the same was not executed because Basilan
Estates, Inc. succeeded in getting the Deputy Commissioner of Internal Revenue to order the
Director of the district in Zamboanga City to hold execution and maintain constructive embargo
instead. Because of its refusal to waive the period of prescription, the corporation's request for
reinvestigation was not given due course, and on December 2, 1960, notice was served the
corporation that the warrant of distraint and levy would be executed.
On December 20, 1960, Basilan Estates, Inc. filed before the Court of Tax Appeals a petition for
review of the Commissioner's assessment, alleging prescription of the period for assessment
and collection; error in disallowing claimed depreciations, travelling and miscellaneous
expenses; and error in finding the existence of unreasonably accumulated profits and the
imposition of 25% surtax thereon. On October 31, 1963, the Court of Tax Appeals found that
there was no prescription and affirmed the deficiency assessment in toto.
On February 21, 1964, the case was appealed to Us by the taxpayer, upon the following issues:
1. Has the Commissioner's right to collect deficiency income tax prescribed?
2. Was the disallowance of items claimed as deductible proper?
3. Have there been unreasonably accumulated profits? If so, should the 25% surtax be
imposed on the balance of the entire surplus from 1947-1953, or only for 1953?
4. Is the petitioner exempt from the penalty tax under Republic Act 1823 amending
Section 25 of the Tax Code?
135

PRESCRIPTION
There is no dispute that the assessment of the deficiency tax was made on February 26, 1959;
but the petitioner claims that it never received notice of such assessment or if it did, it received
the notice beyond the five-year prescriptive period. To show prescription, the annotation on the
notice (Exhibit 10, No. 52, ACR, p. 54-A of the BIR records) "No accompanying letter 11/25/" is
advanced as indicative of the fact that receipt of the notice was after March 24, 1959, the last
date of the five-year period within which to assess deficiency tax, since the original returns were
filed on March 24, 1954.
Although the evidence is not clear on this point, We cannot accept this interpretation of the
petitioner, considering the presence of circumstances that lead Us to presume regularity in the
performance of official functions. The notice of assessment shows the assessment to have been
made on February 26, 1959, well within the five-year period. On the right side of the notice is
also stamped "Feb. 26, 1959" denoting the date of release, according to Bureau of Internal
Revenue practice. The Commissioner himself in his letter (Exh. H, p. 84 of BIR records)
answering petitioner's request to lift, the warrant of distraint and levy, asserts that notice had
been sent to petitioner. In the letter of the Regional Director forwarding the case to the Chief of
the Investigation Division which the latter received on March 10, 1959 (p. 71 of the BIR records),
notice of assessment was said to have been sent to petitioner. Subsequently, the Chief of the
Investigation Division indorsed on March 18, 1959 (p. 24 of the BIR records) the case to the
Chief of the Law Division. There it was alleged that notice was already sent to petitioner on
February 26, 1959. These circumstances pointing to official performance of duty must
necessarily prevail over petitioner's contrary interpretation. Besides, even granting that notice
had been received by the petitioner late, as alleged, under Section 331 of the Tax Code
requiring five years within which to assessdeficiency taxes, the assessment is deemed made
when notice to this effect is released, mailed or sent by the Collector to the taxpayer and it is not
required that the notice be received by the taxpayer within the aforementioned five-year period.1
ASSESSMENT
The questioned assessment is as follows:
Net Income per return
Add: Over-claimed depreciation
Mis. expenses disallowed
Officer's travelling expenses
disallowed

P40,142.90
P10,500.49
6,759.17
2,300.40

Net Income per Investigation


20% tax on P59,702.96
Less: Tax already assessed
Deficiency income tax
Add: Additional tax of 25% on P347,507.01
Tax Due & Collectible

19,560.06
P59,702.96
11,940.00
8,028.00
P3,912.00
86,876.75
P90,788.75
=========

The Commissioner disallowed:


136

Over-claimed depreciation
Miscellaneous expenses
Officer's travelling expenses

P10,500.49
6,759.17
2,300.40
DEDUCTIONS

A. Depreciation. Basilan Estates, Inc. claimed deductions for the depreciation of its assets up
to 1949 on the basis of their acquisition cost. As of January 1, 1950 it changed the depreciable
value of said assets by increasing it to conform with the increase in cost for their replacement.
Accordingly, from 1950 to 1953 it deducted from gross income the value of depreciation
computed on the reappraised value.
In 1953, the year involved in this case, taxpayer claimed the following depreciation deduction:
Reappraised assets
New assets consisting of hospital building and
equipment
Total depreciation

P47,342.53
3,910.45
P51,252.98

Upon investigation and examination of taxpayer's books and papers, the Commissioner of
Internal Revenue found that the reappraised assets depreciated in 1953 were the same ones
upon which depreciation was claimed in 1952. And for the year 1952, the Commissioner had
already determined, with taxpayer's concurrence, the depreciation allowable on said assets to
be P36,842.04, computed on their acquisition cost at rates fixed by the taxpayer. Hence, the
Commissioner pegged the deductible depreciation for 1953 on the same old assets at
P36,842.04 and disallowed the excess thereof in the amount of P10,500.49.
The question for resolution therefore is whether depreciation shall be determined on the
acquisition cost or on the reappraised value of the assets.
Depreciation is the gradual diminution in the useful value of tangible property resulting from
wear and tear and normal obsolescense. The term is also applied to amortization of the value of
intangible assets, the use of which in the trade or business is definitely limited in
duration.2 Depreciation commences with the acquisition of the property and its owner is not
bound to see his property gradually waste, without making provision out of earnings for its
replacement. It is entitled to see that from earnings the value of the property invested is kept
unimpaired, so that at the end of any given term of years, the original investment remains as it
was in the beginning. It is not only the right of a company to make such a provision, but it is its
duty to its bond and stockholders, and, in the case of a public service corporation, at least, its
plain duty to the public.3 Accordingly, the law permits the taxpayer to recover gradually his
capital investment in wasting assets free from income tax. 4 Precisely, Section 30 (f) (1) which
states:
(1)In general. A reasonable allowance for deterioration of property arising out of its
use or employment in the business or trade, or out of its not being used: Provided, That
when the allowance authorized under this subsection shall equal the capital invested by
the taxpayer . . . no further allowance shall be made. . . .
allows a deduction from gross income for depreciation but limits the recovery to the capital
invested in the asset being depreciated.
137

The income tax law does not authorize the depreciation of an asset beyond its acquisition cost.
Hence, a deduction over and above such cost cannot be claimed and allowed. The reason is
that deductions from gross income are privileges,5 not matters of right.6 They are not created by
implication but upon clear expression in the law.7
Moreover, the recovery, free of income tax, of an amount more than the invested capital in an
asset will transgress the underlying purpose of a depreciation allowance. For then what the
taxpayer would recover will be, not only the acquisition cost, but also some profit. Recovery in
due time thru depreciation of investment made is the philosophy behind depreciation allowance;
the idea of profit on the investment made has never been the underlying reason for the
allowance of a deduction for depreciation.
Accordingly, the claim for depreciation beyond P36,842.04 or in the amount of P10,500.49 has
no justification in the law. The determination, therefore, of the Commissioner of Internal
Revenue disallowing said amount, affirmed by the Court of Tax Appeals, is sustained.
B. Expenses. The next item involves disallowed expenses incurred in 1953, broken as
follows:
Miscellaneous expenses
Officer's travelling expenses
Total

P6,759.17
2,300.40
P9,059.57

These were disallowed on the ground that the nature of these expenses could not be
satisfactorily explained nor could the same be supported by appropriate papers.
Felix Gulfin, petitioner's accountant, explained the P6,759.17 was actual expenses credited to
the account of the president of the corporation incurred in the interest of the corporation during
the president's trip to Manila (pp. 33-34 of TSN of Dec. 5, 1962); he stated that the P2,300.40
was the president's travelling expenses to and from Manila as to the vouchers and receipts of
these, he said the same were made but got burned during the Basilan fire on March 30, 1962
(p. 40 of same TSN). Petitioner further argues that when it sent its records to Manila in
February, 1959, the papers in support of these miscellaneous and travelling expenses were not
included for the reason that by February 9, 1959, when the Bureau of Internal Revenue decided
to investigate, petitioner had no more obligation to keep the same since five years had lapsed
from the time these expenses were incurred (p. 41 of same TSN). On this ground, the petitioner
may be sustained, for under Section 337 of the Tax Code, receipts and papers supporting such
expenses need be kept by the taxpayer for a period of five years from the last entry. At the time
of the investigation, said five years had lapsed. Taxpayer's stand on this issue is therefore
sustained.
UNREASONABLY ACCUMULATED PROFITS
Section 25 of the Tax Code which imposes a surtax on profits unreasonably accumulated,
provides:
Sec. 25. Additional tax on corporations improperly accumulating profits or surplus (a)
Imposition of tax. If any corporation, except banks, insurance companies, or personal
holding companies, whether domestic or foreign, is formed or availed of for the purpose
of preventing the imposition of the tax upon its shareholders or members or the
shareholders or members of another corporation, through the medium of permitting its
138

gains and profits to accumulate instead of being divided or distributed, there is levied
and assessed against such corporation, for each taxable year, a tax equal to twentyfive per centum of the undistributed portion of its accumulated profits or surplus which
shall be in addition to the tax imposed by section twenty-four, and shall be computed,
collected and paid in the same manner and subject to the same provisions of law,
including penalties, as that tax.1awphl.nt
The Commissioner found that in violation of the abovequoted section, petitioner had
unreasonably accumulated profits as of 1953 in the amount of P347,507.01, based on the
following circumstances (Examiner's Report pp. 62-68 of BIR records):
1. Strong financial position of the petitioner as of December 31, 1953. Assets were
P388,617.00 while the liabilities amounted to only P61,117.31 or a ratio of 6:1.
2. As of 1953, the corporation had considerable capital adequate to meet the reasonable
needs of the business amounting to P327,499.69 (assets less liabilities).
3. The P200,000 reserved for electrification of drier and mechanization and the P50,000
reserved for malaria control were reverted to its surplus in 1953.
4. Withdrawal by shareholders, of large sums of money as personal loans.
5. Investment of undistributed earnings in assets having no proximate connection with
the business as hospital building and equipment worth P59,794.72.
6. In 1953, with an increase of surplus amounting to P677,232.01, the capital stock was
increased to P500,000 although there was no need for such increase.
Petitioner tried to show that in considering the surplus, the examiner did not take into account
the possible expenses for cultivation, labor, fertilitation, drainage, irrigation, repair, etc. (pp. 235237 of TSN of Dec. 7, 1962). As aptly answered by the examiner himself, however, they were
already included as part of the working capital (pp. 237-238 of TSN of Dec. 7, 1962).
In the unreasonable accumulation of P347,507.01 are included P200,000 for electrification of
driers and mechanization and P50,000 for malaria control which were reserved way back in
1948 (p. 67 of the BIR records) but reverted to the general fund only in 1953. If there were any
plans for these amounts to be used in further expansion through projects, it did not appear in
the records as was properly indicated in 1948 when such amounts were reserved. Thus, while
in 1948 it was already clear that the money was intended to go to future projects, in 1953 upon
reversion to the general fund, no such intention was shown. Such reversion therefore gave
occasion for the Government to consider the same for tax purposes. The P250,000 reverted to
the general fund was sought to be explained as later used elsewhere: "part of it in the Hilano
Industries, Inc. in building the factory site and buildings to house technical men . . . part of it was
spent in the facilities for the waterworks system and for industrialization of the coconut industry"
(p. 117 of TSN of Dec. 6, 1962). This is not sufficient explanation. Persuasive jurisprudence on
the matter such as those in the United States from where our tax law was derived, 8has it that:
"In order to determine whether profits were accumulated for the reasonable needs of the
business or to avoid the surtax upon shareholders, the controlling intention of the taxpayer is
that which is manifested at the time of the accumulation, not subsequently declared intentions
which are merely the products of after-thought."9The reversion here was made because the
reserved amount was not enough for the projects intended, without any intent to channel the
same to some particular future projects in mind.
139

Petitioner argues that since it has P560,717.44 as its expenses for the year 1953, a surplus of
P347,507.01 is not unreasonably accumulated. As rightly contended by the Government, there
is no need to have such a large amount at the beginning of the following year because during
the year, current assets are converted into cash and with the income realized from the business
as the year goes, these expenses may well be taken care of (pp. 238 of TSN of Dec. 7, 1962).
Thus, it is erroneous to say that the taxpayer is entitled to retain enough liquid net assets in
amounts approximately equal to current operating needs for the year to cover "cost of goods
sold and operating expenses" for "it excludes proper consideration of funds generated by the
collection of notes receivable as trade accounts during the course of the year." 10 In fact, just
because the fatal accumulations are less than 70% of the annual operating expenses of the
year, it does not mean that the accumulations are reasonable as a matter of law."11
Petitioner tried to show that investments were made with Basilan Coconut Producers
Cooperative Association and Basilan Hospital (pp. 103-105 of TSN of Dec. 6, 1962) totalling
P59,794.72 as of December 31, 1953. This shows all the more the unreasonable accumulation.
As of December 31, 1953 already P59,794.72 was spent yet as of that date there was still a
surplus of P347,507.01.
Petitioner questions why the examiner covered the period from 1948-1953 when the taxable
year on review was 1953. The surplus of P347,507.01 was taken by the examiner from the
balance sheet of petitioner for 1953. To check the figure arrived at, the examiner traced the
accumulation process from 1947 until 1953, and petitioner's figure stood out to be correct. There
was no error in the process applied, for previous accumulations should be considered in
determining unreasonable accumulations for the year concerned. "In determining whether
accumulations of earnings or profits in a particular year are within the reasonable needs of a
corporation, it is neccessary to take into account prior accumulations, since accumulations prior
to the year involved may have been sufficient to cover the business needs and additional
accumulations during the year involved would not reasonably be necessary."12
Another factor that stands out to show unreasonable accumulation is the fact that large amounts
were withdrawn by or advanced to the stockholders. For the year 1953 alone these totalled
P197,229.26. Yet the surplus of P347,507.01 was left as of December 31, 1953. We find
unacceptable petitioner's explanation that these were advances made in furtherance of the
business purposes of the petitioner. As correctly held by the Court of Tax Appeals, while certain
expenses of the corporation were credited against these amounts, the unspent balance was
retained by the stockholders without refunding them to petitioner at the end of each year. These
advances were in fact indirect loans to the stockholders indicating the unreasonable
accumulation of surplus beyond the needs of the business.
ALLEGED EXEMPTION
Petitioner wishes to avail of the exempting proviso in Sec. 25 of the Internal Revenue Code as
amended by R.A. 1823, approved June 22, 1957, whereby accumulated profits or surplus if
invested in any dollar-producing or dollar-earning industry or in the purchase of bonds issued by
the Central Bank, may not be subject to the 25% surtax. We have but to point out that the
unreasonable accumulation was in 1953. The exemption was by virtue of Republic Act 1823
which amended Sec. 25 only on June 22, 1957 more than three years after the period
covered by the assessment.
In resume, Basilan Estates, Inc. is liable for the payment of deficiency income tax and surtax for
the year 1953 in the amount of P88,977.42, computed as follows:
Net Income per return

P40,142.90
140

Add: Over-claimed
depreciation

10,500.49

Net income per finding

P50,643.39

20% tax on P50,643.39


Less: Tax already assessed

P10,128.67
8,028.00

Deficiency income tax


Add: 25% surtax on
P347,507.01
Total tax due and collectible

P2,100.67
86,876.75
P88,977.42
===========

WHEREFORE, the judgment appealed from is modified to the extent that petitioner is allowed
its deductions for travelling and miscellaneous expenses, but affirmed insofar as the petitioner is
liable for P2,100.67 as deficiency income tax for 1953 and P86,876.75 as 25% surtax on the
unreasonably accumulated profit of P347,507.01. No costs. So ordered.
Concepcion, C.J., Reyes, J.B.L., Dizon, Makalintal, Zaldivar, Sanchez, Castro, Angeles and
Fernando, JJ., concur.

U.S. Supreme Court


United States v. Ludey, 274 U.S. 295 (1927)
United States v. Ludey
No. 289
141

Argued April 21, 22, 1927


Decided May 16, 1927
274 U.S. 295
CERTIORARI TO THE COURT OF CLAIMS
Syllabus
1. Under the income and excess profits provisions of the Revenue Act of 1916, as amended by
Revenue Act of 1917, in determining the existence and amount of profit realized from a sale of
oil mining properties -- land, leases, and equipment -- the cost of the property sold is the original
cost to the taxpayer (if purchased after March 1, 1913, or its value on that date if acquired
earlier for less) diminished by deductions for depreciation and depletion occurring between the
dates of purchase (or March 1, 1913) and sale.
2. The depreciation charge permitted as a deduction from the gross income in determining the
taxable income of a business for any year represents the reduction, during the year, of the
capital assets through wear and tear of the plant used.
3. When a plant is disposed of after years of use, the thing then sold is not the whole thing
originally acquired. The amount of the depreciation must be deducted from the original cost of
the whole in order to determine the cost of that disposed of in the final sale of properties.
4. This rule applies to mining as well as to mercantile business.
5. The depletion charge permitted as a deduction from the gross income in determining the
taxable income of mines for any year represents the reduction in the mineral contents of the
reserves from which the product is taken. Because the quantity originally in the reserve is not
actually known, the percentage of the whole withdrawn in any year, and hence the appropriate
depletion charge, is necessarily a rough estimate.
6. The amounts of depreciation and depletion to be deducted from cost to ascertain gain on a
sale of oil properties are equal to the aggregates of depreciation and depletion which the
taxpayer was entitled to deduct from gross income in his income tax returns for earlier years;
but are not dependent on the amounts which he actually so claimed. P. 274 U. S. 303.
61 Ct.Cls. 126 reversed.
Certiorari (271 U.S. 651) to a judgment of the Court of Claims for an amount exacted as
additional income and excess profits taxes.
MR. JUSTICE BRANDEIS delivered the opinion of the Court.
Ludey brought this suit in the Court of Claims to recover an amount exacted as additional taxes
for 1917 under the income and excess profits provisions of the Revenue Act of 1916,
September 8, 1916, c. 463, Tit. I, 39 Stat. 756-759, as amended by the Revenue Act of 1917,
October 3, 1917, c. 63, 40 Stat. 300, 329. The tax was assessed on the alleged gain from a sale
in 1917 of oil mining properties which had been owned and operated by him for several years.
The Commissioner of Internal Revenue determined that there was a gain on the sale of
$26,904.15. Ludey insists that there was a loss of $14,777.33. The amount sued for is the tax
assessed on the difference. Whether there was the gain or the loss depends primarily upon
142

whether deductions for depletion and depreciation are to be made from the original cost in
determining gain or loss on sale of oil mining properties. The question is one of statutory
construction or application. The Court of Claims entered judgment for the plaintiff. 61 Ct.Cls.
126. This Court granted a writ of certiorari.
The properties consisted, besides mining equipment, in part of oil land held in fee, in part of oil
mining leases. The aggregate original cost of the properties was $95,977.33. [Footnote 1] Of
this amount, $30,977.33 was the cost of the equipment used in the business; $65,000 the cost
of the oil reserves. The 1917 sale price was $81,200. For the purpose of determining the cost of
the properties sold in 1917 the Commissioner deducted from the original cost $10,465.16 on
account of depreciation of the equipment through wear and tear, and $32,258.81 on account of
depletion of the reserves through the taking out of oil by the plaintiff, after March 1, 1913. There
was no dispute of fact concerning the correctness of the estimates upon which these deductions
were made. The finding of the depletion was in accordance with the method of computation
employed by the Bureau of Internal Revenue, and there was no objection specifically to the
method of computation. But Ludey insisted that the amount of depletion, if any, could not be
found or stated as a fact, since, in the nature of the case, it was impossible to determine how
much oil was recoverable, either when he acquired the properties or when he disposed of them.
The finding of the depreciation was, likewise, in accordance with the method of computation
employed by the bureau, and there was no objection to the method of computation. But Ludey
insisted also in respect to depreciation that the property was, as a matter of law, unchanged in
character and quantity throughout the period of operation.
Until 1924, none of the revenue acts provided in terms that, in computing the gain from a sale of
any property, a deduction shall be made from the original cost on account of depreciation and
depletion during the period of operation. [Footnote 2] But ever since March 1, 1913, the revenue
acts have required that gains from sales made within the tax year shall be included in the
taxable income of the year, and that losses on sales may be deducted from gross income. And
each of the acts has provided that, in computing the taxable income derived from operating a
mine, there may be made a deduction from the gross income for the depreciation and that some
deduction may be made for depletion. The applicable provisions of 5(a) of the Revenue Act of
1916 concerning deductions to be allowed in computing net income are these:
"Fourth. Losses actually sustained during the year, incurred in his business or
trade: . . . Provided, that . . . the . . . value of . . . property [acquired before March 1, 1913] as of
March 1, 1913, shall be the basis for determining the amount of such loss. . . ."
"Seventh. A reasonable allowance for the exhaustion, wear and tear of property arising out of its
use or employment in the business or trade. . . ."
"Eighth. (a) In the case of oil and gas wells, a reasonable allowance for actual reduction in flow
and production; . . . (b) in the case of mines, a reasonable allowance for depletion
thereof: . . . Provided, That when the allowances . . . shall equal the capital originally invested . .
. , no further allowance shall be made."
Ludey does not deny that Congress has power to require that deductions for depreciation and
depletion shall be made from the original cost when determining the cost of oil properties sold.
His contention is that, at the time of the sale in question, Congress had not in terms required the
deductions in the case of any property, and that special reasons exist why the acts should be
construed as not requiring the deductions in the case of oil wells. He urges that a corporation
organized for the purpose of utilizing a wasting property, like an iron mine, is not deemed to
have divided a part of its capital merely because it has distributed the net proceeds of its mining
operations; that this is true even where the necessary result of the operation is a reduction of
the mineral reserve; that, a fortiori, the proceeds of oil mining are to be deemed income, not a
143

partial return of capital, since there is no ownership in oil until it is actually reduced to
possession; that a purchase of an oil reserve cannot be likened to the purchase of a certain
number of barrels of oil; that an oil reserve is not a reservoir; that Congress allowed the
deduction from gross income for depreciation and depletion probably as a reward in an extrahazardous enterprise in order to encourage new producing properties, and that to allow the
deductions would result, in the event of a sale of the property, in taking back the rewards so
offered.
The government contends that, in operating the properties, Ludey disposed, in the form of oil, of
part of his capital assets; that, in the extraction of the oil, he consumed so much of the
equipment as was represented by the depreciation, and disposed of so much of the oil reserves
as was represented by the depletion; that the sale of the properties made by him in 1917 was
not a sale of all of the property represented by the original cost of $95,977.33, since physical
equipment to the amount of the depreciation and oil reserves to the amount of the depletion had
been taken from it during the preceding years, and that, for this reason, the cost to plaintiff of
the net property sold in 1917 was not $95,977.33, but $53,258.36.
The Court of Claims did not consider whether, ordinarily, deductions for depreciation and for
depletion from the original cost would be proper in determining whether there had been a profit
on a sale of property. It held that no deduction from original cost should be made here, because
of the nature of oil mining properties. The deduction for depletion was, in its opinion, wrong
because oil properties are, in essence, merely the right to extract from controlled land such oil
as the owner of the right can find and reduce to possession; because the existence of oil in any
parcel of land is dependent upon the movement which the oil makes from time to time under the
surface, and because whether there is oil in place which can be reduced to possession, and if
so, how much, cannot be definitely determined. It held that, in the case at bar, the right to
explore for and take out oil may actually have been more valuable at the time of the sale than at
the time of the purchase, and that, for this reason, the removal of the oil by plaintiff during the
years of operation cannot be said to have depleted the capital. It held that the depreciation was
not deductible, because wear and tear of equipment was an expense or incident of the
business.
We are of opinion that the revenue acts should be construed as requiring deductions for both
depreciation and depletion when determining the original cost of oil properties sold. Congress,
in providing that the basis for determining gain or loss should be the cost or the 1913 value, was
not attempting to provide an exclusive formula for the computation. [Footnote 3] The
depreciation charge permitted as a deduction from the gross income in determining the taxable
income of a business for any year represents the reduction, during the year, of the capital
assets through wear and tear of the plant used. The amount of the allowance for depreciation is
the sum which should be set aside for the taxable year, in order that, at the end of the useful life
of the plant in the business, the aggregate of the sums set aside will (with the salvage value)
suffice to provide an amount equal to the original cost. The theory underlying this allowance for
depreciation is that, by using up the plant, a gradual sale is made of it. The depreciation
charged is the measure of the cost of the part which has been sold. When the plant is disposed
of after years of use, the thing then sold is not the whole thing originally acquired. The amount
of the depreciation must be deducted from the original cost of the whole in order to determine
the cost of that disposed of in the final sale of properties. [Footnote 4] Any other construction
would permit a double deduction for the loss of the same capital assets.
Such being the rule applicable to manufacturing and mercantile businesses, no good reason
appears why the business of mining should be treated differently. The reasons urged for
refusing to apply the rule specifically to oil mining properties seem to us unsound. If the
equipment had been used by its owner on the oil properties owned by another, it would hardly
be contended that the depreciation through wear and tear resulting from its use should be
ignored in determining, on a sale of the equipment, whether its owner had made a gain or a
144

loss. The fact that the equipment sold is owned by the person who owned the mining rights, like
the fact that it is used in one class of mining, rather than in another, may have an important
bearing both upon the price realized on the sale and upon the rate of depreciation which should
be allowed, but these facts cannot affect the question whether the part which has been
theretofore consumed by use shall be ignored in determining whether a sale of what remains
has resulted in a loss or a gain.
The depletion charge permitted as a deduction from the gross income in determining the taxable
income of mines for any year represents the reduction in the mineral contents of the reserves
from which the product is taken. The reserves are recognized as wasting assets. The depletion
effected by operation is likened to the using up of raw material in making the product of a
manufacturing establishment. As the cost of the raw material must be deducted from the gross
income before the net income can be determined, so the estimated cost of the part of the
reserve used up is allowed. The fact that the reserve is hidden from sight presents difficulties in
making an estimate of the amount of the deposits. The actual quantity can rarely be measured.
It must be approximated. And because the quantity originally the reserve is not actually known,
the percentage of the whole withdrawn in any year, and hence the appropriate depletion charge,
is necessarily a rough estimate. But Congress concluded, in the light of experience, that it was
better to act upon a rough estimate than to ignore the fact of depletion.
The Corporation Tax Law of 1909 had failed to provide for any deduction on account of the
depletion of mineral reserves. Stratton's Independence v. Howbert, 231 U. S. 339; Von
Baumbach v. Sargent Land Co., 242 U. S. 503; United States v. Biwabik Mining Co., 247 U. S.
116; Goldfield Consolidated Mines Co. v. Scott, 247 U.S. 126. The resulting hardship to
operators of mines induced Congress to make provision in the Revenue Law of 1913 and all
later acts for some deduction on account of depletion in determining the amount of the taxable
income from mines. [Footnote 5] It is not lightly to be assumed that Congress intended the fact
to be ignored in determining whether there was a loss or a gain on a sale of the mining
properties. The proviso limiting the amount of the deduction for depletion to the amount of the
capital invested shows that the deduction is to be regarded as a return of capital, not as a
special bonus for enterprise and willingness to assume risks. It is argued that, because oil is a
fugacious mineral, it cannot be known that the reserve has been diminished by the operation of
wells. Perhaps some land may be discovered which, like the widow's cruse, will afford an
inexhaustible supply of oil. But the common experience of man has been that oil wells, and the
territory in which they are sunk, become exhausted in time. Congress, in providing for the
deduction for depletion of oil wells, acted on that experience. Compare Lynch v. AlworthStephens Co., 267 U. S. 364. In essence, the deduction for depletion does not differ from the
deduction for depreciation.
The Court of Claims erred in holding that no deduction should be made from the original cost on
account of depreciation and depletion, but it does not follow that the amount deducted by the
Commissioner was the correct one. The aggregate for depreciation and depletion claimed by
Ludey in the income tax returns for the years 1913, 1914, 1915, and 1916, and allowed, was
only $5,156.
He insists that more cannot be deducted from the original cost in making the return for 1917.
The contention is unsound. The amount of the gain on the sale is not dependent on the amount
claimed in earlier years. If in any year he has failed to claim, or has been denied, the amount to
which he was entitled, rectification of the error must be sought through a review of the action of
the bureau for that year. He cannot choose the year in which he will take a reduction. On the
other hand, we cannot accept the government's contention that the full amount of depreciation
and depletion sustained, whether allowable by law as a deduction from gross income in past
years or not, must be deducted from cost in ascertaining gain or loss. Congress doubtless
145

intended that the deduction to be made from the original cost should be the aggregate amount
which the taxpayer was entitled to deduct in the several years.
The findings do not enable us to determine what that aggregate is. The sale included several
properties purchased at different times. The deduction allowable in the several years for each of
the properties is not found. Under the Act of 1913, the full amount of the depletion was not
necessarily deductible. In order that the amount of the gain in 1917 may be determined in the
light of such facts, the case is remanded for further proceedings in accordance with this opinion.
Reversed.
[Footnote 1]
Some of the properties were purchased before March 1, 1913. As to these the term "cost" is
used, throughout the opinion, as meaning their value as of March 1, 1913, that value being
higher than the original cost.
[Footnote 2]
The 1924 Act, June 2, 1924, 202(b), 43 Stat. 253, 255, provided that in computing gain or loss
from sales, adjustment should be made for items of exhaustion, wear and tear, and depletion
"previously allowed with respect to such property."See Regulations 65, Arts. 1591-1603. The
1926 Act (Act. Feb. 26, 1926, 202(b), 44 Stat. 9, 11, 12) has a similar provision with respect to
deductions "allowable . . . under this Act or prior income tax laws." See Regulations 69, Art.
1561.
[Footnote 3]
See Appeal of Even Realty Co., 1 B.T.A. 355. Compare Appeal of Steiner Coal Co., 1 B.T.A.
821; Appeal of W. W. Carter Co., 1 B.T.A. 849; Appeal of Keighley Mfg. Co., 2 B.T.A. 10.
[Footnote 4]
Under regulations of the bureau the amount of the year's depreciation is required to be fixed in
accordance with a reasonably consistent plan, and it must, in order to be allowed, have been
entered on the books of the business either as a deduction from the book value of the plant or
as a credit to a depreciation reserve account. See Regulations 33, revised, Art. 159;
Regulations 45, Art. 169; Regulations 62, Art. 169; Regulations 65, Art. 169; Regulations 69, Art.
169. In either event, it would be reflected in the annual balance sheet. After the total of such
credits equals the original cost, no further deduction is allowed.
[Footnote 5]
The bureau requires that taxpayers claiming depletion deductions shall keep a ledger account in
which deductions claimed are credited against the cost of the property, or that a depletion
reserve account be set up.
Republic of the Philippines
SUPREME COURT
Manila
146

EN BANC
G.R. No. L-25043

April 26, 1968

ANTONIO ROXAS, EDUARDO ROXAS and ROXAS Y CIA., in their own respective behalf
and as judicial co-guardians of JOSE ROXAS, petitioners,
vs.
COURT OF TAX APPEALS and COMMISSIONER OF INTERNAL REVENUE, respondents.
Leido, Andrada, Perez and Associates for petitioners.
Office of the Solicitor General for respondents.
BENGZON, J.P., J.:
Don Pedro Roxas and Dona Carmen Ayala, Spanish subjects, transmitted to their grandchildren
by hereditary succession the following properties:
(1) Agricultural lands with a total area of 19,000 hectares, situated in the municipality of
Nasugbu, Batangas province;
(2) A residential house and lot located at Wright St., Malate, Manila; and
(3) Shares of stocks in different corporations.
To manage the above-mentioned properties, said children, namely, Antonio Roxas, Eduardo
Roxas and Jose Roxas, formed a partnership called Roxas y Compania.
AGRICULTURAL LANDS
At the conclusion of the Second World War, the tenants who have all been tilling the lands in
Nasugbu for generations expressed their desire to purchase from Roxas y Cia. the parcels
which they actually occupied. For its part, the Government, in consonance with the
constitutional mandate to acquire big landed estates and apportion them among landless
tenants-farmers, persuaded the Roxas brothers to part with their landholdings. Conferences
were held with the farmers in the early part of 1948 and finally the Roxas brothers agreed to sell
13,500 hectares to the Government for distribution to actual occupants for a price of
P2,079,048.47 plus P300,000.00 for survey and subdivision expenses.
It turned out however that the Government did not have funds to cover the purchase price, and
so a special arrangement was made for the Rehabilitation Finance Corporation to advance to
Roxas y Cia. the amount of P1,500,000.00 as loan. Collateral for such loan were the lands
proposed to be sold to the farmers. Under the arrangement, Roxas y Cia. allowed the farmers to
buy the lands for the same price but by installment, and contracted with the Rehabilitation
Finance Corporation to pay its loan from the proceeds of the yearly amortizations paid by the
farmers.
In 1953 and 1955 Roxas y Cia. derived from said installment payments a net gain of P42,480.83
and P29,500.71. Fifty percent of said net gain was reported for income tax purposes as gain on
the sale of capital asset held for more than one year pursuant to Section 34 of the Tax Code.
RESIDENTIAL HOUSE
147

During their bachelor days the Roxas brothers lived in the residential house at Wright St.,
Malate, Manila, which they inherited from their grandparents. After Antonio and Eduardo got
married, they resided somewhere else leaving only Jose in the old house. In fairness to his
brothers, Jose paid to Roxas y Cia. rentals for the house in the sum of P8,000.00 a year.
ASSESSMENTS
On June 17, 1958, the Commissioner of Internal Revenue demanded from Roxas y Cia the
payment of real estate dealer's tax for 1952 in the amount of P150.00 plus P10.00 compromise
penalty for late payment, and P150.00 tax for dealers of securities for 1952 plus P10.00
compromise penalty for late payment. The assessment for real estate dealer's tax was based on
the fact that Roxas y Cia. received house rentals from Jose Roxas in the amount of P8,000.00.
Pursuant to Sec. 194 of the Tax Code, an owner of a real estate who derives a yearly rental
income therefrom in the amount of P3,000.00 or more is considered a real estate dealer and is
liable to pay the corresponding fixed tax.
The Commissioner of Internal Revenue justified his demand for the fixed tax on dealers of
securities against Roxas y Cia., on the fact that said partnership made profits from the purchase
and sale of securities.
In the same assessment, the Commissioner assessed deficiency income taxes against the
Roxas Brothers for the years 1953 and 1955, as follows:

Antonio Roxas
Eduardo Roxas
Jose Roxas

1953
P7,010.00
7,281.00
6,323.00

1955
P5,813.00
5,828.00
5,588.00

The deficiency income taxes resulted from the inclusion as income of Roxas y Cia. of the
unreported 50% of the net profits for 1953 and 1955 derived from the sale of the Nasugbu farm
lands to the tenants, and the disallowance of deductions from gross income of various business
expenses and contributions claimed by Roxas y Cia. and the Roxas brothers. For the reason
that Roxas y Cia. subdivided its Nasugbu farm lands and sold them to the farmers on
installment, the Commissioner considered the partnership as engaged in the business of real
estate, hence, 100% of the profits derived therefrom was taxed.
The following deductions were disallowed:
ROXAS Y CIA.:
1953
Tickets for Banquet in honor of
S. Osmea
Gifts of San Miguel beer

P 40.00
28.00

Contributions to
Philippine Air Force Chapel

100.00

Manila Police Trust Fund

150.00

Philippines Herald's fund for

100.00
148

Manila's neediest families


1955
Contributions to Contribution to
Our Lady of Fatima Chapel,
FEU

50.00

ANTONIO ROXAS:
1953
Contributions to
Pasay City Firemen Christmas
Fund

25.00

Pasay City Police Dept. X'mas fund

50.00

1955
Contributions to
Baguio City Police Christmas fund

25.00

Pasay City Firemen Christmas fund

25.00

Pasay City Police Christmas fund

50.00

EDUARDO ROXAS:
1953
Contributions to
Hijas de Jesus' Retiro de Manresa

450.00

Philippines Herald's fund for


Manila's neediest families

100.00

Contributions to Philippines
Herald's fund for Manila's
neediest families

120.00

1955

JOSE ROXAS:
1955
Contributions to Philippines
Herald's fund for Manila's
neediest families

120.00

The Roxas brothers protested the assessment but inasmuch as said protest was denied, they
instituted an appeal in the Court of Tax Appeals on January 9, 1961. The Tax Court heard the
appeal and rendered judgment on July 31, 1965 sustaining the assessment except the demand
for the payment of the fixed tax on dealer of securities and the disallowance of the deductions
149

for contributions to the Philippine Air Force Chapel and Hijas de Jesus' Retiro de Manresa. The
Tax Court's judgment reads:
WHEREFORE, the decision appealed from is hereby affirmed with respect to petitioners
Antonio Roxas, Eduardo Roxas, and Jose Roxas who are hereby ordered to pay the
respondent Commissioner of Internal Revenue the amounts of P12,808.00, P12,887.00
and P11,857.00, respectively, as deficiency income taxes for the years 1953 and 1955,
plus 5% surcharge and 1% monthly interest as provided for in Sec. 51(a) of the Revenue
Code; and modified with respect to the partnership Roxas y Cia. in the sense that it
should pay only P150.00, as real estate dealer's tax. With costs against petitioners.
Not satisfied, Roxas y Cia. and the Roxas brothers appealed to this Court. The Commissioner of
Internal Revenue did not appeal.
The issues:
(1) Is the gain derived from the sale of the Nasugbu farm lands an ordinary gain, hence
100% taxable?
(2) Are the deductions for business expenses and contributions deductible?
(3) Is Roxas y Cia. liable for the payment of the fixed tax on real estate dealers?
The Commissioner of Internal Revenue contends that Roxas y Cia. could be considered a real
estate dealer because it engaged in the business of selling real estate. The business activity
alluded to was the act of subdividing the Nasugbu farm lands and selling them to the farmersoccupants on installment. To bolster his stand on the point, he cites one of the purposes of
Roxas y Cia. as contained in its articles of partnership, quoted below:
4. (a) La explotacion de fincas urbanes pertenecientes a la misma o que pueden
pertenecer a ella en el futuro, alquilandoles por los plazos y demas condiciones, estime
convenientes y vendiendo aquellas que a juicio de sus gerentes no deben conservarse;
The above-quoted purpose notwithstanding, the proposition of the Commissioner of Internal
Revenue cannot be favorably accepted by Us in this isolated transaction with its peculiar
circumstances in spite of the fact that there were hundreds of vendees. Although they paid for
their respective holdings in installment for a period of ten years, it would nevertheless not make
the vendor Roxas y Cia. a real estate dealer during the ten-year amortization period.
It should be borne in mind that the sale of the Nasugbu farm lands to the very farmers who tilled
them for generations was not only in consonance with, but more in obedience to the request
and pursuant to the policy of our Government to allocate lands to the landless. It was the
bounden duty of the Government to pay the agreed compensation after it had persuaded Roxas
y Cia. to sell its haciendas, and to subsequently subdivide them among the farmers at very
reasonable terms and prices. However, the Government could not comply with its duty for lack
of funds. Obligingly, Roxas y Cia. shouldered the Government's burden, went out of its way and
sold lands directly to the farmers in the same way and under the same terms as would have
been the case had the Government done it itself. For this magnanimous act, the municipal
council of Nasugbu passed a resolution expressing the people's gratitude.
The power of taxation is sometimes called also the power to destroy. Therefore it should be
exercised with caution to minimize injury to the proprietary rights of a taxpayer. It must be
150

exercised fairly, equally and uniformly, lest the tax collector kill the "hen that lays the golden
egg". And, in order to maintain the general public's trust and confidence in the Government this
power must be used justly and not treacherously. It does not conform with Our sense of justice
in the instant case for the Government to persuade the taxpayer to lend it a helping hand and
later on to penalize him for duly answering the urgent call.
In fine, Roxas y Cia. cannot be considered a real estate dealer for the sale in question. Hence,
pursuant to Section 34 of the Tax Code the lands sold to the farmers are capital assets, and the
gain derived from the sale thereof is capital gain, taxable only to the extent of 50%.
DISALLOWED DEDUCTIONS
Roxas y Cia. deducted from its gross income the amount of P40.00 for tickets to a banquet
given in honor of Sergio Osmena and P28.00 for San Miguel beer given as gifts to various
persons. The deduction were claimed as representation expenses. Representation expenses
are deductible from gross income as expenditures incurred in carrying on a trade or business
under Section 30(a) of the Tax Code provided the taxpayer proves that they are reasonable in
amount, ordinary and necessary, and incurred in connection with his business. In the case at
bar, the evidence does not show such link between the expenses and the business of Roxas y
Cia. The findings of the Court of Tax Appeals must therefore be sustained.
The petitioners also claim deductions for contributions to the Pasay City Police, Pasay City
Firemen, and Baguio City Police Christmas funds, Manila Police Trust Fund, Philippines
Herald's fund for Manila's neediest families and Our Lady of Fatima chapel at Far Eastern
University.
The contributions to the Christmas funds of the Pasay City Police, Pasay City Firemen and
Baguio City Police are not deductible for the reason that the Christmas funds were not spent for
public purposes but as Christmas gifts to the families of the members of said entities. Under
Section 39(h), a contribution to a government entity is deductible when used exclusively for
public purposes. For this reason, the disallowance must be sustained. On the other hand, the
contribution to the Manila Police trust fund is an allowable deduction for said trust fund belongs
to the Manila Police, a government entity, intended to be used exclusively for its public
functions.
The contributions to the Philippines Herald's fund for Manila's neediest families were disallowed
on the ground that the Philippines Herald is not a corporation or an association contemplated in
Section 30 (h) of the Tax Code. It should be noted however that the contributions were not made
to the Philippines Herald but to a group of civic spirited citizens organized by the Philippines
Herald solely for charitable purposes. There is no question that the members of this group of
citizens do not receive profits, for all the funds they raised were for Manila's neediest families.
Such a group of citizens may be classified as an association organized exclusively for charitable
purposes mentioned in Section 30(h) of the Tax Code.
Rightly, the Commissioner of Internal Revenue disallowed the contribution to Our Lady of
Fatima chapel at the Far Eastern University on the ground that the said university gives
dividends to its stockholders. Located within the premises of the university, the chapel in
question has not been shown to belong to the Catholic Church or any religious organization. On
the other hand, the lower court found that it belongs to the Far Eastern University, contributions
to which are not deductible under Section 30(h) of the Tax Code for the reason that the net
income of said university injures to the benefit of its stockholders. The disallowance should be
sustained.
151

Lastly, Roxas y Cia. questions the imposition of the real estate dealer's fixed tax upon it,
because although it earned a rental income of P8,000.00 per annum in 1952, said rental income
came from Jose Roxas, one of the partners. Section 194 of the Tax Code, in considering as real
estate dealers owners of real estate receiving rentals of at least P3,000.00 a year, does not
provide any qualification as to the persons paying the rentals. The law, which states:
. . . "Real estate dealer" includes any person engaged in the business of buying, selling,
exchanging, leasing or renting property on his own account as principal and holding
himself out as a full or part-time dealer in real estate or as an owner of rental property or
properties rented or offered to rent for an aggregate amount of three thousand pesos or
more a year: . . . (Emphasis supplied) .
is too clear and explicit to admit construction. The findings of the Court of Tax Appeals or, this
point is sustained.
To Summarize, no deficiency income tax is due for 1953 from Antonio Roxas, Eduardo Roxas
and Jose Roxas. For 1955 they are liable to pay deficiency income tax in the sum of P109.00,
P91.00 and P49.00, respectively, computed as follows: *
ANTONIO ROXAS
Net income per return

P315,476.59

Add: 1/3 share, profits in Roxas y


Cia.

P 153,249.15

Less amount declared

146,135.46

Amount understated

P 7,113.69

Contributions disallowed

115.00
P 7,228.69

Less 1/3 share of contributions


amounting to P21,126.06
disallowed from partnership but
allowed to partners

7,042.02

Net income per review

P315,663.26

Less: Exemptions

4,200.00

Net taxable income

P311,463.26

Tax due

154,169.00

Tax paid

154,060.00

Deficiency

186.67

P 109.00
==========
EDUARDO ROXAS
152

P
304,166.92

Net income per return


Add: 1/3 share, profits in Roxas y
Cia

P 153,249.15

Less profits declared

146,052.58

Amount understated

P 7,196.57

Less 1/3 share in contributions


amounting to P21,126.06
disallowed from partnership but
allowed to partners

7,042.02

Net income per review

155.55
P304,322.47

Less: Exemptions

4,800.00

Net taxable income

P299,592.47

Tax Due

P147,250.00

Tax paid

147,159.00

Deficiency

P91.00
===========
JOSE ROXAS

Net income per return


Add: 1/3 share, profits in Roxas y
Cia.
Less amount reported

P222,681.76
P153,429.15
146,135.46

Amount understated

7,113.69

Less 1/3 share of contributions


disallowed from partnership but
allowed as deductions to partners

7,042.02

Net income per review

71.67
P222,753.43

Less: Exemption

1,800.00

Net income subject to tax

P220,953.43

Tax due

P102,763.00

Tax paid

102,714.00

Deficiency
P 49.00
153

===========
WHEREFORE, the decision appealed from is modified. Roxas y Cia. is hereby ordered to pay
the sum of P150.00 as real estate dealer's fixed tax for 1952, and Antonio Roxas, Eduardo
Roxas and Jose Roxas are ordered to pay the respective sums of P109.00, P91.00 and P49.00
as their individual deficiency income tax all corresponding for the year 1955. No costs. So
ordered.
Reyes, J.B.L., Dizon, Makalintal, Sanchez, Castro, Angeles and Fernando, JJ., concur.
Zaldivar, J., took no part.
Concepcion, C.J., is on leave.

154

You might also like